You are on page 1of 169

The Canadian Mathematical Society

in collaboration with

The CENTRE for EDUCATION


in MATHEMATICS and COMPUTING

The
Canadian Open
Mathematics Challenge

Wednesday, November 29, 2000

1
Time: 2 hours © 2000 Canadian Mathematical Society
2

Calculators are NOT permitted.

Do not open this booklet until instructed to do so.


There are two parts to the paper.

PART A
This part of the paper consists of 8 questions, each worth 5 marks. You can earn full value for each
question by entering the correct answer in the space provided. Any work you do in obtaining an answer
will be considered for part marks if you do not have the correct answer, provided that it is done in the
space allocated to that question in your answer booklet.

PART B
This part of the paper consists of 4 questions, each worth 10 marks. Finished solutions must be written
in the appropriate location in the answer booklet. Rough work should be done separately. If you require
extra pages for your finished solutions, foolscap will be provided by your supervising teacher. Any extra
papers should be placed inside your answer booklet.
Marks are awarded for completeness, clarity, and style of presentation. A correct solution poorly
presented will not earn full marks.

NOTE: At the completion of the contest, insert the information sheet inside the answer booklet.
Canadian Open Mathematics Challenge

NOTE: 1. Please read the instructions on the front cover of this booklet.
2. Write solutions in the answer booklet provided.
3. It is expected that all calculations and answers will be expressed as exact numbers such
as 4 p, 2 + 7 , etc.
4. Calculators are not allowed.

PART A

a
1. An operation “ D ” is defined by a D b = 1 – , b π 0.
b
What is the value of (1 D 2)D(3 D 4) ?

2. The sequence 9, 18, 27, 36, 45, 54, … consists of successive multiples of 9. This sequence is then
altered by multiplying every other term by –1, starting with the first term, to produce the new
sequence – 9, 18, – 27, 36, – 45, 54,... . If the sum of the first n terms of this new sequence is 180,
determine n.

3. The symbol n! is used to represent the product n(n – 1)(n – 2) L(3)(2)(1) .


( )( )( )( )
For example, 4!= 4(3)(2)(1) . Determine n such that n!= 215 36 53 72 (11)(13).

4. The symbol Î x û means the greatest integer less than or equal to x. For example,

Î5.7û = 5 , Îp û = 3 and Î4 û = 4 .
Calculate the value of the sum

Î 1û + Î 2 û + Î 3 û + Î 4 û + L + Î 48 û + Î 49 û + Î 50 û .

5. How many five-digit positive integers have the property that the product of their digits is 2000?

6. Solve the equation 4 ÊË16sin x ˆ¯ = 2 6 sin x , for 0 £ x £ 2 p .


2

7. The sequence of numbers …, a–3 , a– 2 , a–1, a0 , a1, a2 , a3 , … is defined by an – (n + 1)a2 – n = (n + 3)2 ,


for all integers n. Calculate a0 .

8. In the diagram, D ABC is equilateral and the radius of its


E C D
inscribed circle is 1. A larger circle is drawn through the vertices
of the rectangle ABDE . What is the diameter of the larger circle?

A B
PART B

1. Triangle ABC has vertices A(0, 0) , B(9, 0) and C(0, 6) . The points P and Q lie on side AB such
that AP = PQ = QB . Similarly, the points R and S lie on side AC so that AR = RS = SC .
The vertex C is joined to each of the points P and Q. In the same way, B is joined to R and S.
(a) Determine the equation of the line through the points R and B.
(b) Determine the equation of the line through the points P and C.
(c) The line segments PC and RB intersect at X, and the line segments QC and SB intersect at Y.
Prove that the points A, X and Y lie on the same straight line.

2. In D ABC , the points D, E and F are on sides BC , CA and AB , A


respectively, such that – AFE = – BFD, – BDF = –CDE, and
E
–CED = – AEF . F
(a) Prove that – BDF = – BAC .
(b) If AB = 5, BC = 8 and CA = 7, determine the length of BD.

B D C

3. (a) Alphonse and Beryl are playing a game, starting with the
geometric shape shown in Figure 1. Alphonse begins the
game by cutting the original shape into two pieces along
one of the lines. He then passes the piece containing the Figure 1
black triangle to Beryl, and discards the other piece.
Beryl repeats these steps with the piece she receives; that is to say, she cuts along the length of
a line, passes the piece containing the black triangle back to Alphonse, and discards the other
piece. This process continues, with the winner being the player who, at the beginning of his or
her turn, receives only the black triangle. Show, with justification, that there is always a
winning strategy for Beryl.

(b) Alphonse and Beryl now play a game with the same rules
as in (a), except this time they use the shape in Figure 2
and Beryl goes first. As in (a), cuts may only be made
along the whole length of a line in the figure. Is there a
strategy that Beryl can use to be guaranteed that she will
win? (Provide justification for your answer.)

Figure 2

4. A sequence t1, t2 , t3 , ..., tn of n terms is defined as follows:


t1 = 1, t2 = 4 , and tk = tk –1 + tk – 2 for k = 3, 4, ..., n .
Let T be the set of all terms in this sequence; that is, T = {t1, t2 , t3 , ..., tn } .
(a) How many positive integers can be expressed as the sum of exactly two distinct elements of
the set T ?
(b) How many positive integers can be expressed as the sum of exactly three distinct elements of
the set T ?
The Canadian Mathematical Society
in collaboration with

The CENTRE for EDUCATION


in MATHEMATICS and COMPUTING

The
Canadian Open
Mathematics Challenge
Wednesday, November 28, 2001

1 © 2001 Canadian Mathematical Society


Time: 2 hours
2

Calculators are NOT permitted.

Do not open this booklet until instructed to do so.


There are two parts to the paper.

PART A
This part of the paper consists of 8 questions, each worth 5 marks. You can earn full value for each
question by entering the correct answer in the space provided. Any work you do in obtaining an answer
will be considered for part marks if you do not have the correct answer, provided that it is done in the
space allocated to that question in your answer booklet.

PART B
This part of the paper consists of 4 questions, each worth 10 marks. Finished solutions must be written
in the appropriate location in the answer booklet. Rough work should be done separately. If you require
extra pages for your finished solutions, paper will be provided by your supervising teacher. Any extra
papers should be placed inside your answer booklet.
Marks are awarded for completeness, clarity, and style of presentation. A correct solution poorly
presented will not earn full marks.

NOTE: At the completion of the contest, insert the information sheet inside the answer booklet.
Canadian Open Mathematics Challenge

NOTE: 1. Please read the instructions on the front cover of this booklet.
2. Write solutions in the answer booklet provided.
3. It is expected that all calculations and answers will be expressed as exact numbers such
as 4 p, 2 + 7 , etc.
4. Calculators are not allowed.

1. An operation “ —” is defined by a — b = a + 3 .
2 b

What is the value of (2 — 0)—(0 —1) ?

2. In the given diagram, what is the value of x? D


5x∞
A C E G
3x∞ 2x∞
6x∞
F
4x∞

3. A regular hexagon is a six-sided figure which has all of its angles equal and all of its side lengths equal.
If P and Q are points on a regular hexagon which has a side length of 1, what is the maximum possible
length of the line segment PQ?

4. Solve for x:
( )
2 2 2 x = 4 x + 64 .

5. Triangle PQR is right-angled at Q and has side lengths PQ = 14 P


and QR = 48. If M is the midpoint of PR, determine the cosine of
M
– MQP .

Q R

tn – 1
6. The sequence of numbers t1, t2 , t3 , ... is defined by t1 = 2 and tn +1 = , for every positive integer n.
tn + 1
Determine the numerical value of t999 .

7. If a can be any positive integer and


2x + a = y
a+y= x
x+y=z
determine the maximum possible value for x + y + z.
8. The graph of the function y = g( x ) is shown. y

Determine the number of solutions of the equation


g( x ) – 1 = .
1
2
4

x
–4 –2 0 2 4

–2

–4

PART B
1. The triangular region T has its vertices determined by the intersections of the three lines x + 2y = 12,
x = 2 and y = 1.
(a) Determine the coordinates of the vertices of T, and show this region on the grid provided.
(b) The line x + y = 8 divides the triangular region T into a quadrilateral Q and a triangle R.
Determine the coordinates of the vertices of the quadrilateral Q.
(c) Determine the area of the quadrilateral Q.

2. (a) Alphonse and Beryl are playing a game, starting with a pack of 7 cards. Alphonse begins by
discarding at least one but not more than half of the cards in the pack. He then passes the remaining
cards in the pack to Beryl. Beryl continues the game by discarding at least one but not more than half
of the remaining cards in the pack. The game continues in this way with the pack being passed back
and forth between the two players. The loser is the player who, at the beginning of his or her turn,
receives only one card. Show, with justification, that there is always a winning strategy for Beryl.
(b) Alphonse and Beryl now play a game with the same rules as in (a), except this time they start with
a pack of 52 cards, and Alphonse goes first again. As in (a), a player on his or her turn must discard
at least one and not more than half of the remaining cards from the pack. Is there a strategy that
Alphonse can use to be guaranteed that he will win? (Provide justification for your answer.)

3. (a) If f ( x ) = x + 6 x + c , where c is an integer, prove that f (0) + f ( –1) is odd.


2

(b) Let g( x ) = x 3 + px 2 + qx + r , where p, q and r are integers. Prove that if g(0) and g( –1) are both odd,
then the equation g( x ) = 0 cannot have three integer roots.

4. Triangle ABC is isosceles with AB = AC = 5 and BC = 6. Point D A


lies on AC and P is the point on BD so that – APC = 90∞ . If
– ABP = – BCP , determine the ratio AD:DC.
D

B C
The Canadian Mathematical Society
in collaboration with

The CENTRE for EDUCATION


in MATHEMATICS and COMPUTING

The
Canadian Open
Mathematics Challenge
Wednesday, November 27, 2002

1 © 2002 Canadian Mathematical Society


Time: 2 hours
2

Calculators are NOT permitted.

Do not open this booklet until instructed to do so.


There are two parts to the paper.

PART A
This part of the paper consists of 8 questions, each worth 5 marks. You can earn full value for each
question by entering the correct answer in the space provided. Any work you do in obtaining an answer
will be considered for part marks if you do not have the correct answer, provided that it is done in the
space allocated to that question in your answer booklet.

PART B
This part of the paper consists of 4 questions, each worth 10 marks. Finished solutions must be written
in the appropriate location in the answer booklet. Rough work should be done separately. If you require
extra pages for your finished solutions, paper will be provided by your supervising teacher. Any extra
papers should be placed inside your answer booklet.
Marks are awarded for completeness, clarity, and style of presentation. A correct solution poorly
presented will not earn full marks.

NOTE: At the completion of the contest, insert the information sheet inside the answer booklet.
Canadian Open Mathematics Challenge

NOTE: 1. Please read the instructions on the front cover of this booklet.
2. Write solutions in the answer booklet provided.
3. It is expected that all calculations and answers will be expressed as exact numbers such
as 4 π, 2 + 7 , etc.
4. Calculators are not allowed.

PART A
1. In triangle PQR, F is the point on QR so that PF is perpendicular P
to QR. If PR = 13, RF = 5 , and FQ = 9 , what is the perimeter
of ∆PQR ?
13

Q 9 F 5 R

2. If x + y = 4 and xy = −12 , what is the value of x 2 + 5 xy + y 2 ?

3. A regular pentagon is a five-sided figure which has all of its A


angles equal and all of its side lengths equal. In the diagram, R
TREND is a regular pentagon, PEA is an equilateral triangle, and
P
OPEN is a square. Determine the size of ∠ EAR . E
T

O N
D

4. In a sequence of numbers, the sum of the first n terms is equal to 5 n 2 + 6 n . What is the sum of the
3rd, 4th and 5th terms in the original sequence?

5. If m and n are non-negative integers with m < n , we define m∇n to be the sum of the integers from
m to n, including m and n. For example, 5∇8 = 5 + 6 + 7 + 8 = 26 .

For every positive integer a, the numerical value of


[(2a − 1)∇(2a + 1)] is the same. Determine this
[(a − 1)∇(a + 1)]
value.

6. Two mirrors meet at an angle of 30o at the point V. A beam of


A
light, from a source S, travels parallel to one mirror and strikes S
the other mirror at point A, as shown. After a number of
reflections, the beam comes back to S. If SA and AV are both 30°
1 metre in length, determine the total distance travelled by the V
beam.

7. N is a five-digit positive integer. A six-digit integer P is constructed by placing a 1 at the right-hand


end of N. A second six-digit integer Q is constructed by placing a 1 at the left-hand end of N. If P is
three times Q, determine the value of N.
8. Suppose that M is an integer with the property that if x is randomly chosen from the set
{1, 2, 3,K, 999,1000} , the probability that x is a divisor of M is 100
1
. If M ≤ 1000 , determine the
maximum possible value of M.

PART B

1. Square ABCD has vertices A(0, 0) , B(0, 8) , C (8, 8) , and D(8, 0) . The points P (0, 5) and Q(0, 3) are on
side AB , and the point F (8,1) is on side CD.
(a) What is the equation of the line through Q parallel to the line through P and F?
(b) If the line from part (a) intersects AD at the point G, what is the equation of the line through F
and G?
(c) The centre of the square is the point H ( 4, 4 ) . Determine the equation of the line through H
perpendicular to FG.
(d) A circle is drawn with centre H that is tangent to the four sides of the square. Does this circle
intersect the line through F and G? Justify your answer. (A sketch is not sufficient justification.)

2. (a) Let A and B be digits (that is, A and B are integers between 0 and 9 inclusive). If the product of
the three-digit integers 2 A5 and 13B is divisible by 36, determine with justification the four
possible ordered pairs ( A, B) .
(b) An integer n is said to be a multiple of 7 if n = 7 k for some integer k.
(i) If a and b are integers and 10 a + b = 7 m for some integer m, prove that a − 2b is a multiple
of 7.
(ii) If c and d are integers and 5c + 4 d is a multiple of 7, prove that 4c − d is also a multiple
of 7.

3. There are some marbles in a bowl. Alphonse, Beryl and Colleen each take turns removing one or two
marbles from the bowl, with Alphonse going first, then Beryl, then Colleen, then Alphonse again, and
so on. The player who takes the last marble from the bowl is the loser, and the other two players are
the winners.
(a) If the game starts with 5 marbles in the bowl, can Beryl and Colleen work together and force
Alphonse to lose?
(b) The game is played again, this time starting with N marbles in the bowl. For what values of N
can Beryl and Colleen work together and force Alphonse to lose?

4. Triangle DEF is acute. Circle C1 is drawn with DF as its D


diameter and circle C2 is drawn with DE as its diameter.
Points Y and Z are on DF and DE respectively so that EY and C1
FZ are altitudes of ∆DEF . EY intersects C1 at P, and FZ C2
intersects C2 at Q. EY extended intersects C1 at R, and FZ
extended intersects C2 at S. Prove that P, Q, R, and S are
E F
concyclic points.
The Canadian Mathematical Society
in collaboration with

The CENTRE for EDUCATION


in MATHEMATICS and COMPUTING

The
Canadian Open
Mathematics Challenge
Wednesday, November 26, 2003

1 © 2003 Canadian Mathematical Society


Time: 2 hours
2

Calculators are NOT permitted.

Do not open this booklet until instructed to do so.


There are two parts to the paper.

PART A
This part of the paper consists of 8 questions, each worth 5. You can earn full value for each question
by entering the correct answer in the space provided. Any work you do in obtaining an answer will be
considered for part marks if you do not have the correct answer, provided that it is done in the space
allocated to that question in your answer booklet.

PART B
This part of the paper consists of 4 questions, each worth 10. Finished solutions must be written in the
appropriate location in the answer booklet. Rough work should be done separately. If you require extra
pages for your finished solutions, paper will be provided by your supervising teacher. Any extra papers
should be placed inside your answer booklet.
Marks are awarded for completeness, clarity, and style of presentation. A correct solution poorly
presented will not earn full marks.

NOTE: At the completion of the contest, insert the information sheet inside the answer booklet.
Canadian Open Mathematics Challenge

NOTE: 1. Please read the instructions on the front cover of this booklet.
2. Write solutions in the answer booklet provided.
3. It is expected that all calculations and answers will be expressed as exact numbers such
as 4 π, 2 + 7 , etc.
4. Calculators are not allowed.

PART A
1. Jeff, Gareth and Ina all share the same birthday. Gareth is one year older than Jeff, and Ina is two years
older than Gareth. This year the sum of their ages is 118. How old is Gareth?

2. The point ( 4, − 2 ) is reflected in the x-axis. The resulting


point is then reflected in the line with equation y = x . What y y=x
are the coordinates of the final point?

x
(4, – 2)

3. A circle of radius 1 is centred at the origin. Two particles start y


moving at the same time from the point (1 , 0) and move around
the circle in opposite directions. One of the particles moves
counterclockwise with constant speed v and the other moves
clockwise with constant speed 3v. After leaving (1 , 0) , the two x
(1, 0)
particles meet first at point P, and continue until they meet next
at point Q. Determine the coordinates of the point Q.

4. Two different numbers are chosen at random from the set {0, 1, 2, 3, 4} . What is the probability that
their sum is greater than their product?

5. In the diagram, square ABCD has a side length of 6. Circular A B


arcs of radius 6 are drawn with centres B and D. What is the area
of the shaded region?

D C

6. The symbol a  means the greatest integer less than or equal to a.


For example, 5.7  = 5, 4  = 4 and −4.2  = −5 .
 3 4 
Determine all values of x for which   +   = 5 .
x x
7. Each of the points P ( 4 , 1) , Q( 7 ,−8) and R(10, 1) is the midpoint of a radius of the circle C. Determine
the length of the radius of circle C.

8. Determine the number of triples ( k, l, m) of positive integers such that


k + l + m = 97
4 k 5l 6m
+ + = 82
5 6 7

PART B
1. In the diagram shown, whole numbers are to be k
placed in the ten circles so that the sum of the
numbers in the circles along any of the ten straight
lines is 15. For example, a + g + k = 15 and i
e + i = 15 .
(a) If k = 2 and e = 5 , fill in the whole numbers g h e
that go in all of the circles in the diagram. f
(b) Suppose that k = 2 and the value of e is d
unknown.
(i) Find a formula for each of b and c in terms a b c
of e. A clearly labelled diagram is sufficient
explanation.
(ii) Show that e must be equal to 5.
(c) Suppose now that k = x , where x is unknown. Prove that e must still be equal to 5.

2. A barn has a foundation in the shape of a trapezoid, with three sides D 6 A


of length 6 m, and one side of length 12 m, as shown.
(a) Determine each of the interior angles in the trapezoid. 6 6
(b) Chuck the Llama is attached by a chain to a point on the outside
wall of the barn. Chuck is smarter than the average llama, and so C 12 B
realizes that he can always reach the area between the barn and
where the chain is fully extended.
(i) If Chuck is attached at the point A with a chain of length 8 m, what is the area outside the barn
that Chuck can reach?
(ii) If Chuck is attached at some point P along the wall between A and B with a chain of length
15 m, determine the location of P which restricts Chuck to the minimum area.

3. (a) In the diagram, the two circles C1 and C2 have a P


A
common chord AB. Point P is chosen on C1 so that it is
C1
outside C2 . Lines PA and PB are extended to cut C2 at
X
X and Y, respectively. If AB = 6 , PA = 5 , PB = 7 and
AX = 16, determine the length of XY. B
C2

Y
(b) Two circles C3 and C4 have a common chord GH. Point Q is chosen on C3 so that it is outside
C4 . Lines QG and QH are extended to cut C4 at V and W, respectively. Show that, no matter
where Q is chosen, the length of VW is constant.

over ...
Canadian Open
Mathematics
Challenge
(English)

2003
4. The polynomial equation x 3 − 6 x 2 + 5 x − 1 = 0 has three real roots a, b and c.
(a) Determine the value of a 5 + b 5 + c 5 .
(b) If a < b < c , show that c 2004 is closer to its nearest integer than c 2003 is to its nearest integer.
The Canadian Mathematical Society
in collaboration with

The CENTRE for EDUCATION


in MATHEMATICS and COMPUTING
presents the

Canadian Open
Mathematics Challenge
Wednesday, November 24, 2004

Time: 2 12 hours 2004


c Canadian Mathematical Society

Calculators are NOT permitted.

Do not open this booklet until instructed to do so.


There are two parts to this paper.

PART A
This part of the paper consists of 8 questions, each worth 5 marks. You can earn full value for each
question by entering the correct answer in the space provided. Any work you do in obtaining an answer
will be considered for part marks if you do not have the correct answer, provided that it is done in the
space allocated to that question in your answer booklet.

PART B
This part of the paper consists of 4 questions, each worth 10 marks. Finished solutions must be written
in the appropriate location in the answer booklet. Rough work should be done separately. If you require
extra pages for your finished solutions, paper will be provided by your supervising teacher. Any extra
papers should be placed inside your answer booklet.
Marks are awarded for completeness, clarity, and style of presentation. A correct solution poorly
presented will not earn full marks.

NOTE: At the completion of the contest, insert the information sheet inside the answer
booklet.
Canadian Open Mathematics Challenge

NOTE: 1. Please read the instructions on the front cover of this booklet.
2. Write solutions in the answer booklet provided.
3. It is expected that all calculations
√ and answers will be expressed as
exact numbers such as 4π, 2 + 7, etc.
4. Calculators are not allowed.

PART A
1. If x + 2y = 84 = 2x + y, what is the value of x + y?

2. Let S be the set of all three-digit positive integers whose digits are 3, 5 and 7, with no digit
repeated in the same integer. Calculate the remainder when the sum of all of the integers in
S is divided by 9.

3. In the diagram, point E has coordinates


√ (0, 2), and B lies on y
the positive x-axis so that BE = 7. Also, point C lies on
the positive x-axis so that BC = OB. If point D lies in the E
first quadrant such that ∠CBD = 30◦ and ∠BCD = 90◦ ,
what is the length of ED?
D

x
O B C
4. A function f (x) has the following properties:

i) f (1) = 1
ii) f (2x) = 4f (x) + 6
iii) f (x + 2) = f (x) + 12x + 12

Calculate f (6).

5. The Rice Tent Company sells tents in two different sizes, large and small. Last year, the
Company sold 200 tents, of which one quarter were large. The sale of the large tents pro-
duced one third of the company’s income. What was the ratio of the price of a large tent to
the price of a small tent?

6. In the diagram, a square of side length 2 has semicircles


drawn on each side. An “elastic band” is stretched tightly
around the figure. What is the length of the elastic band in
this position?
7. Let a and b be real numbers, with a > 1 and b > 0.
a
If ab = ab and = a3b , determine the value of a.
b
8. A rectangular sheet of paper, ABCD, has AD = 1 and AB = r, where 1 < r < 2. The
paper is folded along a line through A so that the edge AD falls onto the edge AB. Without
unfolding, the paper is folded again along a line through B so that the edge CB also lies
on AB. The result is a triangular piece of paper. A region of this triangle is four sheets
thick. In terms of r, what is the area of this region?

PART B
1. The points A(−8, 6) and B(−6, −8) lie on the circle x2 + y 2 = 100.

(a) Determine the equation of the line through A and B.


(b) Determine the equation of the perpendicular bisector of AB.
(c) The perpendicular bisector of AB cuts the circle at two points, P in the first quadrant
and Q in the third quadrant. Determine the coordinates of P and Q.
(d) What is the length of P Q? Justify your answer.

2. (a) Determine the two values of x such that x2 − 4x − 12 = 0.



(b) Determine the one value of x such that x − 4x + 12 = 0. Justify your answer.
(c) Determine all real values of c such that
p
x2 − 4x − c − 8x2 − 32x − 8c = 0

has precisely two distinct real solutions for x.

3. A map shows all Beryl’s Llamaburgers restaurant locations in North America. On this map,
a line segment is drawn from each restaurant to the restaurant that is closest to it. Every
restaurant has a unique closest neighbour. (Note that if A and B are two of the restaurants,
then A may be the closest to B without B being closest to A.)

(a) Prove that no three line segments on the map can form a triangle.
(b) Prove that no restaurant can be connected to more than five other restaurants.

4. In a sumac sequence, t1 , t2 , t3 , . . ., tm , each term is an integer greater than or equal to 0.


Also, each term, starting with the third, is the difference of the preceding two terms (that
is, tn+2 = tn − tn+1 for n ≥ 1). The sequence terminates at tm if tm−1 − tm < 0. For
example, 120, 71, 49, 22, 27 is a sumac sequence of length 5.

(a) Find the positive integer B so that the sumac sequence 150, B, . . . has the maximum
possible number of terms.
(b) Let m be a positive integer with m ≥ 5. Determine the number of sumac sequences
of length m with tm ≤ 2000 and with no term divisible by 5.
2004
Canadian Open
Mathematics
Challenge
(English)
The Canadian Mathematical Society
in collaboration with

The CENTRE for EDUCATION


in MATHEMATICS and COMPUTING

presents the

Canadian Open
Mathematics Challenge
Wednesday, November 23, 2005

Time: 2 12 hours 2005


c Canadian Mathematical Society

Calculators are NOT permitted.

Do not open this booklet until instructed to do so.


There are two parts to this paper.

PART A
This part of the paper consists of 8 questions, each worth 5 marks. You can earn full value
for each question by entering the correct answer in the space provided. If you do not have the
correct answer, any work you do in obtaining an answer will be considered for part marks,
provided that it is done in the space allocated to that question in your answer booklet.
PART B
This part of the paper consists of 4 questions, each worth 10 marks. Finished solutions must
be written in the appropriate location in the answer booklet. Rough work should be done
separately. If you require extra pages for your finished solutions, paper will be provided by
your supervising teacher. Any extra papers should be placed inside your answer booklet. Be
sure to write your name and school name on any inserted pages.
Marks are awarded for completeness, clarity, and style of presentation. A correct solution
poorly presented will not earn full marks.
NOTE: At the completion of the contest, insert the information sheet inside the
answer booklet.
Canadian Open Mathematics Challenge
NOTE: 1. Please read the instructions on the front cover of this booklet.
2. Write solutions in the answer booklet provided.
3. It is expected that all calculations
√ and answers will be expressed as
exact numbers such as 4π, 2 + 7, etc.
4. Calculators are not allowed.

PART A
1. Determine the value of 102 − 92 + 82 − 72 + 62 − 52 + 42 − 32 + 22 − 12 .

2. A bug in the xy-plane starts at the point (1, 9). It moves first to the point (2, 10) and
then to the point (3, 11), and so on. It continues to move in this way until it reaches
a point whose y-coordinate is twice its x-coordinate. What are the coordinates of
this point?

3. If ax3 + bx2 + cx + d = (x2 + x − 2)(x − 4) − (x + 2)(x2 − 5x + 4) for all values of x,


what is the value of a + b + c + d?
p
4. A fraction is in lowest terms if p and q have no common factor larger than 1.
q
1 2 70 71
How many of the 71 fractions , ,..., , are in lowest terms?
72 72 72 72
5. An office building has 50 storeys, 25 of which are painted black and the other 25 of
which are painted gold. If the number of gold storeys in the top half of the building
is added to the number of black storeys in the bottom half of the building, the sum
is 28. How many gold storeys are there in the top half of the building?

6. In the grid shown, each row has a value assigned


to it and each column has a value assigned to it. 3 0 5 6 −2
The number in each cell is the sum of its row and −2 −5 0 1 y
column values. For example, the “8” is the sum 5 2 x 8 0
of the value assigned to the 3rd row and the value 0 −3 2 3 −5
assigned to the 4th column. Determine the values −4 −7 −2 −1 −9
of x and y.
7. In the diagram, the semi-circle has centre O and diameter AB. A ray of light leaves
point P in a direction perpendicular to AB. It bounces off the semi-circle at point D
in such a way that ∠P DO = ∠EDO. (In other words, the angle of incidence equals
the angle of reflection at D.) The ray DE then bounces off the circle in a similar
way at E before finally hitting the semicircle again at B. Determine ∠DOP .

A B
P O
8. The number 18 is not the sum of any 2 consecutive positive integers, but is the sum
of consecutive positive integers in at least 2 different ways, since 5 + 6 + 7 = 18 and
3 + 4 + 5 + 6 = 18. Determine a positive integer less than 400 that is not the sum of
any 11 consecutive positive integers, but is the sum of consecutive positive integers
in at least 11 different ways.

PART B
1. A line with slope −3 intersects the positive x-axis at A and the positive y-axis at B.
A second line intersects the x-axis at C(7, 0) and the y-axis at D. The lines intersect
at E(3, 4).
(a) Find the slope of the line through C and E.
y

(b) Find the equation of the line through C B


and E, and the coordinates of the point D.

(c) Find the equation of the line through A D


and B, and the coordinates of the point B.
E (3, 4)
(d) Determine the area of the shaded region.

x
O A C (7, 0)
2. (a) Determine all possible ordered pairs (a, b) such that
a−b = 1
2 2
2a + ab − 3b = 22

(b) Determine all possible ordered triples (x, y, z) such that


x2 − yz + xy + zx = 82
2
y − zx + xy + yz = −18
z 2 − xy + zx + yz = 18

3. Four tiles identical to the one shown, with a > b > 0,


b
are arranged without overlap to form a square with a
square hole in the middle. b
a
a

(a) If the outer square has area (a + b)2 , show that the area of the inner square
is (a − b)2 .
(b) Determine the smallest integer value of N for which there are prime numbers
a and b such that the ratio of the area of the inner square to the area of the
outer square is 1 : N .
(c) Determine, with justification, all positive integers N for which there are odd
integers a > b > 0 such that the ratio of the area of the inner square to the
area of the outer square is 1 : N .
Mathematics
Challenge

Canadian
(English)

Open

2005
4. Triangle ABC has its base on line segment P N and vertex A on line P M . Circles
with centres O and Q, having radii r1 and r2 , respectively, are tangent to the triangle
ABC externally and to each of P M and P N .

E
A
D Q
O L
K
P F B C G N

(a) Prove that the line through K and L cuts the perimeter of triangle ABC into
two equal pieces.
(b) Let T be the point of contact of BC with the circle inscribed in triangle ABC.
Prove that (T C)(r1 ) + (T B)(r2 ) is equal to the area of triangle ABC.
The Canadian Mathematical Society
in collaboration with

The CENTRE for EDUCATION


in MATHEMATICS and COMPUTING
presents the

Canadian Open
Mathematics Challenge
Wednesday, November 22, 2006

Supported by:

Time: 2 12 hours 2006


c Canadian Mathematical Society

Calculators are NOT permitted.

Do not open this booklet until instructed to do so.


There are two parts to this paper.

PART A
This part of the paper consists of 8 questions, each worth 5 marks. You can earn full value
for each question by entering the correct answer in the space provided. If you do not have
the correct answer, any work you do in obtaining an answer will be considered for part marks,
provided that it is done in the space allocated to that question in your answer booklet.
PART B
This part of the paper consists of 4 questions, each worth 10 marks. Finished solutions must
be written in the appropriate location in the answer booklet. Rough work should be done
separately. If you require extra pages for your finished solutions, paper will be provided by
your supervising teacher. Any extra papers should be placed inside your answer booklet. Be
sure to write your name and school name on any inserted pages.
Marks are awarded for completeness, clarity, and style of presentation. A correct solution poorly
presented will not earn full marks.
NOTES:
At the completion of the contest, insert the information sheet inside the answer
booklet.
The names of top scoring competitors will be published on the Web sites of the
CMS and CEMC.
Canadian Open Mathematics Challenge
NOTE: 1. Please read the instructions on the front cover of this booklet.
2. Write solutions in the answer booklet provided.
3. It is expected that all calculations
√ and answers will be expressed as
exact numbers such as 4π, 2 + 7, etc.
4. Calculators are not allowed.

PART A
1 1 1 1
   
1. What is the value of 1 + 2 1+ 3 1+ 4 1+ 5 ?

2. If f (2x + 1) = (x − 12)(x + 13), what is the value of f (31)?

3. In 4ABC, M is the midpoint of BC, as shown. If ∠ABM = 15◦


and ∠AM C = 30◦ , what is the size of ∠BCA? A

B M C

4. Determine all solutions (x, y) to the system of equations


4 5
+ = 12
x y2
3 7
+ = 22
x y2

x+8
5. In 4ABC, BC = 4, AB = x, AC = x + 2, and cos(∠BAC) = .
2x + 4
Determine all possible values of x.

6. Determine the number of integers n that satisfy all three of the conditions below:
• each digit of n is either 1 or 0,
• n is divisible by 6, and
• 0 < n < 107 .

7. Suppose n and D are integers with n positive and 0 ≤ D ≤ 9.


n
Determine n if = 0.9D5 = 0.9D59D59D5 . . . .
810
8. What is the probability that 2 or more successive heads will occur at least once in
10 tosses of a fair coin?
PART B
1. Piotr places numbers on a 3 by 3 grid using the following rule, called “Piotr’s
Principle”:

For any three adjacent numbers in a horizontal, vertical or diagonal


line, the middle number is always the average (mean) of its two
neighbours.

(a) Using Piotr’s principle, determine the missing numbers in the 3 19


grid to the right. (You should fill in the missing numbers in
8
the grid in your answer booklet.)

(b) Determine, with justification, the total of the nine numbers x


when the grid to the right is completed using Piotr’s
5 23
Principle.

(c) Determine, with justification, the values of x and y when the x 7


grid to the right is completed using Piotr’s Principle.
9 y
20

2. In the diagram, the circle x2 + y 2 = 25 intersects


the x-axis at points A and B. The line x = 11 y x = 11
intersects the x-axis at point C. Point P moves
along the line x = 11 above the x-axis and AP P
intersects the circle at Q. Q
(a) Determine the coordinates of P when
4AQB has maximum area. Justify your x
A B C
answer.

(b) Determine the coordinates of P when Q is


the midpoint of AP . Justify your answer.

(c) Determine the coordinates of P when the


area of 4AQB is 41 of the area of 4AP C.
Justify your answer.
3. (a) In the diagram, trapezoid ABCD has parallel sides
AB and DC of lengths 10 and 20, respectively. Also, A B
the length of AD is 6 and the length of BC is 8.
Determine the area of trapezoid ABCD.
D C
(b) In the diagram, P QRS is a rectangle and
T is the midpoint of RS. The inscribed R T S
circles of 4P T S and 4RT Q each have
radius 3. The inscribed circle of 4QP T
has radius 4. Determine the dimensions of
rectangle P QRS.

Q P
Mathematics
Challenge

Canadian
(English)

Open

2006
p
4. (a) Determine, with justification, the fraction , where p and q are positive integers
q
and q < 100, that is closest to, but not equal to, 73 .
a c a+c
(b) The baseball sum of two rational numbers and is defined to be .
b d b+d
(A rational number is a fraction whose numerator and denominator are both
integers and whose denominator is not equal to 0.) Starting with the rational
numbers 01 and 11 as Stage 0, the baseball sum of each consecutive pair of
rational numbers in a stage is inserted between the pair to arrive at the next
stage. The first few stages of this process are shown below:
0 1
STAGE 0: 1 1

0 1 1
STAGE 1: 1 2 1

0 1 1 2 1
STAGE 2: 1 3 2 3 1

0 1 1 2 1 3 2 3 1
STAGE 3: 1 4 3 5 2 5 3 4 1
Prove that
(i) no rational number will be inserted more than once,
(ii) no inserted fraction is reducible, and
(iii) every rational number between 0 and 1 will be inserted in the pattern at
some stage.
The Canadian Mathematical Society
in collaboration with

The CENTRE for EDUCATION


in MATHEMATICS and COMPUTING
presents the

Sun Life Financial Canadian


Open Mathematics Challenge

Wednesday, November 21, 2007

Time: 2 12 hours 2007


c Canadian Mathematical Society
Calculators are NOT permitted.
Do not open this booklet until instructed to do so.
There are two parts to this paper.
PART A
This part of the paper consists of 8 questions, each worth 5 marks. You can earn full value
for each question by entering the correct answer(s) in the space provided. If your answer is
incorrect, any work that you do will be considered for part marks, provided that it is done
in the space allocated to that question in your answer booklet.
PART B
This part of the paper consists of 4 questions, each worth 10 marks. Finished solutions must
be written in the appropriate location in the answer booklet. Rough work should be done
separately. If you require extra pages for your finished solutions, paper will be provided by
your supervising teacher. Any extra papers should be placed inside your answer booklet. Be
sure to write your name and school name on any inserted pages.
Marks are awarded for completeness, clarity, and style of presentation. A correct solution poorly
presented will not earn full marks.
NOTES:
At the completion of the contest, insert the information sheet inside the answer
booklet.
The names of top scoring competitors will be published on the Web sites of the
CMS and CEMC.
Sun Life Financial Canadian Open Mathematics Challenge
NOTE: 1. Please read the instructions on the front cover of this booklet.
2. Write solutions in the answer booklet provided.
3. It is expected that all calculations
√ and answers will be expressed as
exact numbers such as 4π, 2 + 7, etc., rather than as 12.566 . . .
or 4.646 . . ..
4. Calculators are not allowed.

PART A
1. If a = 15 and b = −9, what is the value of a2 + 2ab + b2 ?

2. A circular wind power generator turns at a rate of 30 complete revolutions per


minute. Through how many degrees does it turn in one second? y
3. In the diagram, ABCD is a rectangle with A on the
line y = x + 10, B on the line y = −2x + 10, and
C and D on the x-axis. If AD = 4, what is the area
of rectangle ABCD?

A B

x
D C

4. In June, the ratio of boys to girls in a school was 3 : 2. In September, there were
80 fewer boys and 20 fewer girls in the school and the ratio of boys to girls was 7 : 5.
What was the total number of students at the school in June?

5. The numbers 1, 2, 3, . . . , 9 are placed in a square array. The sum of the three rows,
the sum of the three columns, and the sum of the two diagonals are added together
to form a “grand sum”, S.
For example, if the numbers are placed as shown, the grand sum is

1 2 3 S = row sums + column sums + diagonal sums


4 5 6 = 45 + 45 + 30
7 8 9 = 120 .

What is the maximum possible value of the grand sum S?

6. In the diagram, O is the centre of the circle, AN is tangent to


the circle at A, P lies on the circle, and P N is perpendicular
to AN . If AN = 15 and P N = 9, determine the radius of O
the circle.
P

A N
7. Determine all ordered triples of real numbers, (x, y, z), that satisfy the system of
equations

xy = z 2
x+y+z = 7
x + y 2 + z 2 = 133 .
2

8. In the diagram, there are 28 line segments of length 1 arranged as shown to form
9 squares. There are various routes from A to B travelling along the segments so
that no segment is travelled more than once. Of these possible routes, determine

• the length of route that occurs the most often, and


• the number of different routes of this length.

A
B

PART B
1. An arithmetic sequence a, a + d, a + 2d, . . . is a sequence in which successive terms
have a common difference d. For example, 2, 5, 8, . . . is an arithmetic sequence with
common difference d = 3 because 5 − 2 = 8 − 5 = 3.

(a) If x − 1, 2x + 2 and 7x + 1 are the first three terms of an arithmetic sequence,


determine the value of x.
(b) For the value of x from (a), what is the middle term of the arithmetic sequence
x − 1, 2x + 2, 7x + 1, . . . , 72?

A geometric sequence a, ar, ar2 , . . . is a sequence in which successive terms have a


common ratio r. For example, the sequence 2, 10, 50, . . . is a geometric sequence with
10 50
common ratio r = 5 because = = 5.
2 10
(c) If y − 1, 2y + 2 and 7y + 1 are the first three terms of a geometric sequence,
determine all possible values of y.
(d) For each of the values of y from (c), determine the 6th term of the geometric
sequence y − 1, 2y + 2, 7y + 1, . . . .

2. In the diagram, ∠ABC = ∠BCD = 90◦ . Also, AB = 9, BC = 24 and CD = 18.


The diagonals AC and BD of quadrilateral ABCD meet at E.

(a) Determine the area of the quadrilateral ABCD.

(b) Show that the ratio DE : EB = 2 : 1. 9 A


B
(c) Determine the area of triangle DEC.
E
(d) Determine the area of triangle DAE.
24

C 18 D
Canadian Open
Mathematics
Challenge

Financial
(English)

Sun Life
2007
3. Alphonse and Beryl are back! They are playing a two person game with the following
rules:

• Initially there is a pile of N stones, with N ≥ 2.


• The players alternate turns, with Alphonse going first. On his first
turn, Alphonse must remove at least 1 and at most N − 1 stones from
the pile.
• If a player removes k stones on their turn, then the other player must
remove at least 1 and at most 2k − 1 stones on their next turn.
• The player who removes the last stone wins the game.

(a) Determine who should win the game when N = 7, and explain the winning
strategy.
(b) Determine who should win the game when N = 8, and explain the winning
strategy.
(c) Determine all values of N for which Beryl has a winning strategy. Explain this
strategy.

4. A cat is located at C, 60 metres directly west of a mouse located at M . The mouse


is trying to escape by running at 7 m/s in a direction 30◦ east of north. The cat, an
expert in geometry, runs at 13 m/s in a suitable straight line path that will intercept
the mouse as quickly as possible.
(a) If t is the length of time, in seconds, that it
takes the cat to catch the mouse, determine N
the value of t.

(b) Suppose that the mouse instead chooses a


different direction to try to escape. Show that
no matter which direction it runs, all points of
13 m/s 30
interception lie on a circle.
7 m/s

(c) Suppose that the mouse is intercepted after W 60 m M E


C
running a distance of d1 metres in a particular
direction. If the mouse would have been S
intercepted after it had run a distance of d2
metres in the
√ opposite direction, show that
d1 + d2 ≥ 14 30.
Canadian Open Mathematics Challenge

Comments on the Paper

Part A
a
1. An operation “ ∆ ” is defined by a ∆b = 1– , b ≠ 0.
b
What is the value of (1∆ 2)∆(3 ∆ 4) ?

Solution
By the definition of “ ∆ ”
1∆ 2 = 1 – 1 = 1
2 2
3 1
3∆ 4 = 1– =
4 4
1
and so (1∆ 2)∆(3∆ 4) =   ∆  = 1– = 1– 2 = –1
1 1 2
2 4 1
4
ANSWER: –1

2. The sequence 9, 18, 27, 36, 45, 54, … consists of successive multiples of 9. This sequence is then altered by multiplying
every other term by –1, starting with the first term, to produce the new sequence –9, 18, – 27, 36, – 45, 54,... . If the sum
of the first n terms of this new sequence is 180, determine n.

Solution
The terms in the sequence are paired, by combining each odd-numbered term with the next term
(that is, we combine terms 1 and 2, 3 and 4, 5 and 6, etc).

The sum of each of these pairs is 9.

So we need 20 of these pairs to reach a sum of 180.

Thus we need 2 × 20 or 40 terms.


ANSWER: 40
3. The symbol n! is used to represent the product n(n – 1)(n – 2) … (3)(2)(1) .
For example,4!= 4(3)(2 )(1) . Determine n such that n!= 2( 15 )(36 )(5 3)(72 )(11)(13) .
Solution
Since n! has a prime factor of 13, n must be at least 13.
Since n! has no prime factor of 17, n must be less than 17.
These two facts are true because if m ≤ n , then m divides n!.
Since n! has 53 as a factor, then n ≥ 15 , since we need n! to have 3 factors which are multiples of 5.
We must thus determine if n = 15 or n = 16 .
So we look at the number of factors of 2 in 16!.
16! gets 1 factor of 2 each from 2, 6, 10, 14
2 factors of 2 each from 4, 12
3 factors of 2 from 8
4 factors of 2 from 16
We have a total of 15 twos which then corresponds to n = 16 .
ANSWER: 16

4. The symbol x  means the greatest integer less than or equal to x. For example,

5.7 = 5 , π = 3 and 4 = 4 .


Calculate the value of the sum

 1 +  2 +  3 +  4 +      
… + 48 + 49 + 50 .

Solution
We note that for k a positive integer and k ≤ n < (k + 1) , then k ≤ n < k +1 and so  n = k .
2 2

Thus for 1 ≤ n ≤ 3,  n = 1
4 ≤ n ≤ 8,  n = 2

9 ≤ n ≤ 15,  n = 3

etc.
So the sum equals
(1+1+ 1) + (2 + 2 + 2 + 2 + 2) + (3 + … + 3) + … + (6 + … + 6) + (7 + 7)
= 3(1) + 5(2) + 7(3) + 9(4 ) +11(5) +13(6) + 2 (7)
= 3 +10 + 21+ 36 + 55 + 78+ 14
= 217
ANSWER: 217

5. How many five-digit positive integers have the property that the product of their digits is 2000?

Solution
Let a five-digit number have the form a b c d e where 0 ≤ a, b, c, d, e ≤ 9, a ≠ 0 .
Since the product of the digits is 2000, we must have the product abcde = 2000 = 24 53 .
Since the product of the digits is 2000, then 3 of the digits have to be 5. The remaining 2 digits must have a product of
4
16 or 2 .
Thus the two remaining digits must be 4 and 4, or 2 and 8.

Possibility 1
5!
Case 1 Using the numbers 5, 5, 5, 4, 4 there are = 10 possible numbers.
3!2!
5!
Case 2 Using the numbers 5, 5, 5, 2, 8 there are = 20 possible numbers.
3!
There are 30 possible such numbers.

OR

Possibility 2
 5
We choose 3 of the 5 positions for the “5s” in   ways; there are 3 possibilities for the remaining two digits (including
 3
order): 2, 8; 4, 4; 8, 2.
5 
So there are 3×   = 3 ×10 = 30 possible such 5 digit numbers.
3 
ANSWER: 30

6. Solve the equation 4 16sin ( 2


x
)= 2 6 sin x
, for 0 ≤ x ≤ 2π .

Solution
We write all factors as powers of 2. Thus
(
4 16 sin
2
x
)= 2 6 sin x

2 (2 )= 2
2 4 sin2 x 6 sin x

2
x +2
2 4sin = 26 sin x (*)
Equating exponents (which we can do by taking base 2 logarithms),
4 sin 2 x + 2 = 6 sin x
2 sin 2 x – 3 sin x + 1 = 0
(2 sin x – 1)(sin x – 1) = 0
1
Therefore, sin x = or sin x = 1 .
2
π 5π π
Since 0 ≤ x ≤ 2 π , x = , or .
6 6 2
π 5π π
ANSWER: , ,
6 6 2
The sequence of numbers …, a –3 , a– 2 , a–1 , a0 , a1 , a2 , a3 , … is defined by an – (n + 1)a2– n = (n + 3) , for all integers n.
2
7.
Calculate a0 .

Solution
Using the given general equation, we note that there are only two choices of n which yield equations containing a2 ,
n = 0 or 2.
i.e. a0 – a2 = 9 from n = 0
a2 – 3a0 = 25 from n = 2
Adding the first equation to the second, we obtain –2a0 = 34 , so a0 = –17 .
ANSWER: –17

8. In the diagram, ∆ ABC is equilateral and the radius of its E C D


inscribed circle is 1. A larger circle is drawn through the vertices
of the rectangle ABDE .
What is the diameter of the larger circle?
A B

Solution
First, we calculate the side length of the equilateral triangle ABC . C
Let O be the centre of the smaller circle and P the point of tangency of
the circle to the side AB .
O 30°
Join OP and OB . Then ∠OPB = 90° by tangency and ∠OBP = 30°
by symmetry since ∠CBA = 60° . A P B
Since OP = 1 and ∆ BOP is 30°-60°-90°, then OB = 2 and BP = 3 . Thus AB = 2 3 .
Also by symmetry, CO = OB = 2 , so CP = 3 .

Since ABDE is a rectangle and CP ⊥ AB , then AE = 3 .


We now look at the rectangle ABDE and its circumcircle.
Since ABDE is a rectangle, ∠ EAB = 90° . E D
So BE is a diameter.
By Pythagoras, 3
BE 2 = EA2 + AB 2

( )
2
=3 + 2 3
2 A B
2 3
= 21
The diameter is 21 .
ANSWER: 21
Part B

1. Triangle ABC has vertices A(0, 0) , B(9, 0 ) and C (0, 6) . The points P and Q lie on side AB such that AP = PQ = QB .
Similarly, the points R and S lie on side AC so that AR = RS = SC . The vertex C is joined to each of the points P and Q.
In the same way, B is joined to R and S.
(a) Determine the equation of the line through the points R and B.
(b) Determine the equation of the line through the points P and C.
(c) The line segments PC and RB intersect at X, and the line segments QC and SB intersect at Y. Prove that the
points A, X and Y lie on the same straight line.

Solution
Since A(0, 0) , B(9, 0 ) and AP = PQ = QB , then P has coordinates (3, 0 ) and Q has coordinates (6, 0) .
Similarly, R is the point (0, 2) and S is the point (0, 4).
2–0 2
(a) Since R is (0, 2) and B is (9, 0), then the slope of RB is m = = – and so the equation of the line is
0–9 9
2
y – 2 = – (x – 0)
9
2
y = – x+ 2
9
0–6
(b) Since P is (3, 0) and C is (0, 6), then the slope of PC is m = = – 2 and so the equation of the line is
3–0
y – 0 = –2(x – 3)
y = –2x + 6
C(0, 6)
(c) First, we determine the coordinates of X.
Equating the lines from (a) and (b), we have S(0, 4)
2 Y
– x + 2 = – 2x + 6
9 R(0, 2)
16 X
x=4
9
9 A(0, 1) P(3, 0)
x= Q(6, 0) B(9, 0)
4

and substituting into y = – 2 x + 6 = –2  + 6 = , so X is the point  ,  .


9 3 9 3
 4 2  4 2
We calculate the equations of the lines QC and SB as in (a) and (b).
0–6
For QC , slope = = –1 and so y – 0 = –1(x – 6) or y = – x + 6.
6–0
0– 4 4 4 4
For SB , slope = = – and so y – 0 = – (x – 9) or y = – x + 4 .
9–0 9 9 9
So the point Y, which lies at the intersection QC and SB , we obtain by equating these lines
4
–x + 6 = – x + 4
9
5
2= x
9
18
x=
5
and thus Y is the point  ,  .
18 12 18 12
and so y = – x + 6 = – +6 =
5 5  5 5
3
–0
Now the line through A(0, 0) and X  ,  has slope m = 29
9 3 2 2
= and so is y = x .
 4 2 –0 3 3
4
12 2  18 
The point Y lies on this line, as = . [This could be done with L.S./R.S. format using equation of line.]
5 3 5 
Therefore A, X, Y lie on the same line.

2. In ∆ ABC , the points D, E and F are on sides BC , CA and AB , A


respectively, such that ∠ AFE = ∠ BFD , ∠ BDF = ∠CDE , and E
F
∠CED = ∠ AEF .
(a) Prove that ∠ BDF = ∠ BAC .
(b) If AB = 5 , BC = 8 and CA = 7 , determine the length of
B D C
BD .

Solution
(a) Let ∠ AFE = ∠ BFD = x A
∠BDF = ∠CDE = y z E
∠CED = ∠AEF = z F x z
x
Thus ∠ FAE = 180° – x – z
∠FBD = 180° – x – y y y
∠ECD = 180° – y – z B D C
and these 3 angles add to 180° , so
540° – 2(x + y + z) = 180°
x + y + z = 180°
Since ∠ FAE + ∠ AFE + ∠ AEF = 180° (from ∆ AEF )
∠FAE + x + z = x + y + z
∠FAE = y
Therefore ∠ BDF = ∠ BAC .

(b) Similarly to part (a), ∠ ECD = ∠ BFD = x , ∠ FBD = ∠CED = z .


BD BA 5 CD CA 7 AE AB 5
By equal angles, ∆ ABC ~ ∆ DBF ~ ∆ DEC ~ ∆ AEF and so = = , = = , = = .
BF BC 8 CE CB 8 AF AC 7
Therefore, let BD = 5k , BF = 8k , CD = 7l , CE = 8l , AE = 5m , AF = 7m for some k, l, m.
Then 5k + 7l = 8 (1) A
7m y 5m E
7m + 8k = 5 (2)
x z
5m + 8l = 7 (3) F z
x 8l
Determining 7 × (3) – 5 × (1) to eliminate m, we get 8k
56l – 40k = 49 – 25 = 24 z y y x
7l – 5k = 3 (4) B 5k D 7l C
5
Calculating (1) – (4), we get 10 k = 5 or BD = 5k = .
2
3. (a) Alphonse and Beryl are playing a game, starting with the
geometric shape shown in Figure 1. Alphonse begins the
game by cutting the original shape into two pieces along one
of the lines. He then passes the piece containing the black
Figure 1
triangle to Beryl, and discards the other piece.
Beryl repeats these steps with the piece she receives; that is to say she cuts along the length of a line, passes the
piece containing the black triangle back to Alphonse, and discards the other piece. This process continues, with
the winner being the player who, at the beginning of his or her turn receives only the black triangle. Show, with
justification, that there is always a winning strategy for Beryl.

Solution
We first consider Alphonse’s possible moves to begin the game. We can assume, without loss of generality, that he cuts
on the left side of the black triangle.

Case 1
Alphonse removes two white triangles, leaving .
In this case, Beryl removes only one white triangle, and passes the shape back to Alphonse, forcing him to
remove the last white triangle and lose.

Case 2

Alphonse removes one white triangle only, leaving .


Beryl removes both of the white triangles on the right, leaving Alphonse in the same position as in Case 1 for his second
turn.
Therefore Beryl can always win, regardless of Alphonse’s strategy.
(b) Alphonse and Beryl now play a game with the same rules
as in (a), except this time they use the shape in Figure 2
and Beryl goes first. As in (a), cuts may only be made
along the whole length of a line in the figure. Is there a
strategy that Beryl can use to be guaranteed that she will
win? (Provide justification for your answer.)

Figure 2

Solution
We show that, again, Beryl always has a winning strategy.
The strategy is to reduce the shape in Figure 2 to the shape in Figure 1, and to have Alphonse make the first cut at this
stage. Beryl also knows that if she is forced into a position of being the first to cut when Figure 2 is reduced to Figure 1,
then Alphonse can force her to lose.
We number the lines on the diagram for convenience. (9)
(1)
(8)
(2)
(7)
(3)

(4) (5) (6)


We can assume without loss of generality (because of symmetry) that Beryl cuts along (1), (2) or (3) to begin.
If she cuts (2) or (3), then Alphonse cuts the other of these two and leaves Beryl with Figure 1, where she will lose.
Therefore Beryl cuts (1) to begin.

If Alphonse now cuts (2) or (3), Beryl cuts the other of these two and passes the shape in Figure 1 back to Alphonse, and
so he loses.
If Alphonse cuts (8) or (9), Beryl cuts the opposite and passes the shape in Figure 1 to Alphonse, and so he loses.
(Similarly, if he cuts (5) or (6)).
So assume that Alphonse cuts (4) or (7), say (4) by symmetry.

If Beryl now cuts any of (2), (3), (5), (6), (8), or (9), then Alphonse can force Beryl to lose, in the same way as she could
have forced him to lose, as above. So Beryl cuts (7).

Now Alphonse is forced to cut one of (2), (3), (5), (6), (8), or (9), and so Beryl makes the appropriate cut, passing the
shape in Figure 1 back to Alphonse, and so he must lose.
Therefore Beryl always can have a winning strategy.
4. A sequence t1, t2 , t3 , ..., tn of n terms is defined as follows:
t1 = 1 , t2 = 4 , and tk = tk –1 + tk –2 for k = 3, 4, ..., n .
Let T be the set of all terms in this sequence; that is, T = {t1 , t2 , t3 , ..., tn }.
(a) How many positive integers can be expressed as the sum of exactly two distinct elements of the set T ?

Summary
Part (a) 4✓
Part (b) 6✓

Solution
tk > 0 for all k, 1 ≤ k ≤ n .
Also tk < tk+1 for all k ≤ n – 1 since tk +1 = tk + tk –1 .
Hence the sequence is monotone increasing.
 n
The sum of any pair of terms is an integer and there are   pairs.
 2
Can any two pairs produce the same integer?
Consider ta + tb and tc + td . Clearly if tb = td then ta = tc and vice versa, implying the same pair.
Hence none of the four terms is equal, so we can assume one term, say td to be the largest.
Then td = td –1 + td –2 ≥ ta + tb , since the maximum values of ta and tb are td –1 and td –2 and they cannot be alike.
 n
But since tc > 0 , tc + td > ta + td and there are no two pairs that add to the same integer, so there are exactly   integers
 2
possible.
(b) How many positive integers can be expressed as the sum of exactly three distinct elements of the set T ?

Solution
Consider ta + tb + tc and td + te + t f . If any of the first three equals any of the second three we are left with pair
sums of the remaining ones being equal, which is impossible from part (a). Hence there are six unlike terms, and
again we can assume one, say t f , to be the greatest.
It is clearly possible for equality by setting ta and tb equal to t f –1 and t f –2 and then td and te equal to tc –1 and
tc –2 .
In how many ways can this be done for given f. Clearly,6 ≤ f ≤ n , and since 2 < c < f – 2 , for any given f there
n
 n – 4
are f – 5 choices for c and the number of ways possible is ∑
( f – 5) = 1 + 2 + 3 + + (n – 5) = 
 2 
.
f =6
 n  n – 4
There are a maximum of   –   possible integers.
 3  2 
Of these, are any two of like sum?
In ta + tb + tc , the maximal values are t f +1 , t f –3 , and t f –4 , since if one is t f –1 and one t f –2 we revert to the
discussed state. Hence
ta + tb + tc ≤ t f –1 + t f –3 + t f –4
= t f –1 + t f –2
= tf .
But td + te + t f > t f .
Hence there are no other triples for which equality of sums exist, and the number of possible integers is
 n  n – 4
  – .
 3  2 
Comments on the Paper Commentaires sur l'épreuve

PART A PARTIE A
1. Solution 1. Solution
By the definition, 2∇0 = 2 2 + 30 = 4 + 1 = 5 D’après la définition, on a : 2∇0 = 2 2 + 30 = 4 + 1 = 5
0∇1 = 0 2 + 31 = 0 + 3 = 3 0∇1 = 0 2 + 31 = 0 + 3 = 3
and so (2∇0)∇(0∇1) = 5∇3 Donc : (2∇0)∇(0∇1) = 5∇3

= 52 + 33 = 52 + 33
= 25 + 27 = 25 + 27
= 52 = 52
Commentaires
Comments
Les élèves ont bien réussi cette question. La plupart des
This question was quite well done. Most students correctly
élèves ont correctement interprété les opérations données
interpreted the given operation to do the required calcula-
nécessaires aux calculs requis.
tions. Moyenne: 3,6
Average: 3.6
2. Solution D
2. Solution D D’après le diagramme, 180° – 7x°
5x°
180° – 8x°

From the diagram, 180° – 7x°


5x°
180° – 8x° ∠ACB = 180o − 7 x o et A C E G
3x° 2x°

∠ACB = 180o − 7 x o and


A
3x°
C E
2x°
G
∠FEG = 180o − 8 x o . 6x°

∠FEG = 180o − 8 x o .
6x°
Donc ∠DCE = 180o − 7 x o F
F 4x°

Therefore, ∠DCE = 180o − 7 x o 4x° et ∠DEC = 180o − 8 x o . B


and ∠DEC = 180o − 8 x o , so B Dans le triangle CDE, on a donc :
from ∆CDE , 5 x o + 180o − 7 x o + 180o − 8 x o = 180o (*)
5 x o + 180o − 7 x o + 180o − 8 x o = 180o (*)
360o − 10 x o = 180o
o o o
360 − 10 x = 180
10 x o = 180o
10 x o = 180o x = 18
x = 18 Commentaires
Comments Chapeau! La quasi-totalité des concurrents ont bien maîtrisé
Extremely well done! Almost all of the competitors had cette question qui nécessitait l’application des connaissances
a good handle on dealing with angles in triangles. des angles aux triangles.
Average: 4.1 Moyenne: 4,1
3. Solution 1 3. Solution 1
A B A B
Let ABCDEF be a regular hexagon with Soit ABCDEF un hexagone régulier dont
o les côtés ont une longueur de 1. Chacun
a side length of 1. Each angle is 120 .
Thus, if we join FC, EB, DA, each of the
F C de ses angles mesure 120o . Les segments F C

interior angles is bisected, and so each FC, EB et DA sont des bissectrices des
E D angles. L’hexagone est donc décomposé E D
part is 60o . Thus the hexagon is decom-
en 6 triangles équilatéraux.
posed into 6 equilateral triangles, as shown. La distance maximale possible entre deux points sur
The maximum distance between any two points on the l’hexagone est celle entre deux sommets opposés. Puisque
hexagon is the distance between two opposite vertices. chaque triangle équilatéral a des côtés de longueur 1, la
Since each of the triangles is equilateral with a side length longueur maximale possible du segment est égale à 2.
of 1, the diagonal distance is 2, ie. the maximum possible
length of PQ is 2. Version abrégée de la solution 1
Comme l’indique le diagramme, l’hexagone régulier peut
Brief version of Solution 1 être divisé en 6 triangles équilatéraux ayant des côtés de
A regular hexagon with side length 1 can be decomposed longueur 1. La distance maximale possible entre deux
into 6 equilateral triangles with a side length of 1, as shown. points sur l’hexagone est celle entre deux sommets opposés.
Elle est égale à 2.
The maximum distance between any two points is between
opposite vertices, and this distance is 2. 6
Comments on the Paper Commentaires sur l'épreuve

Solution 2 Solution 2
A B Par symétrie, la distance maximale A B
The maximum distance is between
two opposite vertices, say F and C entre deux points sur l’hexagone est
celle entre deux sommets opposés,
by symmetry. 60° 60° 60° 60°
F R S C disons F et C. Aux points A et B, on F R S C
Drop perpendiculars from A and B abaisse des perpendiculaires AR et AS au segment FC.
to meet FC at R and S respectively. Puisque AB = 1 et que AB est parallèle à RS, alors RS = 1.
Since AB = 1 and AB is parallel to RS, then RS = 1.
Par symétrie, FR = CS . Or FR = AF cos60o = 1( 12 ) = 12 .
By symmetry, FR = CS . But
Donc CF = 2 et la longueur maximale possible est donc
FR = AF cos60o = 1( 12 ) = 12 . Therefore, CF = 2, and so égale à 2.
the maximum possible distance is 2.
Commentaires
Comments Une grande partie des difficultés de cette question résidait
The key problems here were to interpret the question and dans son interprétation. Les élèves devaient par la suite
to then figure out that the longest distance between any two déduire que la distance entre deux sommets opposés
points is the distance between opposite vertices. The représentait la distance maximale entre deux points. La façon
easiest way to calculate this length was to break the la plus rapide de la calculer consistait à diviser l’hexagone en
hexagon up into 6 equilateral triangles each of side length six triangles équilatéraux dont chacune des arêtes équivalait
one. If you didn’t notice this, have a look at this idea. à un. Nous invitons les élèves qui n’ont pas envisagé cette
Average: 3.6 approche de résolution de l’examiner de plus près.
Moyenne: 3,6
4. Solution
Solving the equation, 4. Solution

( )
2 2 2 x = 4 x + 64 or ( )
2 2 2 x = 4 x + 64
On a :
( )
2 2 2 x = 4 x + 64 ou ( )
2 2 2 x = 4 x + 64
2( 4 ) = 4
x x
+ 64 2( 2 ) = 2
2x 2x
+ 64
2( 4 ) = 4
x x
+ 64 2( 2 ) = 2
2x 2x
+ 64
4 = 64
x 2x
2 = 64
x=3 4 x = 64 2 2 x = 64
2x = 6
x=3 2x = 6
x=3
x=3
Comments
This question was reasonably well done. Students who are Commentaires
comfortable dealing with exponents had a great deal of Les élèves ont généralement bien réussi cette question, tout
success on this question. Many students continue to have particulièrement ceux qui maîtrisaient bien le concept
difficulty with exponents. d’exposant (qui d’ailleurs pose encore des difficultés pour
Average: 3.8 bon nombre d’élèves).
Moyenne: 3,8
5. Solution 1
5. Solution 1
Join M to Q. P P
On joint M et Q.
M M
Through M, draw a line parallel F Au point M, on trace un segment F
to QR meeting PQ at F. parallèle au côté QR . Ce segment
Q R Q R
Therefore, PF = FQ = 7 and MF = 24 . coupe le côté PQ en F.
By Pythagoras, MQ = 25, and so Donc PF = FQ = 7 et MF = 24 .
D’après le théorème de Pythagore dans le triangle MFQ,
cos( ∠MQP ) = cos( ∠MQF ) MQ = 25, d’où :
FQ cos( ∠MQP ) = cos( ∠MQF )
=
MQ FQ
=
7 MQ
=
25 7
=
7 25
Comments on the Paper Commentaires sur l'épreuve

Solution 2 Solution 2
P On joint M et Q.
Join M to Q. By Pythagoras, P
M D’après le théorème de M
14
PR = 14 2 + 482 = 50 . Pythagore dans le triangle
14

Since M is the midpoint of the Q R Q R


PQR, PR = 14 2 + 482 = 50 .
hypotenuse, then MQ = MP = MR . (This is because the
Puisque M est le milieu de l’hypoténuse,
circle circumscribed around ∆PQR has PR as diameter
o
MQ = MP = MR . (Puisque ∠PQR = 90o , PQ est le
(since ∠PQR = 90 ) and so M is the centre and thus MP, diamètre du cercle circonscrit au triangle et M en est le
MQ and MR are radii.) Therefore, ∠MQP = ∠MPQ and centre. Donc MP, MQ et MR sont des rayons.)
so cos( ∠MQP ) = cos( ∠MPQ ) = 14
50 =
7
25 . Donc ∠MQP = ∠MPQ , d’où
cos( ∠MQP ) = cos( ∠MPQ ) = 14 50 = 25
7 .
Comments
This is a nice question because it can be done with a Commentaires
Euclidean approach or a more analytic approach. Some Il s’agissait d’une question intéressante en raison de ses
students had difficulty when they encountered a triangle that deux méthodes de résolution : l’approche euclidienne ou
was not right-angled in which they had to calculate a cosine. une approche analytique plus générale. Quelques élèves
Average: 2.7 n’ont pu contourner la difficulté que posait la présence d’un
triangle sans angle droit, ce qui nécessitait le calcul du
6. Solution cosinus.
We calculate the first few terms in the series Moyenne: 2,7
t1 = 2
t1 − 1 2 − 1 1 6. Solution
t2 = = = Les premiers termes de la suite sont :
t1 + 1 2 + 1 3
t1 = 2
t −1 1
−1 1
t3 = 2 = 3
=− t1 − 1 2 − 1 1
t2 + 1 1
3 +1 2 t2 = = =
t1 + 1 2 + 1 3
t3 − 1 − 12 − 1
t4 = = = −3 t2 − 1 1
−1 1
t3 + 1 − 12 + 1 t3 = = 3
=−
t2 + 1 1
3 +1 2
t4 − 1 −3 − 1
t5 = = =2 t3 − 1 − 12 − 1
t4 + 1 −3 + 1 t4 = = = −3
t3 + 1 − 12 + 1
Since a term in the sequence depends only on the previous
one, then the sequence will cycle with a period of 4. Thus, t4 − 1 −3 − 1
t5 = = =2
t1 = t5 = L = t997 = t1001 = 2 . Therefore, t998 = 13 and t999 = − 12 . t4 + 1 −3 + 1
Puisque chaque terme de la suite dépend seulement du
Comments terme précédent, les valeurs se reproduisent à tous les 4
This question was very well done! Most students quickly termes. Les valeurs ont donc un cycle de longueur 4.
determined after a few calculations that the sequence was Donc t1 = t5 = L = t997 = t1001 = 2 . Donc t998 = 13 et
cyclic, and on this basis determined the value of the required t999 = − 12 .
term. The nicest solution by a student was to repeatedly
Commentaires
apply the definition:
t n −1 −1 Cette question a été très bien réussie! La plupart des élèves
t n − 1 tn −1 +1 − 1 (t n −1 − 1) − (t n −1 + 1) 1
t n +1 = = t −1 = =− ont conclu, moyennant quelques calculs, qu’ils se trouvaient
tn + 1 n −1
+ 1 (t n −1 − 1) + (t n −1 + 1)
t n −1 +1
t n −1
en présence d’une séquence cyclique : ils ont par la suite été
In a similar way, en mesure de trouver la valeur du terme requis. La solution
1 la plus astucieuse consistait à appliquer la définition de
t n −1 = −
tn− 3 façon répétitive :
and so t n −1 −1
1
t n +1 = − 1 = t n − 3 t −1
t n +1 = n =
t n −1 +1 − 1
=
(tn−1 − 1) − (tn−1 + 1) = − 1
−t
n −3
tn + 1 t n −1 −1
t n −1 +1 + 1
(tn−1 − 1) + (tn−1 + 1) tn−1
8
Comments on the Paper Commentaires sur l'épreuve

Thus, the sequence repeats every four terms. What a nice De la même manière,
solution! 1
t n −1 = −
Average: 2.7 tn− 3
On peut conclure que
1
7. Solution t n +1 = − 1 = t n − 3
−t
We treat a as a constant and solve for x, y, z in terms of n −3

a. Par conséquent, on peut affirmer que la séquence se répétera


From the second equation, y = x − a (*) à tous les quatre termes. Il suffisait d’y penser!
Therefore, from the first equation, Moyenne: 2,7
2x + a = x − a
7. Solution
x = −2 a On exprime d’abord x, y et z en fonction de a.
Substituting into (*), y = −3a . D’après la deuxième équation, y = x − a (*) .
Substituting into the 3rd equation, z = x + y = −5a . On reporte y = x – a dans la première équation :
So x + y + z = −10 a . 2x + a = x − a
Since a is a positive integer, the maximum value for x = −2 a
x + y + z is –10 (which occurs when a = 1). On reporte x = –2a dans l’équation (*) pour obtenir y = −3a .
On reporte y = −3a dans la troisième équation pour obtenir
Comments z = x + y = −5a .
This question was done very well. We anticipated that this Donc x + y + z = −10 a .
would be quite a difficult question, and so we were Puisque a peut prendre la valeur de n’importe quel entier
extremely happy with the results. Many students handled strictement positif, la valeur maximale possible de
this system of equations with relative ease by figuring out l’expression x + y + z est –10. On l’obtient lorsque a = 1.
that they had to solve for x, y and z in terms of a.
Commentaires
Average: 2.8
Nous avons été agréablement surpris du succès des élèves à
8. Solution 1 (Graphical) résoudre cette question qui se voulait extrêmement difficile;
y y de nombreux élèves ont facilement résolu ce système
d’équations où il fallait trouver la valeur de x, y et de z en
fonction de a.
4 4
Moyenne: 2,8
2 2

x 0 x
8. Solution 1 (Graphique)
–4 –2 0 2 4 –4 –2 2 4 y y

–2 –2

y = g( x ) y = g( x ) – 1 4 4

y 2 2
so the number of solutions
x 0 x
of g( x ) − 1 = 1
2 is 8, –4 –2 0 2 4 –4 –2 2 4

4 –2 –2
from the third graph.
2
1 y = g( x ) y = g( x ) – 1
y=2
x
–4 –2 0 2 4
y
–2
Selon le troisième graphique,
l’équation g( x ) − 1 = 1
2
y = g( x ) –1 4
admet 8 solutions.
2
1
y=2
x
–4 –2 0 2 4

–2

9 y = g( x ) –1
Comments on the Paper Commentaires sur l'épreuve

Solution 2 (Algebraic) Solution 2 (Algébrique)


From the original equation g( x ) − 1 = 1
2, using the D’après l’équation g( x ) − 1 = 12 , on a :
definition of absolute value we obtain, g( x ) − 1 = 1
or
ou g( x ) − 1 = − 12
2
g( x ) − 1 = 1
2 or g( x ) − 1 = − 12 g( x ) = 3
2 or
ou g( x ) = 1
2
g( x ) = 3
2 or g( x ) = 1
2 g( x ) = ± 23 or
ou g( x ) = ± 12
g( x ) = ± 23 or g( x ) = ± 12 D’après le graphique donné :
a) g( x ) = 23 admet 1 solution,
From the original graph,
(a) g( x ) = 23 has 1 solution, b) g( x ) = − 23 admet 1 solution,

(b) g( x ) = − 23 has 1 solution, c) g( x ) = 1


2 admet 3 solutions,

(c) g( x ) = 1
has 3 solutions, d) g( x ) = − 12 admet 3 solutions.
2
(d) g( x ) = − 12 has 3 solutions. Donc g( x ) − 1 = 1
2 admet 8 solutions.

Therefore, g( x ) − 1 = 1
2 has 8 solutions. Commentaires
Cette question a permis aux élèves de mettre à l’épreuve
Comments
leurs connaissances de la valeur absolue dans le contexte
This question was a good test of the concept of absolute
d’une représentation graphique. Certains élèves ont choisi
value from a graphical perspective. Some students used a
une approche graphique pour déterminer le graphique
graphical approach to convert the original graph to the
requis à partir de la transposition du premier graphique. En
desired one. Quite a few students used the graphical ap-
fait, plusieurs élèves ont eu recours à cette méthode de
proach to determine the potential values for either g( x ) or
résolution pour trouver les valeurs possibles de g( x ) ou
g( x ) and then read the appropriate number of solutions off
g( x ) ; ils ont ensuite déterminer le nombre de solutions
the graph. Students who tried to determine the actual
appropriées à partir des informations du graphique. Les
equation of the curve tended to get bogged down in their
élèves qui ont tenté de déterminer l’équation de la courbe
calculations.
ont en général éprouvé des difficultés au niveau des calculs.
Average: 2.0 Moyenne: 2,0
Part B
1. Solution Partie B
y
(a) The lines x = 2 and 1. Solution
y = 1 intersect at (2,1) .
x=2
a) Les droites définies y
The lines x = 2 and (2, 5) par x = 2 et y = 1 se x=2

x + 2 y = 12 intersect x + 2y = 12
coupent au point (2,1) .
at (2, 5) , since
(2, 5)
(2, 1)
(10, 1)
y=1 Les droites définies
x
x = 2 ⇒ 2 + 2 y = 12 ⇒ y = 5. par x = 2 et x + 2 y = 12 x + 2y = 12

(10, 1)
se coupent au point y=1
The lines y = 1 and x + 2 y = 12 intersect at (10,1) , (2, 1)
x
(2, 5) , car en reportant
since y = 1 ⇒ x + 2 = 12 ⇒ x = 10 .
x = 2 dans x + 2 y = 12 , on obtient 2 + 2y = 12, d’où y
(b) x + y = 8 intersects y
= 5. Les droites définies par y = 1 et x + 2 y = 12 se
x = 2 at (2, 6) , which x=2

is above the point of coupent au point (10,1) , car en reportant y = 1 dans


intersection of x = 2
(2, 5)
(4, 4)
x + 2 y = 12 , on obtient x + 2 = 12, d’où x = 10.
x + 2y = 12
b) Les droites d’équations y
and x + 2 y = 12 . Q
R (10, 1)
y=1 x + y = 8 et se coupent x = 2
x + y = 8 intersects (2, 1) (7, 1)
x
au point (2, 6) . Ce
y = 1 at (7,1). (2, 5)
point est au-dessus du (4, 4)
To find the intersection point of x + y = 8 and point d’intersection des Q
x + 2y = 12

R
x + 2 y = 12 , subtract the first equation from the sec- droites d’équations (2, 1) (7, 1)
(10, 1)
y=1
x
ond to obtain y = 4 , so x = 4. x = 2 et x + 2 y = 12 .
10
Comments on the Paper Commentaires sur l'épreuve

Therefore, the vertices of Q are (2,1) , (2, 5) , ( 4, 4) , Les droites d’équations x + y = 8 ett y = 1 se coupent au
(7,1). point (7,1).
(c) Area of Q = Area of T − Area of R Pour déterminer le point d’intersection des droites
définies par x + y = 8 et x + 2 y = 12 , on soustrait la
= 1
2
( 8)( 4 ) − 12 ( 3)( 3) première équation de la deuxième, membre par membre,
= 16 − 29 pour obtenir y = 4 , d’où x = 4.
Les coordonnées du quadrilatère Q sont (2,1) , (2, 5) ,
= 23
2
(4, 4) et (7,1).
Comments c) Aire de Q = Aire de T – Aire de R
This question was exceptionally well done. Students = 1
2
( 8)( 4 ) − 12 ( 3)( 3)
either approached this strictly graphically or with a com-
bination of graphical and analytic approaches. In either = 16 − 29
case, they tended to do very well. Part (c) was quite well = 23
2
done. Students managed to determine one of the many
Commentaires
ways to calculate the area of Q, either by subtracting the
Les élèves ont exceptionnellement bien réussi cette question
area of R from the area of T, by breaking Q up into two
peu importe la méthode de résolution utilisée. Des élèves ont
triangles, or by breaking Q up into one rectangle and two ainsi choisi la méthode de résolution graphique tandis que
triangles. d’autres ont opté pour un agencement des méthodes graphique
Average: 8.4 et analytique. Ils ont tout particulièrement bien réussi la partie
(c). Ils ont trouvé l’une des nombreuses façons de déterminer
2. (a) Solution 1 l’aire de Q, soit en soustrayant l’aire de R de celle de T, ou en
We define a “losing position” to be a number of divisant Q en deux triangles ou encore en un rectangle et deux
cards, such that if a player receives this number of triangles.
Moyenne: 8,4
cards at the beginning of his or her turn, he or she is
guaranteed to lose assuming best play by both play- 2. a) Solution 1
ers. A “winning position” is defined similarly. On définit une « position perdante » comme étant un
nombre de cartes qu’un joueur ou une joueuse reçoit au
Clearly, by the rules of the game, 1 is a losing début de son tour et qui lui assure une défaite si les deux
position. adversaires jouent à leur meilleur. On définit une « position
For a player to receive 1 card at the beginning of a gagnante » de façon semblable.
turn, the previous player must start with 2 cards. Selon les règles du jeu, 1 est une position perdante.
(This is true since a player can never remove more Pour qu’un joueur reçoive 1 carte au début de son tour, le
than half of the deck, so the number of cards at the joueur précédent doit commencer avec 2 cartes. (Un joueur
beginning of the previous turn can never be more ne peut retirer plus de la moitié des cartes, donc le nombre
than double that of the current turn.) Therefore, 2 is de cartes qu’il reçoit ne peut être supérieur au double du
nombre de cartes remises au joueur suivant.)
a winning position, since a player starting with 2
Donc 2 est une position gagnante, puisqu’un joueur qui
cards can only remove 1, and so passes 1 card to the reçoit 2 cartes peut seulement en enlever une et il remet 1
other player, who loses. carte à l’adversaire qui perd.
Is 3 a winning position or a losing position? Est-ce que 3 est une position gagnante ou perdante?
Given a pack of 3 cards, the rules of the game say that Si on reçoit 3 cartes, on peut seulement retirer une carte du
a player can only remove 1 card, and so pass a pack jeu et on remet 2 cartes à l’adversaire qui reçoit une position
gagnante. Donc 3 est une position perdante.
of 2 cards (a winning position) to the other player.
Therefore, 3 is a losing position. On peut constater que 4, 5 et 6 sont des positions gagnantes,
car on peut retirer respectivement 1, 2 ou 3 cartes et
We can then see that 4, 5 and 6 are all winning remettre 3 cartes à l’adversaire qui reçoit alors une position
positions, as given 4, 5 or 6 cards, a player can perdante. On constate que 7 est une position perdante, car
remove 1, 2 or 3 cards respectively to pass the other on doit retirer 1, 2 ou 3 cartes et on remet respectivement
player 3 cards, a losing position. 6, 5 ou 4 cartes à l’adversaire qui reçoit à chaque fois une
11
position gagnante.
Comments on the Paper Commentaires sur l'épreuve

Therefore, 7 is a losing position, since a player remov- Si Alain reçoit 7 cartes, Brigitte peut donc toujours
gagner.
ing 7 cards must remove 1, 2 or 3 cards, thus giving the
other player 6, 5 or 4 cards respectively, any of which Résumé de la stratégie de Brigitte
is a winning position. So if Alphonse starts with 7 • Elle recevra 4, 5 ou 6 cartes d’Alain.
cards, Beryl can always win. • Elle retirera 1, 2 ou 3 cartes de manière à remettre 3
cartes à Alain.
Summary of Beryl’s Strategy • Alain est forcé à retirer 1 carte et à remettre 2 cartes à
• She will receive 4, 5 or 6 cards from Alphonse. Brigitte.
• Remove 1, 2 or 3 cards in order to pass 3 cards back • Brigitte retire 1 carte et remet 1 carte à Alain qui perd.
to Alphonse.
• Alphonse is forced to remove 1 only, and pass back Solution 2 (suffisante pour recevoir le maximum de points)
2 to Beryl. Alain reçoit 7 cartes et il peut retirer 1, 2 ou 3 cartes pour
remettre 6, 5 ou 4 cartes à Brigitte.
• Beryl removes 1 and passes 1 back, so Alphonse
Brigitte doit retirer 3, 2 ou 1 carte de manière à remettre 3
loses. cartes à Alain.
Solution 2 (Sufficient for full marks) Alain est forcé à retirer 1 carte et à remettre 2 cartes à
Alphonse starts with 7 cards, and so can remove 1, 2 or Brigitte.
Brigitte retire 1 carte, (elle n’a pas d’autre choix) et elle
3 cards, passing 6, 5 or 4 cards to Beryl.
remet 1 carte à Alain qui perd.
Beryl should remove 3, 2 or 1 cards respectively, Brigitte a donc une stratégie gagnante.
leaving 3 cards only, and pass these 3 cards back to
Alphonse. b) Solution 1
Alphonse now is forced to remove 1 card only, and On doit déterminer si 52 est une position gagnante ou
pass 2 back to Beryl. perdante.
Beryl removes 1 card (her only option) and passes 1 Comme dans la partie précédente, on peut démontrer que 8,
9, 10, …, 14 sont des positions gagnantes, puisqu’on peut
back to Alphonse, who thus loses.
retirer suffisamment de cartes pour remettre 7 cartes à
Therefore, Beryl is guaranteed to win. l’adversaire qui reçoit alors une position perdante.
(b) Solution 1 Donc 15 est une position perdante, puisqu’en recevant 15
We must determine if 52 is a winning position or a cartes, on doit remettre de 8 à 14 cartes à l’adversaire qui
reçoit alors des positions gagnantes.
losing position. By a similar argument to above, since
7 is a losing position, 8 through 14 are all winning De la même manière, les nombres de 16 à 30 sont des
positions, since they can all be reduced to 7 in one turn. positions gagnantes, 31 est une position perdante et les
nombres de 32 à 62 sont des positions gagnantes.
Therefore, 15 is a losing position, since given 15 cards,
a player is forced to reduce to some number between Donc 52 est une position gagnante. Alain peut donc utiliser
8 and 14, since no more than 7 cards can be removed. une stratégie gagnante.

Similarly, 16 through 30 are winning positions, 31 is Résumé de la stratégie d’Alain


a losing position, and 32 through 62 are winning • Alain retire 21 cartes du jeu de 52 cartes et remet 31
positions. cartes à Brigitte.
Therefore, 52 is a winning position, so Alphonse has • Si Brigitte retire b1 cartes, 1 ≤ b1 ≤ 15 , Alain retire alors
a winning strategy. 16 − b1 cartes et remet 15 cartes à Brigitte. [On remarque
que c’est toujours permis, car
Summary of Alphonse’s strategy 2(16 − b1 ) = 32 − 2b1 ≤ 31 − b1 et 16 − b1 n’est jamais
• Alphonse removes 21 cards from original 52, and supérieur à la moitié du nombre de cartes reçues.]
pass 31 cards to Beryl. • Si Brigitte retire b2 cartes, 1 ≤ b2 ≤ 7 , retire alors 8 − b2
• If Beryl removes b1 cards with 1 ≤ b1 ≤ 15 , cartes et remet 7 cartes à Brigitte. [Ce nombre est
Alphonse removes 16 − b1 cards to reduce the toujours permis selon un argument semblable.]
pack to 15 cards. [Notice that this is always a • Alain adopte maintenant la même stratégie que Brigitte
dans la partie a).
legal move, since 2(16 − b1 ) = 32 − 2b1 ≤ 31 − b1,
so 16 − b1 is never more than half of the pack.]
12
Comments on the Paper Commentaires sur l'épreuve

• If Beryl removes b2 cards with 1 ≤ b2 ≤ 7 ,


Solution 2
Alphonse removes 8 − b2 to reduce the pack to 7 Alain retire 21 cartes du jeu de 52 cartes et remet 31 cartes à
cards. [This move is always legal by a similar Brigitte.
argument.]
Si Brigitte retire b1 cartes, 1 ≤ b1 ≤ 15 , Alain retire alors
• Beryl now has 7 cards, so Alphonse should adopt
Beryl’s strategy from (a). 16 − b1 cartes et remet 15 cartes à Brigitte. [On remarque que
c’est toujours permis, car 2(16 − b1 ) = 32 − 2b1 ≤ 31 − b1 et
Solution 2
16 − b1 n’est jamais supérieur à la moitié du nombre de cartes
Alphonse removes 21 cards from original 52, and
reçues.]
passes 31 cards to Beryl.
Si Brigitte retire b2 cartes, 1 ≤ b2 ≤ 7 , retire alors 8 − b2
If Beryl removes b1 cards with 1 ≤ b1 ≤ 15 , Alphonse
cartes et remet 7 cartes à Brigitte. [Ce nombre est toujours
removes 16 − b1 cards to reduce the pack to 15 cards. permis selon un argument semblable.]
[This is always a legal move, since
Alain adopte maintenant la même stratégie que Brigitte dans
2(16 − b1 ) = 32 − 2b1 ≤ 31 − b1, so 16 − b1 is never la partie a). Il a donc une stratégie gagnante.
more than half of the pack.]
Commentaires
If Beryl removes b2 cards with 1 ≤ b2 ≤ 7 , Alphonse Les questions pertinentes à « Alphonse et Beryl » demeurent
removes 8 − b2 to reduce the pack to 7 cards. [This sans contredit un des points marquants du concours du Défi
move is always legal by a similar argument.] ouvert canadien de mathématiques (DOCM). Les élèves ont
Beryl now has 7 cards, so Alphonse should adopt bien réussi la partie (a). La plupart ont rapidement compris
que la position des trois cartes constituait l’aspect essentiel
Beryl’s strategy from (a), so Alphonse has a winning
du problème. Les élèves ont cependant éprouvé des difficultés
strategy. à la partie (b) où bon nombre ont supposé que la méthode de
Comments résolution était liée au nombre égal de cartes paires et
impaires ou à la correspondance du nombre de cartes (si
The "Alphonse and Beryl" questions continue to be a
Beryl prend cinq cartes Alphonse devrait faire de même). Les
highlight of the COMC. Part (a) met with a good degree concurrents devraient lire la solution puis inviter un camarade
of success. Most students quickly realized that the posi- à se laisser prendre au jeu!
tion of 3 cards was the important one on which to focus. Moyenne: 4,0
Part (b) did not meet with as much success – many students
thought that the strategy had something to do with parity 3. a) Solution
(ie. even or odd numbers of cards) or with a "matching" f (0) + f ( −1) = c + (1 − 6 + c)
strategy (if Beryl takes 5 cards, Alphonse should take 5 = 2c − 5
cards). Competitors should have a look at the solution and Puisque c est un entier, 2c est toujours pair et 2c − 5 est
then try playing the game with an unsuspecting friend! donc toujours impair.
Average: 4.0
b) Solution 1
3. (a) Solution Supposons que l’équation g( x ) = 0 admet trois racines
Calculating, entières, a, b et c.
f (0) + f ( −1) = c + (1 − 6 + c) Donc g( x ) = ( x − a)( x − b)( x − c) = x 3 + px 2 + qx + r .
= 2c − 5 Donc g(0) = − abc et puisque g(0) est impair, alors
Now 2c is always even for c an integer, so 2c − 5 chacune des racines doit être impaire pour que leur
produit soit impair.
is always odd.
On a g( −1) = ( −1 − a)( −1 − b)( −1 − c).
(b) Solution 1 Puisque a, b et c sont impairs, alors −1 − a, –1 – b et
Assume that g( x ) = 0 has three integer roots a, b, c, –1 – c sont pairs et g( −1) est donc pair, ce qui est une
ie. contradiction.
g( x ) = ( x − a)( x − b)( x − c) = x 3 + px 2 + qx + r . Donc l’équation g( x ) = 0 ne peut admettre trois racines
Now g(0) = − abc from the above and is odd, so entières.
each of a, b and c must be odd for their product to be
odd. 13
Comments on the Paper Commentaires sur l'épreuve

Therefore, g( −1) = ( −1 − a)( −1 − b)( −1 − c). Solution 2


Since a is odd, −1 − a is even, and so g( −1) is even, a Supposons que l’équation g( x ) = 0 admet trois
racines entières, a, b et c.
contradiction. (In fact, g( −1) is divisible by 8.) Donc
Thus, g( x ) = 0 cannot have three integer roots.
g( x ) = ( x − a)( x − b)( x − c) = x 3 + px 2 + qx + r .
Solution 2 On développe pour obtenir :
Assume that g( x ) = 0 has three integer roots a, b, c, x 3 − ( a + b + c) x 2 + ( ab + ac + bc) x − abc
ie. g( x ) = ( x − a)( x − b)( x − c) = x 3 + px 2 + qx + r = x 3 + px 2 + qx + r
or, expanding,
Puisque g(0) et g(–1) sont impairs, et que g(0) = r
x 3 − ( a + b + c) x 2 + ( ab + ac + bc) x − abc et g( −1) = −1 + p − q + r alors r et –1 + p – q + r
= x 3 + px 2 + qx + r sont impairs.
Donc p − q est impair.
Now g(0) = r so r is odd, and
Donc p ou q doit être pair (ils ne peuvent être
g( −1) = −1 + p − q + r , which is odd. impairs tous les deux, car leur différence serait
Combining these, since r is odd, then p − q is odd too. paire).
Therefore, one of p and q is even (they cannot both be
Puisque r est impair et que r = − abc , alors a, b et c
odd, since odd – odd = even). sont tous impairs. Donc les nombres
Since r is odd and r = − abc , then each of a, b and c p = −( a + b + c )
is odd. This implies that q = ab + ac + bc
p = −( a + b + c ) sont impairs tous les deux, ce qui est une
q = ab + ac + bc contradiction car on vient de conclure que p ou q
doit être pair.
are both odd, a contradiction (since we have shown
Donc l’équation g( x ) = 0 ne peut admettre trois
above that one of p and q must be even.)
racines entières.
Therefore, g( x ) = 0 cannot have three integer roots.
Comments Commentaires
The purpose of part (a) was to have a relatively straightfor- Les élèves ont généralement très bien réussi la partie (a)
ward "proof" question, which required students to write a où il fallait résoudre une question au moyen d’un
raisonnement logique, mais ont éprouvé davantage de
logical argument. In general, students did exceptionally
difficultés avec la partie (b). De nombreux élèves ont
well on (a). Part (b) was a fair bit more difficult. Many compris que r était impair et que de p et q l’un devait
students recognized that r had to be odd and then that one être pair et l’autre impair sans toutefois aller plus loin
of p and q is even and the other odd, but were then stuck. A dans la résolution du problème. Quelques élèves
few students ingeniously pointed out that not only can this ingénieux ont indiqué que l’équation cubique ne pouvait
cubic equation not have 3 integer roots, it cannot even have avoir trois racines entières ni même une comme le
1 integer root, as they showed with the following proof: démontre le raisonnement suivant :
Suppose that q and r are both odd and p even.
Supposons que q et r sont tous deux pairs et que p est
Let a be an integer root of x 3 + px 2 + qx + r = 0 , ie. impair.
a 3 + pa 2 + qa + r = 0 . Soit a est une racine entière de l’équation

If a is even, then a 3 , pa 2 , and qa are all even, and r is odd, x 3 + px 2 + qx + r = 0 , c’est-à-dire

so a 3 + pa 2 + qa + r is odd and so cannot be 0. a 3 + pa 2 + qa + r = 0 .

If a is odd, then a 3 , qa, and r are all odd, and is even (since Si a est impair, alors a 3 , pa 2 et qa sont tous pairs, et

p is even), so a 3 + pa 2 + qa + r is odd and so cannot be 0. r est impair, a 3 + pa 2 + qa + r a pour solution un


Therefore, there cannot be any integer roots. nombre impair autre que 0.
(A similar argument works for p odd and q even.) Si a est impair, alors a 3 , qa et r sont tous impairs et pa 2
Congratulations to those students who had this brilliant est impair (puisque p est impair, alors a 3 + pa 2 + qa + r
insight! a pour solution un nombre pair autre que 0).
Average: 4.0 14
Comments on the Paper Commentaires sur l'épreuve

4. Solution 1 (Trigonometry) Par conséquent, il n’existe pas de racine entière.


A
Let ∠BCP = ∠ABP = α 90 – θ (Le raisonnement semblable voulant que p soit impair et q
and ∠ACP = θ . pair est également valable.)
x α D Félicitations aux élèves qui sont arrivés à cette brillante
Then ∠PBC = θ since 90 + θ + α 2θ
+
déduction!
∆ABC is isosceles. Also, Moyenne: 4,0
α P θ
∠PAC = 90o − θ (from θ α
B C
∆APC), ∠ADP = 2θ + α 4. Solution 1 (par trigonométrie et géométrie analytique)
A
(exterior angle), and ∠APB = 90 + θ + α o Soit ∠BCP = ∠ABP = α et ∠ACP = θ . 90 – θ

(exterior angle). Donc ∠PBC = θ , puisque le triangle


x D
Let AP = x . Then from ∆ APC , ABC est isocèle. 90 + θ + α 2θ

x x Dans le triangle APC,


sin θ = = ⇒ x = 5 sin θ (*) ∠PAC = 90o − θ . ∠ ADP = 2θ + α α P θ
AC 5 θ α
(angle extérieur du triangle BCD) B C
By the sine law in ∆ABP , o
x 5 ∠APB = 90 + θ + α (angle extérieur du triangle ADP)
= o
(
sin α sin 90 + θ + α ) Soit AP = x . Dans le triangle APC, on a sin θ =
x
AC 5
x
= ,
5
= d’où x = 5 sin θ (*).
cos(θ + α )
D’après la loi des sinus dans le triangle ABP :
5
= x 5
cos( ∠ABC ) =
5
o
(
sin α sin 90 + θ + α )
= 3 5
5 =
cos(θ + α )
x 25
= (**) 5
sin α 3 =
[Note that cos(∠ABC ) = 3
since drawing a perpen- cos( ∠ABC )
5
dicular from A bisects BC. Also, sin(∠ABC ) = 45 .] 5
= 3
Therefore combining (*) and (**) 5
x 25
= (**)
5 sin θ 25 sin α 3
=
sin α 3
3 sin θ = 5 sin α [On remarque que cos(∠ABC ) = 35 , car la hauteur au point A

3 sin θ = 5 sin(∠ABC − θ ) est aussi la médiatrice de BC. On a aussi sin(∠ABC ) = 45 .]


On utilise (*) et (**) pour obtenir :
3 sin θ = 5 sin(∠ABC ) cosθ − 5 cos(∠ABC ) sin θ
5 sin θ 25
3 sin θ = 4 cosθ − 3 sin θ =
sin α 3
6 sin θ = 4 cosθ 3 sin θ = 5 sin α
tan θ = 2
3 3 sin θ = 5 sin(∠ABC − θ )
3 sin θ = 5 sin(∠ABC ) cosθ − 5 cos(∠ABC ) sin θ
To determine the ratio of AD to DC, we use coordinates.
3 sin θ = 4 cosθ − 3 sin θ
Let B have coordinates (0, 0) and C have coordinates
6 sin θ = 4 cosθ
(6, 0). Thus, A has coordinates (3, 4) , since the altitude
from A to BC has length 4. tan θ = 2
3

Since tan θ = 23 , the line from B to D has equation Pour déterminer le rapport de AD à DC, on utilise un repère
y = 23 x . Also, the line from A to C has equation cartésien.
Soit (0, 0) les coordonnées du point B et (6, 0) les coordonnées
y = − 43 ( x − 6) . To find D, we find the intersection of
du point C. Les coordonnées du point A sont (3, 4), puisque
these two lines:
la hauteur de A à BC a une longueur de 4. Puisque tan θ = 23 ,
15
Comments on the Paper Commentaires sur l'épreuve

la droite qui passe par B et D a pour équation y = 23 x . De


2
3 x = − 43 x + 8
plus, la droite qui passe par A et C a pour équation
2x = 8 y = − 43 ( x − 6) . Le point D est le point d’intersection de ces
x=4 deux droites. Donc :
Therefore, D has coordinates ( 4, 83 ) . Since the x-coordi- 2
3 x = − 43 x + 8
nate of D is 13 of the way between those of A and C, then 2x = 8
AD : DC = 1 : 2 . x=4
Solution 2 (Similar triangles) Les coordonnées de D sont donc ( 4, 83 ) .
Draw a perpendicular from A to A On compare les déplacements horizontaux de A à D et de
DàC: 4–3 1
meet BC at M. Then since α D =
5 6–4 2
AB = AC , BM = MC = 3 and so
Donc AD : DC = 1 : 2 .
AM = 4 . α P
θ θ
θ α
Let ∠BCP = ∠ABP = α and B 3 M 3 C Solution 2 (par triangles semblables)
A
∠ACP = θ . Then ∠PBC = θ On abaisse une perpendiculaire AM de
since ∆ABC is isosceles. A à BC. Puisque AB = AC , alors α D
5
Draw circle with AC has diameter. This circle passes BM = MC = 3, d’où AM = 4 .
through both P and M, since ∠APC = ∠AMC = 90o . Soit ∠BCP = ∠ABP = α et α P
θ
θ
α
θ

Join P to M. Then ∠PAM = α since ∠PAM = ∠PCM ∠ACP = θ . Donc ∠PBC = θ , B 3 M 3 C


puisque le triangle ABC est isocèle.
(subtended by the same chord). Also, ∠AMP = θ for
On trace un cercle ayant AC pour diamètre. Ce cercle
similar reasons. Therefore, ∆MPA is similar to ∆BPC.
passe aux points P et M, puisque ∠APC = ∠AMC = 90o .
Thus,
On joint P et M. Donc ∠PAM = α , puisque
PA MA 4 PA 2
= = ⇒ tan θ = = ∠PAM = ∠PCM (ces angles interceptent le même arc).
PC BC 6 PC 3 De même, ∠AMP = θ . Les triangles MPA et BPC sont
So now we must compute the length of DC. Consider donc semblables.
∆BDC . By the sine law, PA MA 4 PA 2
Donc = = , d’où tan θ = = .
PC BC 6 PC 3
On cherche maintenant à déterminer DC. Selon la loi des
DC BC sinus dans le triangle BDC :
=
sin θ sin(∠BDC ) DC BC
=
6 sin θ sin θ sin(∠BDC )
DC = o
(
sin 180 − θ − ∠DCB ) DC =
6 sin θ

=
6 sin θ ( o
sin 180 − θ − ∠DCB )
sin(θ + ∠DCB) 6 sin θ
=
6 sin θ sin(θ + ∠DCB)
=
sin θ cos(∠DCB) + cosθ sin(∠DCB) 6 sin θ
=
=
6 sin θ cos(∠DCB) + cosθ sin(∠DCB)
cos(∠DCB) + cot θ sin(∠DCB) 6
=
=
6 cos(∠DCB) + cot θ sin(∠DCB)
3
5 + 3 4
2 5() 6
=
=
10 3
5 + 3 4
2 5()
3 10
=
3
which yields also that AD = 5 − 103 = 5
and so
3 Donc AD = 5 − 103 = 5
3 , d’où AD : DC = 1 : 2 .
AD : DC = 1 : 2 .
16
Comments on the Paper Commentaires sur l'épreuve

Solution 3 (Geometry) Solution 3 (par géométrie)


Draw a line from A perpen- A K On abaisse une perpendiculaire AM de A A K
dicular to BC at M. D à BC. Soit O le milieu de AC. On trace un
D
Let O be the midpoint of AC.
X O cercle de centre O et de rayon OC. Ce X O
cercle passe au point A, puisque
Draw circle with centre O P P
AO = OC , et aux points P et M,
and radius OC. B M C o
puisque ∠APC = ∠AMC = 90 . B M C
Then this circle passes through A (since AO = OC ), P
On joint M et O et on prolonge le segment obtenu jusqu’au
and M (since ∠APC = ∠AMC = 90o ). cercle au point K.
Join M to O and extend this line segment to meet the Puisque CO = 12 CA et CM = 12 CB , alors MK est parallèle à
circle at K. BA. On prolonge BD jusqu’à MK au point K ′ .
Since CO = 12 CA and CM = 12 CB , then MK is parallel Donc ∠MK ′B = ∠ABK ′ à cause des segments parallèles.
to BA. Or ∠ ABK ′ = ∠ ABP = ∠ BCP , d’où ∠MK ′P = ∠MCP .
Extend BD to meet MK at K ′ . Donc K ′ est situé sur le cercle, ce qui indique que K ′ et K
Then ∠MK ′B = ∠ABK ′ because of parallel lines. coïncident.
On joint A et K, ainsi que K et C.
But ∠ABK ′ = ∠ABP = ∠BCP ⇒ ∠MK ′P = ∠MCP.
Puisque KM et AC sont des diamètres du cercle, le quadrilatère
Therefore, K ′ lies on the circle; that is, K ′ coincides with AKCM est un rectangle.
K. Donc AK = MC = BM .
Next, join A to K and K to C. Donc AKMB est un parallélogramme. Donc AM et BK se
Then AKCM is a rectangle, since KM and AC are diameters coupent en leur milieu au point X.
On considère maintenant le triangle AKM. KX et AO sont des
of the circle, so the quadrilateral has four right angles.
médianes de ce triangle. Elles se coupent donc dans le rapport
Therefore, AK = MC = BM .
2:1. Donc AD : DO = 2 : 1. Puisque AO : OC = 1 : 1, alors
Then AKMB is a parallelogram, which implies that AM and
AD : DC = 1 : 2 .
BK bisect each other (meeting at X).
Commentaires
Consider now ∆AKM . Then KX and AO are medians, and
Nous nous réjouissons des tentatives de bon nombre d’élèves
so intersect in the ratio 2 : 1 , ie. AD : DO = 2 : 1. Since à résoudre cette question d’un haut niveau de difficulté. Dans
AO : OC = 1 : 1, then AD : DC = 1 : 2 . un premier temps, il fallait déterminer la valeur de quelques
angles du diagramme, une étape somme toute simple. Dans
Comments un deuxième temps, il fallait discerner la présence d’un
This question was extremely difficult, but it was gratifying cercle, puis trouver des triangles semblables par une approche
déductive complexe. Parmi les trois solutions données par les
to see many students at least making an effort to start the
élèves, la seconde comportait sans doute les éléments les plus
question. The first step was to determine some of the intéressants tandis que la première nécessitait un raisonnement
angles in the diagram, which was relatively straightfor- moins complexe mais davantage de calculs difficiles.
ward. The second step of recognizing the presence of a Moyenne: 0,3
circle and then finding similar triangles required a great
deal of insight. Of the three solutions presented, the
second is probably the nicest. The first solution requires
less insight, but is more difficult computationally.
Average: 0.3

17
The Canadian Mathematical Society
in collaboration with

The CENTRE for EDUCATION


in MATHEMATICS and COMPUTING

The
Canadian Open
Mathematics Challenge
Wednesday, November 27, 2002

Solutions

© 2002 Canadian Mathematical Society


2002 COMC Solutions 2

Part A

1. By Pythagoras in ∆PFR , PF 2 = 132 − 5 2 = 144 , or P


PF = 12 .
By Pythagoras in ∆PFQ , PQ2 = 9 2 + 12 2 = 225 , or
13
PQ = 15 . 12
Therefore, the side lengths of ∆PQR are 13, 14 and 15,
i.e. the perimeter is 42.
Q 9 F 5 R

2. Solution 1
x 2 + 5 xy + y 2 = x 2 + 2 xy + y 2 + 3 xy
= ( x + y ) + 3 xy
2

= 4 2 + 3(−12)
= −20

Solution 2
Examining the two given equations, we see that x = 6 and y = −2 is a solution.
Therefore, x 2 + 5 xy + y 2 = 6 2 + 5(6)(−2) + (−2) = 36 − 60 + 4 = −20 .
2

Solution 3
We solve the first equation for x and substitute into the second equation.
From the first equation, x = 4 − y .
Substituting into the second equation, ( 4 − y ) y = −12 or 0 = y 2 − 4 y − 12 .
Factoring, 0 = ( y − 6)( y + 2) , i.e. y = 6 or y = −2 . The corresponding values of x are
x = −2 and x = 6 , which give the same answer as in Solution 2, i.e. x 2 + 5 xy + y 2 = −20 .

3. To determine ∠EAR , we look at the angles around the A


point E. R
We know that ∠AER + ∠REN + ∠NEP + ∠PEA = 360o . P 60° 102°
90° 108°
Since ∠PEA is an angle in an equilateral triangle, E T
∠PEA = 60o .
Since ∠NEP is an angle in a square, ∠NEP = 90o . O N
D
Since ∠REN is an angle in a regular pentagon, ∠REN = 1
5 (540 ) = 108 .
o o
2002 COMC Solutions 3

Therefore,
∠AER = 360o − ∠REN − ∠NEP − ∠PEA
= 360o − 108o − 90o − 60o
= 102o
Now since PEA is an equilateral triangle, OPEN is a square, and TREND is a regular
pentagon, then their side lengths must all be the same, since OPEN and TREND share a
side, and since OPEN and PEA share a side. In particular, AE = ER .
Therefore, ∆ARE is an isosceles triangle, and so
( ) ( )
∠ARE = 12 180o − ∠AER = 12 180o − 102o = 39o .

4. Solution 1
The sum of the 3rd, 4th and 5th terms of the sequence is equal to the sum of the first five
terms of the sequence minus the sum of the first two terms of the sequence.
[ ][ ]
Thus, the sum is 5(5) + 6(5) − 5(2) + 6(2) = 155 − 32 = 123.
2 2

Solution 2
We determine the first 5 terms in the sequence and then add up the 3rd, 4th and 5th terms.
From the formula given, the sum of the first 1 terms is 11.
This tells us that the first term is 11.
From the formula given, the sum of the first 2 terms is 32. Since the first term is 11, then
the second term is 21.
Next, the sum of the first 3 terms is 63, and so the third term is 31, since the first two terms
are 11 and 21. (We could use the fact that the sum of the first two terms is 32, instead.)
Next, the sum of the first 4 terms is 104, and so the fourth term is 41.
Lastly, the sum of the first 5 terms is 155, and so the fifth term is 51.
Therefore, the sum of the 3rd, 4th and 5th terms is 31 + 41 + 51 = 123 .

Solution 3
Since the sum of the first n terms has a quadratic formula, then the terms in the sequence
have a common difference, i.e. The sequence is an arithmetic sequence.
Therefore, the sum of the 3rd, 4th and 5th terms is equal to three times the 4th term.
The 4th term is the sum of the first four terms minus the sum of the first three terms, i.e.
104 − 63 = 41.
Thus, the sum of the 3rd, 4th and 5th terms is 3( 41) = 123.
2002 COMC Solutions 4

5. Solution 1
Since the value of this expression is the same for every positive integer a, then we can find
the value by substituting in a = 1.
Thus,
[(2a − 1)∇(2a + 1)] = [1∇3] = 1 + 2 + 3 = 6 = 2
[(a − 1)∇(a + 1)] [0∇2] 0 + 1 + 2 3
Therefore, the value required is 2.

Solution 2
If a is a positive integer, the only integer between 2 a − 1 and 2 a + 1 is 2a . Similarly, the
only integer between a − 1 and a + 1 is a.
Thus,
[(2a − 1)∇(2a + 1)] = (2a − 1) + 2a + (2a + 1) = 6a = 2
[(a − 1)∇(a + 1)] (a − 1) + a + (a + 1) 3a
Therefore, the value required is 2.

6. Label the two ends of the mirrors U and W, as shown.


Since the initial beam is parallel to the mirror WV, then ∠UAS = 30o . Since the angle of
incidence equals the angle of reflection, then the reflected beam of light also makes an
angle of 30o with the mirror UV.

Let B be the point on the mirror WV that the light next strikes.
Since ∠VAB = ∠AVB = 30o , then the angle of incidence, ∠ABW , is equal to 60o , because
it is an external angle of ∆ABV . ( ∠ABW could also have been calculated using the facts
that ∠SAB = 120o and SA is parallel to WV.)
Therefore, the angle of reflection is also 60o .
Let C be the point on the mirror UV where the light next U
o o
strikes. Since ∠CVB = 30 and ∠VBC = 60 , then S 30° A 1
∠BCV = 90o . This tells us that the light is reflected 1 2
30° C 1
straight back along its path from C back to S. 2
Therefore, the required distance is 2( SA + AB + BC ) . W
60° 60° 30°
B V
Since we are given that SA = AV = 1, then since ∆ABV is isosceles with BC an altitude,
then AC = CV = 12 , and so BC = 1 ( AC ) = 1 and AB = 2 ( AC ) = 1 .
3 2 3 3 3
Therefore, the required distance is
(
2( SA + AB + BC ) = 2 1 + 1
3
+ 1
2 3 )=2+ 3
3
=2+ 3

( )
Thus, the total distance travelled by the beam is 2 + 3 m, or about 3.73 m.
2002 COMC Solutions 5

7. Solution 1
Since P is formed by adding a 1 at the end of N, then P = 10 N + 1.
Since Q is formed by adding a 1 in front of the 5 digits of N, then Q = 100000 + N .
Since P = 3Q ,
10 N + 1 = 3(100000 + N )
10 N + 1 = 300000 + 3N
7 N = 299999
N = 42857
Therefore, N is 42857.

Solution 2
Suppose N has digits abcde. Then since P = 3Q , we have abcde1 = 3(1abcde) .
Since the units digit on the left side is 1, then the units digit on the right is also 1, which
means that e = 7 .
Thus, abcd 71 = 3(1abcd 7) . Since the tens digit on the left side is 7 and we get a “carry” of
2 from multiplying the last digit on the right side by 3, then 3 × d has a units digit of 5, i.e.
d = 5.
Thus, abc 571 = 3(1abc 57) . Since the hundreds digit on the left side is 5 and we get a carry
of 1 from multiplying the last two digits on the right side by 3, then the units digit of 3 × c
must be a 4, i.e. c = 8 .
Thus, ab8571 = 3(1ab857) . In a similar fashion, we see that b = 2 and a = 4 .
Therefore, N = 42857 .

8. We are not told that M must be a positive integer, but it makes sense to look for a positive
integer M that satisfies these conditions, since we want the maximum possible value of M.
Since there are 1000 numbers in the set {1, 2, 3,K, 999,1000} and the probability that an x
1
chosen randomly from this set is a divisor of M is 100 , then M must have 10 divisors
between 1 and 1000.
Since we are told that M ≤ 1000 , then M must have exactly 10 positive divisors.
Therefore, M must be of the form p 9 where p is a prime number, or p 4 q where p and q are
both primes.
(Recall that to find the number of positive divisors of M, we find the prime factorization of
M and then take each of the exponents, add 1, and find the product of these numbers. For
example, if M = 48 = 2 4 3, then the number of positive divisors is ( 4 + 1)(1 + 1) = 10 .)
Now, we want to determine the maximum M in each of these two forms.
Case 1 M = p 9
Since M ≤ 1000 , then we must have p = 2 , i.e. M = 512 .
(If p = 3 , then p 9 = 19683 is too large.)
2002 COMC Solutions 6

Case 2 M = p 4 q
Since M ≤ 1000 and 5 4 = 625 , then we must have p = 2 or p = 3 .
If p = 2 , then the largest q can be so that q is prime and M ≤ 1000 is 61, i.e.
M = (16)(61) = 976 .
If p = 3 , then the largest q can be so that q is prime and M ≤ 1000 is 11, i.e.
M = (81)(11) = 891.
Therefore, the maximum possible value of M is 976.

Part B

1. (a) The slope of the line through P and F is


5 −1 1
=−
0−8 2
and so the slope of the desired line is also − 12 .
Since the point Q lies on the y-axis, then the y-intercept of the line is 3.
Therefore, the line is y = − 12 x + 3 .

(b) Since AD lies along the x-axis, then G is the y


point where the line from (a) crosses the x-axis. C(8, 8)
To find the coordinates of G, we set y = 0 in B(0, 8)

the line from (a) to get 0 = − 12 x + 3 or x = 6 .


P(0, 5)
Therefore, the desired line passes through the
points G(6, 0) and F (8,1) . Thus its slope is Q(0, 3)
1− 0 1
= , and so its equation is F(8, 1)
8−6 2 x
y − 0 = 12 ( x − 6) or y = 12 x − 3. A(0, 0) G(6, 0) D(8, 0)

(c) Since FG has slope 1


2
, then a line perpendicular to FG has slope −2 , the negative
1
reciprocal of 2
.
Since the desired line passes through H ( 4, 4 ) , it has equation y − 4 = −2( x − 4 ) or
y = −2 x + 12 .
2002 COMC Solutions 7

(d) The circle has centre H ( 4, 4 ) , and it is tangent to y

all four sides of the square, and so its radius must C(8, 8)
B(0, 8)
be 4, since the distance from the centre to each of
the four sides is 4.
P(0, 5)
Does this circle intersect the line y = 12 x − 3, i.e. H(4, 4)
the line through F and G? Q(0, 3)

F(8, 1)
x
A(0, 0) G(6, 0) D(8, 0)

We must find the shortest distance between the centre of the circle and the line, i.e. the
perpendicular distance. We already have the equation of a line through H that is
perpendicular to the line through F and G, the line y = −2 x + 12 . Where do these lines
intersect? Setting y-coordinates equal,
1
2
x − 3 = −2 x + 12
5
2
x = 15
x=6
i.e. the lines intersect at the point G(6, 0) ! Therefore, the shortest distance from H
to the line through F and G is the distance from H to G, which is
(6 − 4) 2 + (4 − 0) 2 = 20
i.e. is greater than than 4 = 16 , the radius of the circle.
Therefore, the circle does not intersect the line.

2. (a) For the product (2 A5)(13B) to be divisible by 36, we need it to be divisible by both 4
and 9. Since 2A5 is odd, it does not contain a factor of 2.
Therefore, 13B must be divisible by 4.
For a positive integer to be divisible by 4, the number formed by its last two digits must
be divisible by 4, i.e. 3B is divisible by 4, i.e. B = 2 or B = 6 .

Case 1 B = 2
In this case, 132 is divisible by 3, but not by 9. Therefore, for the original product to be
divisible by 9, we need 2A5 to be divisible by 3.
For a positive integer to be divisible by 3, the sum of its digits is divisible by 3, i.e.
2 + A + 5 = A + 7 is divisible by 3.
Therefore, A = 2 or 5 or 8 .

Case 2 B = 6
In this case, 136 contains no factors of 3, so for the original product to be divisible by 9,
we need 2A5 to be divisible by 9.
2002 COMC Solutions 8

For a positive integer to be divisible by 9, the sum of its digits is divisible by 9, i.e.
2 + A + 5 = A + 7 is divisible by 9. Therefore, A = 2 .

Therefore, the four possible ordered pairs are ( A, B) = (2, 2), (8, 2), (5, 2), (2, 6) .

(b) (i) If 10 a + b = 7 m , then b = 7 m − 10 a . Thus,


a − 2b = a − 2( 7 m − 10 a) = 21a − 14 m = 7( 3a − 2 m)
Since 3a − 2 m is an integer, then by definition, a − 2b is divisible by 7.

(ii) Solution 1
If 5c + 4 d is divisible by 7, then 5c + 4 d = 7 k for some integer k.
Therefore, d = 41 ( 7 k − 5c ) .
7( 3c − k )
So 4 c − d = 4 c − 41 ( 7 k − 5c ) = 41 (21c − 7 k ) = .
4
Since 4c − d is an integer, then 7( 3c − k ) must be divisible by 4. But 4 has no
3c − k
common factors with 7, so 4 must divide into 3c − k , i.e. is an integer.
4
 3c − k 
Therefore, 4 c − d = 7  , i.e. 4c − d is divisible by 7.
 4 

Solution 2
We note that 4 c − d = (14 c + 7 d ) − 2(5c + 4 d ) .
Since both terms on the right side are divisible by 7, then 4c − d is divisible by 7.

Solution 3
Multiplying the expression 4c − d by 5 does not affect its divisibility by 7.
Thus, we can consider whether or not 20c − 5 d is divisible by 7, and this will be
equivalent to considering 4c − d .
Since we are told that 5c + 4 d = 7 t for some integer t, then we know that
4 (5c + 4 d ) = 20c + 16 d = 28 t or 20c = 28 t − 16 d .
If we now consider 20c − 5 d , we see
20c − 5 d = (28 t − 16 d ) − 5 d
= 28 t − 21d
= 7( 4 t − 3d )
Since 20c − 5 d is divisible by 7 by definition, then 4c − d is divisible by 7.
2002 COMC Solutions 9

3. (a) We consider the possible cases. On his first turn, Alphonse can take either 1 marble or
2 marbles.
If Alphonse takes 1 marble, Beryl can take 2 marbles and then Colleen 1 marble, to
leave Alphonse with 1 marble left in the bowl. Therefore, Alphonse loses. (Note that
Beryl and Colleen can agree on their strategy before the game starts.)
If Alphonse takes 2 marbles, Beryl can take 1 marble and then Colleen 1 marble, to
leave Alphonse again with 1 marble left in the bowl. Therefore, Alphonse loses.
In either case, Beryl and Colleen can work together and force Alphonse to lose.

(b) Solution 1
On their two consecutive turns, Beryl and Colleen remove in total 2, 3 or 4 marbles.
On his turn, Alphonse removes either 1 marble or 2 marbles. Therefore, by working
together, Beryl and Colleen can ensure that the total number of marbles removed on
any three consecutive turns beginning with Alphonse’s turn is 4 or 5. (Totals of 3 and 6
cannot be guaranteed because of Alphonse’s choice.)

Therefore, if N is a number of marbles in which Alphonse can be forced to lose, then so


are N + 4 and N + 5 , because Beryl and Colleen can force Alphonse to choose from N
marbles on his second turn.

From (a), we know that 5 is a losing position for Alphonse. Also, 1 is a losing position
for Alphonse. (Since 1 is a losing position, then 5 and 6 are both losing positions,
based on our earlier comment.)

Since 5 and 6 are losing positions, then we can determine that 9, 10 and 11 are also
losing positions, as are 13, 14, 15, and 16. If we add 4 to each of these repeatedly, we
see that N is a losing position for every N ≥ 13 .

What about the remaining possibilities, i.e. 2, 3, 4, 7, 8, and 12?


For N = 2 or N = 3, if Alphonse chooses 1 marble, then either Beryl or Colleen is
forced to take the last marble, so these are not losing positions for Alphonse, i.e. they
are winning positions.
For N = 4 , if Alphonse chooses 2 marbles, then either Beryl or Colleen is forced to
take the last marble, so this is also not a losing position for Alphonse.

Next, we notice that if Alphonse chooses 1 marble, then the total number of marbles
chosen by the three players will be 3, 4 or 5, and if Alphonse chooses 2 marbles, then
the total number chosen will be 4, 5 or 6.
So if N = 7 , then Alphonse can choose 1 marble and ensure that he receives 2, 3 or 4
marbles on his next turn. So 7 is a winning position for Alphonse.
2002 COMC Solutions 10

If N = 8 , then Alphonse can choose 2 marbles and ensure that he receives 2, 3 or 4


marbles on his next turn. So 8 is also a winning position for Alphonse.

Lastly, we consider N = 12 .
If Alphonse chooses 1 marble, Beryl and Colleen can choose 1 each and return 9
marbles to Alphonse. As we have shown, this is a losing position for Alphonse.
If Alphonse chooses 2 marbles, Beryl and Colleen can choose 2 each and return 6
marbles to Alphonse. This is a losing position for Alphonse.

Therefore, the values of N for which Beryl and Colleen can force Alphonse to lose are
1, 5, 6, and all N for which N ≥ 9 .

Solution 2
First, we notice that if Alphonse chooses 1 marble, then the total number of marbles
chosen by the three players will be 3, 4 or 5, and if Alphonse chooses 2 marbles, then
the total number chosen will be 4, 5 or 6.

We define a “losing position” to be a number of marbles in the bowl so that if Alphonse


starts with this number, he can be forced to lose.

From (a), we know that 5 is a losing position for Alphonse. Also, 1 is a losing position
for Alphonse.
For N = 2 or N = 3, if Alphonse chooses 1 marble, then either Beryl or Colleen is
forced to take the last marble, so these are not losing positions (ie. they are winning
positions) for Alphonse.
For N = 4 , if Alphonse chooses 2 marbles, then either Beryl or Colleen is forced to
take the last marble, so this is a winning position for Alphonse.

How can we ensure that a starting position N ≥ 6 is not a losing position?


N will not be a losing position if either none of N − 3, N − 4 or N − 5 are losing
positions, or none of N − 4 , N − 5 or N − 6 are losing positions. (If either group
consists of three non-losing positions, then Alphonse can ensure that he gets a position
from the appropriate set at the beginning of his next turn by choosing either 1 or 2
marbles respectively.)

Also, N will be a losing position as long as at least one of N − 3, N − 4 , N − 5 and at


least one of N − 4 , N − 5 , N − 6 are losing positions. (If there is a losing position in
each group of 3, then no matter whether Alphonse chooses 1 or 2 marbles, then Beryl
and Colleen will be able to force Alphonse into one of these previously known losing
positions.)

Using these two criteria for checking whether a position is a losing position or not a
losing position, we can see
2002 COMC Solutions 11

i) N = 6 is a losing position, since N − 5 = 1 is a losing position in both groups of


three
ii) N = 7 is not a losing position, since N − 3, N − 4 , N − 5 (namely, 4, 3, 2) are not
losing positions.
iii) N = 8 is not a losing position, since N − 4 , N − 5 , N − 6 (namely, 4, 3, 2) are not
losing positions.
iv) N = 9 is a losing position, since N − 4 = 5 is a losing position in both groups of
three
v) N = 10 is a losing position, since N − 5 = 5 is a losing position in both groups of
three
vi) N = 11 is a losing position, since N − 5 = 6 is a losing position in both groups of
three
vii) N = 12 is a losing position, since N − 3 = 9 and N − 6 = 6 are both losing
positions

And so we have obtained 4 consecutive losing positions, which guarantees us that any
N ≥ 13 will also be a losing position, since N − 4 will be a losing position in both
groups of 3.

Therefore, the values of N for which Beryl and Colleen can force Alphonse to lose are
1, 5, 6, and all N for which N ≥ 9 .

But among the first eight possibilities, there are now no more sets of three consecutive
non-losing positions. This tells us that every position for N ≥ 9 is a losing position,
since we cannot find three consecutive non-losing positions as described above.

Therefore, the values of N for which Beryl and Colleen can force Alphonse to lose are
1, 5, 6, and all N for which N ≥ 9 .

4. Solution 1
Join E to P, Y and R, and join F to Q, Z and S. D
Let O be the point of intersection of EY and FZ. R
Since EY and FZ are altitudes in ∆DEF , then the S
third altitude, DX say, passes through O. Y
Z O
If we look at altitude DX, we see that ∠DXE = 90o .
Since circle C2 has DE as its diameter, then point X P Q
must lie on circle C2 , since a right angle is subtended E X F
by the diameter at point X.

Similarly, point X lies on circle C1 .


Therefore, DX is a chord of both circle C1 and circle C2 .
We can now use the “Chord-Chord Theorem” in each of circle C1 and C2 , to say
2002 COMC Solutions 12

SO ⋅ OQ = DO ⋅ OX (from circle C2 )
RO ⋅ OP = DO ⋅ OX (from circle C1 )
From this we can conclude that SO ⋅ OQ = RO ⋅ OP .

Why does this allow us to conclude that P, Q, R, and S lie on the same circle?
SO RO
From the equation, we obtain = , which tells us that ∆SOP is similar to ∆ROQ ,
OP OQ
and so ∠PSQ = ∠PSO = ∠ORQ = ∠PRQ .
Since the chord PQ subtends the equal angles ∠PSQ and ∠PRQ (in an undrawn circle),
then the points P, Q, R, and S are concyclic.

Solution 2
In order to show that the four points lie on a circle, we will show that the points are
equidistant from a fifth point, which will thus be the centre of the circle on which the four
points lie.
Consider first the points Q and S. Any point equidistant from Q and S lies on the
perpendicular bisector of the line joining these points. Since Q and S both lie on circle C2 ,
DE is a diameter of C2 , and QS is perpendicular to DE (since they lie on an altitude of the
triangle), then DE is the perpendicular bisector of QS.
Similarly, DF is the perpendicular bisector of PR.
Therefore, any point that is equidistant from all four of the given points must lie on both
DE and DF. Thus, the only possible candidate is point D. (And we already know that
DS = DQ and DP = DR from our discussion of perpendicular bisectors.)

Thus, if we can show that DS = DR , then we will have shown what we need to show.

Method 1
Let SZ = c , DZ = a and EZ = b . D
Then DS 2 = DZ 2 + SZ 2 = c 2 + a 2 (Pythagoras). R
Now if we extract ∆DSE , we see that ∠DSE = 90o , S a
Z Y
since DE is a diameter of circle C2 . Therefore, c O
DZ SZ b
∆DSZ is similar to ∆SEZ , or = or c 2 = ab .
SZ EZ P Q
Thus, DS 2 = a 2 + ab = a( a + b) = DZ ⋅ DE . E F
Similarly, DR 2 = DY ⋅ DF , looking at ∆DRF .

Now consider the points E, Z, Y, and F. Since ∠EZF = ∠EYF = 90o , then EF must be the
diameter of the circle containing points Y and Z (and points E and F).
Therefore, DE and DF are secants of the circle which intersect the circle at Z and Y,
respectively. By the “Secant-Secant Theorem”, DZ ⋅ DE = DY ⋅ DF .
2002 COMC Solutions 13

From above, we can conclude that DS 2 = DR 2 , or DS = DR , and thus


DP = DQ = DR = DS .

Method 2
As above, we can obtain that DS 2 = a 2 + ab = a( a + b) = DZ ⋅ DE .
Since ∠DZF = 90o , then DZ = DF cos(∠ZDF ) = DF cos(∠EDF ) , and so
DS 2 = DZ ⋅ DE = DE ⋅ DF cos(∠EDF ) .
Repeating the process on the other side of the triangle gives us that
DR 2 = DY ⋅ DF = DF ⋅ DE cos(∠EDF ) , or DR 2 = DS 2 .
Therefore, DP = DQ = DR = DS .
Therefore, we can conclude that the points P, Q, R, and S are concyclic.
The Canadian Mathematical Society
in collaboration with

The CENTRE for Education


in MATHEMATICS and COMPUTING

The
Canadian Open
Mathematics Challenge

Wednesday, November 26, 2003

Solutions

© 2003 Canadian Mathematical Society


2004 Canadian Open Mathematics Challenge Solutions 2

Part A

1. Let Gareth’s present age, in years, be G.


Then Jeff’s age is G − 1, and Ina’s age is G + 2.
Since the sum of their three ages is 118, then
(G − 1) + G + (G + 2) = 118
G = 39
Therefore, Gareth’s age is 39.

2. When the point (4,−2 ) is reflected in the x-axis, its image is (4,2) .
When the point (4,2) is reflected in the line y = x , its image is (2,4) .
Therefore, the coordinates of the final point are (2,4) .

3. The particle which moves clockwise is moving three times as fast as the particle moving
counterclockwise. Therefore, the particle moving clockwise moves three times as far as the
particle moving counterclockwise in the same amount of time.

This tells us that in the time that the clockwise particle y


travels 34 of the way around the circle, the counterclockwise
P(0, 1)
particle will travel 14 of the way around the circle, and so the
two particles will meet at P(0,1) .

x
(1, 0)

Using the same reasoning, the particles will meet at Q(−1, 0) y


when they meet the second time.
P
Q(–1, 0)
x
2004 Canadian Open Mathematics Challenge Solutions 3

4. Solution 1
In choosing a pair of numbers from the five given numbers, there are 10 different ways of choosing
these numbers. These pairs are {(0,1), (0,2),(0,3),(0,4), (1,2), (1,3),(1,4),(2,3), (2,4 ), (3,4 )} . The only
pairs in which the sum is greater than the product are those containing a 0 or a 1. Since there are 7
of these, the required probability is 107 .

Solution 2
The most straightforward way to approach this problem is to make a chart:
Numbers chosen Sum Product
0,1 1 0
0,2 2 0
0,3 3 0
0,4 4 0
1,2 3 2
1,3 4 3
1,4 5 4
2,3 5 6
2,4 6 8
3,4 7 12

So there are 10 possible ways that two different numbers can be chosen, and for 7 of these
possibilities, the sum of the two numbers is greater than the product.
7
Therefore, the probability is 10 .

5. Join A to C.
This line divides the shaded region into two identical pieces. A B

D C
Consider the shaded region above AC.
This piece of the region is formed by taking the sector DAC of A
the circle, centre D and radius 6, and then removing ∆ADC .
6

D 6 C
2004 Canadian Open Mathematics Challenge Solutions 4

o
Since ∠ADC = 90 , then the sector is one quarter of the whole circle, and has area
2
( )
2
4 π r = 4 π 6 = 9π .
1 1

Also, ∆ADC is right-angled with base DC of length 6 and height DA of length 6, and so has area
2 bh = 2 (6)(6) = 18.
1 1

Therefore, the area of the region above the line is 9π − 18 , and so the area of the entire shaded
region is 2(9π − 18) = 18π − 36 square units.

< 0 , so ⎢⎢ ⎥⎥ ≤ < 0. Similarly, ⎢⎢ ⎥⎥ < 0 , so we cannot possibly have


3 3 3 4
6. If x < 0 , then
x ⎣x⎦ x ⎣x⎦
⎢ 3 ⎥ + ⎢ 4 ⎥ = 5 . Therefore, x > 0 .
⎢⎣ x ⎥⎦ ⎢⎣ x ⎥⎦

< , so ⎢⎢ ⎥⎥ ≤ ⎢⎢ ⎥⎥ .
3 4 3 4
When x > 0 , we have
x x ⎣x⎦ ⎣ x⎦
⎢ ⎥
3 ⎢ ⎥
4
Since each of ⎢ ⎥ and ⎢ ⎥ is an integer, then there are three possibilities:
⎣x⎦ ⎣x⎦
i) ⎢⎢ ⎥⎥ = 0 and ⎢⎢ ⎥⎥ = 5
3 4
⎣x⎦ ⎣x⎦
ii) ⎢⎢ ⎥⎥ = 1 and ⎢⎢ ⎥⎥ = 4
3 4
⎣x⎦ ⎣x⎦
iii) ⎢⎢ ⎥⎥ = 2 and ⎢⎢ ⎥⎥ = 3
3 4
⎣x⎦ ⎣x⎦

If ⎢⎢ ⎥⎥ = 0 , then 0 ≤ < 1 or x > 3. If ⎢⎢ ⎥⎥ = 5, then 5 ≤ < 6 or < x ≤ . These intervals do


3 3 4 4 2 4
⎣x⎦ x ⎣x⎦ x 3 5
not overlap, so there are no solutions in this case.
If ⎢⎢ ⎥⎥ = 1, then 1 ≤ < 2 or < x ≤ 3 . If ⎢⎢ ⎥⎥ = 4 , then < x ≤ 1 . These intervals do not
3 3 3 4 4
⎣x⎦ x 2 ⎣x⎦ 5
overlap, so there are no solutions in this case.
If ⎢⎢ ⎥⎥ = 2 , then 2 ≤ < 3 or 1 < x ≤ . If ⎢⎢ ⎥⎥ = 3, then 1 < x ≤ . In this case, the intervals do
3 3 3 4 4
⎣x⎦ x 2 ⎣x⎦ 3
overlap. When we combine these intervals, we see that if 1 < x ≤ , then ⎢⎢ ⎥⎥ + ⎢⎢ ⎥⎥ = 5 .
4 3 4
3 ⎣x⎦ ⎣x⎦
4
Therefore, the range of values is 1 < x ≤ .
3
2004 Canadian Open Mathematics Challenge Solutions 5

7. Solution 1
Let the radius of circle C be r.
1 1
r
Since P, Q and R are given as midpoints of the radius of the 2 2
r

large circle, they themselves lie on a circle with the same centre P(4, 1) R(10, 1)
1 1
as the given circle, but with half its radius. 2
r
2
r
1
r
2
1
r
Q(7, – 8)
2

Method 1 – Perpendicular bisectors


To find the centre of the circle passing through P, Q and R, we must find the intersection of the
perpendicular bisectors of the sides of the triangle formed by the three points.

Consider first side PR. Since PR is a line segment parallel to (7, 1)


P(4, 1) R(10, 1)
the x-axis, its perpendicular bisector has equation x = 7 .

⎛ 11, 7 ⎞
⎝ 2 2⎠
Q(7, – 8)

Consider next side PQ. Since P has coordinates (4,1) and Q has coordinates (7,−8 ) , then PQ
has slope –3 and has midpoint ( 112 ,− 72) . Therefore, the perpendicular bisector of PQ has slope
3 and has equation y + 2 = 3 (x − 2 ) .
1 7 1 11

Therefore, at the intersection of these two perpendicular bisectors, y + 72 = 1


3 (7 − 112) or y = −3 .
Thus the centre of the circle is the point (7,−3) , and since (7,−8 ) lies on the circle, the radius of
the small circle is 5. Therefore, the radius of circle C is 10.

Method 2 – Geometric
As in Method 1, we proceed by trying to find the centre of P(4, 1) (7, 1) R(10, 1)
the circle. Also, we again know that the centre is the 1
r (7, b)
2
intersection of the perpendicular bisectors of the sides of the 1
r
∆PQR . One perpendicular bisector is very easy to find – 2

that of PR, which has equation x = 7 as we found above.


Q(7, – 8)
This tells us that the centre lies on the line x = 7 . Thus, the
centre of the circle can be represented by O( 7,b) .
2004 Canadian Open Mathematics Challenge Solutions 6

Since radii of a circle are equal,


2 2
OP = OQ
(7 − 4) 2 + (b − 1)2 = (7 − 7)2 + (b + 8)2
9 + b 2 − 2b + 1= b2 + 16b + 64
b = −3
32 + (−4 ) = 5 and the radius of the larger circle is 10.
2
The radius of the circle P, Q and R is

8. The first thing that we must notice in this problem is because we are looking for positive integers k,
l and m such that
4k 5l 6m
+ + = 82
5 6 7
then k must be divisible by 5, l must be divisible by 6, and m must be divisible by 7.
So we make the substitution k = 5K , l = 6L and m = 7M , where K, L and M are positive integers.
Therefore, we obtain, by substitution
5K + 6L + 7M = 97
4K + 5L + 6M = 82
Subtracting the second equation from the first, we get
K + L + M = 15
4K + 5L + 6M = 82
Subtracting six times the first equation from the second equation, we get
K + L + M = 15
−2K − L = −8
or
K + L + M = 15
2K + L = 8
Since K, L and M are all positive integers, we obtain from the second equation the following
possibilities for K and L which give us M from the first equation, and thus k, l and m:

K L M k l m
1 6 8 5 36 56
2 4 9 10 24 63
3 2 10 15 12 70

Therefore, there are three triples (k,l,m) of positive integers which are solutions to the system of
equations.
2004 Canadian Open Mathematics Challenge Solutions 7

Part B

1. (a) We will proceed systematically to fill in the circles based on the initial knowledge that
k = 2 and e = 5 :
i = 10 (5, i)
[Throughout the solution to this problem, we have used notation such as (5, 7, c) to indicate
that there is a straight line joining the circles containing 5, 7 and c. Thus, 5 + 7 + c = 15 or
c = 3.]
b=8 (2, 5, b) k 2
d=7 (8, d)
c=3 (5, 7, c)
a= 4 (a, 8, 3) i 10
g= 9 (4, g, 2)
1
f =6 (4, f, 5)
9 g h e 5
h =1 (9, h, 5) f 6
d 7

4 a b c 3
8
We can verify that the numbers along each of the ten straight lines add to 15.

(b) (i) Solution 1


We start with knowing that k = 2 and e is unknown.
Then k 2
i = 15 − e (e, i)
c = 15 − 2 − (15 − e) = e − 2 (2, 15 − e , c)
b = 15 − 2 − e = 13− e (2, e, b) i 15 – e
Therefore, b = 13 − e and c = e − 2 .
g h e
f
d

a b c
13 – e e–2
2004 Canadian Open Mathematics Challenge Solutions 8

Solution 2
We start with knowing that k = 2 and e is k 2
unknown.
Then
i = 15 − e (e, i) i 15 – e
c = 15 − 2 − (15 − e) = e − 2 (2, 15 − e , c)
d = 15 − e − (e − 2) = 17 − 2e (e, d, e − 2 ) g h e
b = 15 − (17 − 2e) = 2e − 2 (b, 17 − 2e ) f 17 – 2e
Therefore, b = 2e − 2 and c = e − 2 . d

a b c
2e – 2 e–2

(ii) Solution
We know from (i) that k = 2 , b = 13 − e , k 2
and c = e − 2 .
Therefore, d = 15 − b = 15 − (13− e ) = 2 + e .
But we also know that i 15 – e
e + d + c = 15
e + (2 + e ) + (e − 2) = 15 g h e
3e = 15 f 2+e
e=5 d
Therefore, e = 5 .
a b c
13 – e e–2

(c) Solution
We can model our approach from (b). k x
Starting with e being unknown and k = x , we
express some of the circles in terms of x and e:
i = 15 − e (e, i) i 15 – e
c = 15 − x − (15 − e) = e − x (x, 15 − e , c)
b = 15 − x − e (x, e, b) g h e
d = 15 − (15 − x − e) = x + e (15 − x − e , d) f x+e
But we know that d
e + d + c = 15
e + ( x + e ) + (e − x ) = 15 a b c
15 – x – e e–x
3e = 15
e=5
Therefore, e must still be equal to 5.
2004 Canadian Open Mathematics Challenge Solutions 9

2. (a) Solution 1
Drop perpendiculars from D and A to BC, meeting BC at E and F respectively.
Since DA is parallel to CB, then DE and AF are also perpendicular to DA.
Since DAFE is a rectangle, then EF = 6 .
Since DC = AB , DE = AF and ∆DEC and ∆AFB are right- D 6 A
angled, then they are congruent triangles, and so CE = BF ,
6 6
and so both of these lengths must be equal to 3.

C 3E 6 F 3 B

2 2 2 2
By the Pythagorean Theorem, DE = DC − CE = 6 − 3 = 27 = 3 3 .
Therefore, the sides of ∆DEC are in the ratio 1 to 3 to 2, so DEC is a 30-60-90 triangle, with
o o
∠DCE = 60 and ∠CDE = 30 .
o
Therefore, since we have congruent triangles, ∠DCB = ∠ABC = ∠DCE = 60 and
o o
∠CDA = ∠DAB = 90 + ∠CDE = 120 .

Solution 2
Join D to the midpoint M of CB.
Then CM = MB = 6 .
Since DM and MB are parallel and of equal length, then AB D 6 A
and DM will also be parallel and equal length.
6 6
Thus, DM = 6 , and so ∆DCM is equilateral.

C 6 M 6 B
o o
Therefore, ∠DCB = ∠DCM = 60 . By symmetry, ∠ABC = ∠DCB = 60 .
o
Since DA and CB are parallel, ∠CDA = ∠DAB = 120 .

(b) (i) If Chuck was attached to a point P and there were no obstructions, he would be able to
reach a circle of radius 8 m. (If Chuck stays at the end of his chain, he could trace out a
circle of radius 8 m, but Chuck can move everywhere inside this circle, since his chain does
not have to be tight.) However, here we have an obstruction – the trapezoidal barn.

o
Since the interior angle of the barn at point A is 120 , then the exterior angle of the barn is
o o
240 . So Chuck can certainly reach the area which is a 240 sector of radius 8 m, centred
at A. (If Chuck extends the chain as far as possible in a straight line in the direction of D
from A, can then walk in a clockwise direction, keeping the chain at its full length until the
o
chain lies along AB. He will have moved through 240 , and the region is the sector of a
circle.)
2004 Canadian Open Mathematics Challenge Solutions 10

However, when the chain is fully extended in the


direction of D, Chuck will be 2 m past point D.
He will thus be free to move towards side DC of
the barn. If he does this and keeps the chain tight,
he will trace out part of a circle of radius 2 m
centred at D. (Point D now serves as a “pivot” 240°
point for the chain.) Since the exterior angle of 2 D 6 A
o 60° 120°
the barn at point D is 240 , then the angle 2
o 6
between AD extended and DC is 60 . Therefore,
o 60° B
Chuck can reach a 60 sector of a circle of radius
C 120° 2
2 m, centred at D. 2

When the chain is fully extended in the direction of B, Chuck will be 2 m past point B. He
will thus be free to move towards side BC of the barn. If he does this and keeps the chain
tight, he will trace out part of a circle of radius 2 m centred at B. (Point B now serves as a
o
“pivot” point for the chain.) Since the exterior angle of the barn at point B is 300 (the
o o
interior angle at B is 60 ), then the angle between AB extended and BC is 120 . Therefore,
o
Chuck can reach a 120 sector of a circle of radius 2 m, centred at B.

o θ
The area of a sector of angle θ of a circle of radius r is πr 2 .
360

Therefore, the total area that Chuck can reach is


240 60 120 2 1 1 128π 134π
(π )82 + (π )2 2 + (π )2 2 = (64π ) + (4π ) + (4π ) = + 2π =
360 360 360 3 6 3 3 3
square metres.

(ii) Let x be the distance along AB from A to P.


Since the total perimeter of the barn is 30 m and Chuck is attached with a 15 m chain, then
Chuck can reach the same point on the barn whether he wraps the chain around the barn in
a clockwise direction or a counterclockwise direction. This point will move, however, as P
moves. For example, if Chuck was attached at point A (ie. if x = 0 ), then he could reach a
point 3 m along CB from C towards B wrapping in either the clockwise or counterclockwise
direction. If Chuck was attached at point B (ie. if x = 6 ), he could reach the midpoint of
CD in either direction. As point P moves from A towards B, this furthest point on the barn
that Chuck can reach will slide along BC towards C and then up CD towards D. If P is at
the midpoint of AB (ie. if x = 3), the furthest point along the barn that he can reach will be
point C. So in our analysis, we must be careful as to whether 0 ≤ x ≤ 3 or 3 ≤ x ≤ 6 .

o
Regardless of the value of x, Chuck can certainly reach a 180 sector of a circle of radius 15
centred at P.
2004 Canadian Open Mathematics Challenge Solutions 11

We start in the counterclockwise


direction.
Also, regardless of the value of x, Chuck
o 15 – x
can reach a 60 sector of a circle of radius
15 − x centred at A (using A as the new
pivot point for the chain). 9–x D 6 60° A
Still regardless of the value of x, Chuck 60° x
o 6 6–x
can reach a 60 sector of a circle of radius
9 − x = (15 − x ) − 6 centred at D (using D C 9+x B
3–x 120° 120°
as the new pivot point for the chain).
9+x
If 3 ≤ x ≤ 6 , then 9 − x ≤ 6 , so Chuck
cannot reach past point C.

o
If 0 ≤ x ≤ 3, then 9 − x ≥ 6 , so Chuck can reach past point C, and so can reach a 120 sector
of a circle of radius 3− x = (9 − x ) − 6 centred at C (using C as the new pivot point for the
chain).
Next, we consider the clockwise direction.
o
Regardless of the value of x, Chuck can reach a 120 sector of a circle of radius
9 + x = 15 − (6 − x ) (the distance from B to P is 6 − x ) centred at B (using B as the new
pivot point for the chain).
If 0 ≤ x ≤ 3, then 9 + x ≤ 12 , so Chuck cannot reach past point C.
o
If 3 ≤ x ≤ 6 , then 9 + x ≥ 12 , so Chuck can reach past point C, and so can reach a 120
sector of a circle of radius x − 3 = (9 + x ) − 12 centred at C (using C as the new pivot point
for the chain).

We now calculate the total area that Chuck can reach.


If 0 ≤ x ≤ 3, then the area that Chuck can reach is
180 60 60 120 120
π15 2 + π (15 − x ) 2 + π (9 − x )2 + π (3 − x )2 + π (9 + x )2
360 360 360 360 360
1
2
1
6
( ) (
1
6
) 1
(
= π (225) + π 225 − 30x + x 2 + π 81− 18x + x 2 + π 9 − 6x + x 2 + π 81+ 18x + x 2
3
1
3
) ( )
1
(
= π 675 + 225 − 30x + x 2 + 81− 18x + x 2 + 18 − 12x + 2x 2 + 162 + 36x + 2x 2
6 )
1
(
= π 1171− 24x + 6x 2
6
)
387
= πx 2 − 4πx + π
2
2004 Canadian Open Mathematics Challenge Solutions 12

If 3 ≤ x ≤ 6 , then the area that Chuck can reach is

180 60 60 120 120


π15 2 + π (15 − x ) 2 + π (9 − x )2 + π (9 + x ) 2 + π ( x − 3)2
360 360 360 360 360
387
= πx 2 − 4πx + π
2

(Notice that the one term between these two initial expressions that seems to be different is
actually the same!)

Therefore, no matter what the value of x is, the area that Chuck can reach is
387
πx 2 − 4πx + π . This is a parabola opening upwards, so the vertex of the parabola gives
2
−4π
us the minimum of the parabola. This vertex is at x = − = 2 . Since x = 2 is between
2(π )
the endpoints of the allowable interval (0 and 6), then this will give the minimum.

Therefore, the location of P which minimizes the area that Chuck can reach is 2 m along the
wall from A towards B.

3. (a) Solution 1
Let ∠PAB = θ . P
A 5
o
Then ∠XAB = 180 − θ , and so ∠XYB = θ since XYBA is 16 θ C1
0° –θ 7
a cyclic quadrilateral, and so opposite angles sum to X 18 6
o
180 .
B
Therefore, ∆PAB is similar to ∆PYX (common angle at C2
θ
P, equal angle θ ).
XY PX BA ⋅ PX 6(5 + 16) Y
So, = or XY = = = 18 .
BA PB PB 7

Solution 2
By the cosine law in ∆APB ,
AB2 = PA 2 + PB 2 − 2(PA)( PB) cos(∠APB)
36 = 25 + 49 − 2(5)(7) cos(∠APB)
cos(∠APB) = 38
70
= 19
35
2004 Canadian Open Mathematics Challenge Solutions 13

Now, PX and PY are both secants of circle C2 , so by the P


A 5
Secant-Secant Theorem, 16
PA ⋅ PX = PB ⋅ PY 6
7
X
5(5 + 16) = 7(7 + BY )
8 B
105 = 7(7 + BY )
15 = 7 + BY Y
BY = 8

Now in ∆PXY we know the lengths of sides PX (length 21), PY (length 15) and the cosine of
∠XPY = ∠APB , so we can use the cosine law to calculate the length of XY.
XY 2 = PX 2 + PY 2 − 2(PX )(PY ) cos(∠XPY )
XY 2 = 441+ 225 − 2(21)(15) ( 1935 )
XY 2 = 441+ 225 − 2( 3)( 3)(19)
2
XY = 441+ 225 − 342
XY 2 = 324
XY = 18
Therefore, the length of XY is 18.

(b) Solution 1
Since the circle C4 is fixed, the length VW will be fixed if the angle it subtends on the circle is
fixed, ie. if the angle ∠VHW does not depend on the position of Q.
Now Q
o
G
∠VHW = 180 − ∠VHQ C 3
o
( o
= 180 − 180 − ∠GVH − ∠GQH ) V
= ∠GVH + ∠GQH H
C4

W
But since chord GH is a chord of fixed length in both circles (ie. it doesn’t change as Q moves),
then the angles that it subtends in both circles are constant. In particular, ∠GVH and ∠GQH
are both fixed (that is, they do not depend on the position of Q).
Since both of these angles are fixed, then ∠VHW = ∠GVH + ∠GQH is also fixed.
Therefore, the length of VW is fixed.
2004 Canadian Open Mathematics Challenge Solutions 14

Solution 2
We start by noting that chord GH has a constant length, Q
G
ie. does not depend on the the position of Q. Thus, GH is C3
a constant chord in both C3 and C4 .
V
In C3 , let ∠GQH = α .
H
In C4 , let ∠GVH = ∠GWH = β . C4
These angles are constant since GH is of constant length.
W
Therefore, ∠VHQ = ∠QGW = 180 − (α + β ) , and so ∠VGW = ∠WHV = α + β .
o

Since these last two angles are constant for all position of Q, then VW is a chord of constant
length.

Solution 3
We proceed by considering two different positions for the point Q, which we call Q1 and Q2 .
These points will create two different positions for the line segment VW, which we call V1 W1
and V2 W2 .
To show that the length of VW is constant, we must show that V1 W1 and V2 W2 have the same
length. We do know that the points G and H are fixed, so the length of GH does not vary.

Q1 V2
G
G Q2

H
H
V1
W1
W2

As in part (a) Solution 1, ∆Q1 HG and ∆Q1V1W1 are similar triangles, as are ∆Q2 HG and
∆Q2V2W2 . (The two positions for Q play the same role as P, points G and H play the same role
as A and B, and the points V and W play the same role as X and Y.)
VW QV QV
Therefore, by similar triangles, 1 1 = 1 1 or V1 W1 = HG ⋅ 1 1 .
HG Q1 H Q1 H
VW QV QV
Also, by similar triangles, 2 2 = 2 2 or V2 W2 = HG ⋅ 2 2 .
HG Q2 H Q2 H
Since the length of HG (or GH) does not change, to show that V1 W1 and V2 W2 have the same
QV Q V
length, we must show that 1 1 = 2 2 .
Q1 H Q2 H

Join H to each of V1 and V2 .


Now GH is a chord of fixed length in both circles, so the angle that it subtends at any point on
the circumference of each circle is the same.
2004 Canadian Open Mathematics Challenge Solutions 15

Therefore, ∠GQ1 H = ∠GQ2 H and ∠GV1 H = ∠GV2 H .


But this tells us that ∆Q1 HV1 and ∆Q2 HV2 are similar.
QV QH QV Q V
Thus, 1 1 = 1 or 1 1 = 2 2 .
Q2 V2 Q2 H Q1 H Q2 H
QV QV
Therefore, V1 W1 = HG ⋅ 1 1 = HG ⋅ 2 2 = V2 W2 , ie. the length of VW is constant.
Q1 H Q2 H

4. (a) Solution 1
3 2
Since a, b and c are the roots of the equation x − 6x + 5x − 1 = 0 , then using the properties of
the coefficients of a cubic equation,
a +b+c =6
ab + ac + bc = 5
abc = 1
Since we know that each of a, b and c is a roots of the equation, then
a 3 − 6a2 + 5a − 1 = 0
b3 − 6b2 + 5b − 1 = 0
c3 − 6c 2 + 5c − 1 = 0
or after rearranging
a 3 = 6a2 − 5a + 1
b 3 = 6b2 − 5b + 1 (*)
3 2
c = 6c − 5c + 1
Adding these three equations, we obtain
a 3 + b3 + c3 = 6a2 + 6b 2 + 6c 2 − 5a − 5b − 5c + 3
( )
= 6 a 2 + b 2 + c 2 − 5( a + b + c ) + 3
2 2 2
We already know that a + b + c = 6, so if we could determine the value of a + b + c , then
3 3 3
we would know the value of a + b + c .
But
(a + b + c) 2 = a2 + b2 + c 2 + 2ab + 2bc + 2ac
6 2 = a2 + b2 + c 2 + 2( ab + bc + ac)
a 2 + b 2 + c 2 = 36 − 2(5)
a 2 + b 2 + c 2 = 26
and so
( )
a 3 + b3 + c3 = 6 a 2 + b 2 + c 2 − 5( a + b + c ) + 3 = 6(26) − 5(6) + 3 = 129

If we know take the equations in (*) and multiply both sides in the first, second and third
equations by a, b and c, respectively, we get
2004 Canadian Open Mathematics Challenge Solutions 16

a 4 = 6a 3 − 5a2 + a
b 4 = 6b 3 − 5b 2 + b (**)
4 3 2
c = 6c − 5c + c
which we then can add to obtain
4 3
(3 3 2 2
) (
a + b + c = 6 a + b + c − 5 a + b + c + (a + b + c )
4 4 2
)
= 6(129) − 5(26) + 6
= 650
Repeating the process one more time by multiplying the first, second and third equations in
(**) by a, b and c, respectively, and adding, we obtain
5 5 5 4
( 4 4
) ( 3
a +b +c = 6 a +b +c −5 a +b +c + a +b +c
3 3 2
) (
2 2
)
= 6(650) − 5(129) + 26
= 3281
5 5 5
Therefore, the value of a + b + c is 3281.

Solution 2
3 2
Since a, b and c are the roots of the equation x − 6x + 5x − 1 = 0 , then using the properties of
the coefficients of a cubic equation,
s= a +b+ c =6
t = ab + ac + bc = 5
p = abc = 1
5 5 5
We will attempt to express a + b + c in terms of s, t and p, which will thus allow us to
5 5 5
calculate the value of a + b + c .

First,
(a + b + c) 2 = a2 + b2 + c 2 + 2ab + 2bc + 2ac
a 2 + b 2 + c 2 = s2 − 2t
Next,
( )
a 2 + b2 + c2 (a + b + c) = a3 + b3 + c 3 + a2 b + a 2 c + b2 a + b2 c + c 2 a + c 2b
3 3 3 2
( 2
) [ 2 2 2 2 2
a + b + c = s s − 2t − a b + a c + b a + b c + c a + c b ]
= s( s2 − 2t ) − [(ab + ac + bc )( a + b + c ) − 3abc]

= s( s2 − 2t ) − [ts − 3p ]

= s3 − 3st + 3p
5 5 5
We can now attempt to express a + b + c as
2004 Canadian Open Mathematics Challenge Solutions 17

a 5 + b5 + c 5
( )( ) [
= a 2 + b2 + c 2 a3 + b 3 + c3 − a2 b3 + a 2 c3 + b 2 a3 + b2 c 3 + c 2 a3 + c 2 b3 ]
= (a 2 + b2 + c 2 )(a3 + b 3 + c3 ) − [(a 2 b2 + a2 c 2 + b 2c 2 )(a + b + c ) − (a 2 b2 c + a 2 bc 2 + ab2 c 2 )]

= (s2 − 2t )( s3 − 3st + 3p) − [[(ab + ac + bc )2 − 2( a2 bc + ab 2 c + abc 2 )](a + b + c ) − abc (ab + ac + bc )]

= (s2 − 2t )( s3 − 3st + 3p) − [[t 2 − 2abc( a + b + c )]s − pt ]

= (s2 − 2t )( s3 − 3st + 3p) − [[t 2 − 2ps]s − pt ]

= (62 − 2( 5))(63 − 3(6 )(5) + 3(1)) − [[ 52 − 2(1)(6)](6 ) − 1(5)]

= (26)(129) − [[13](6 ) − 5]
= 3354 − [73]
= 3281

5 5 5
Therefore, the value of a + b + c is 3281.

(b) We will proceed by dividing our proof into several steps.


Step 1: Estimate the values of a, b and c
n n n
Step 2: Show that a + b + c is an integer for every positive integer n
Step 3: Final conclusion

Step 1: Estimate the values of a, b and c


Define f ( x ) = x 3 − 6x 2 + 5x − 1.
If x < 0 , then x < 0 , −6 x < 0 , 5x < 0 , and −1 < 0 , so f ( x ) = x 3 − 6x 2 + 5x − 1< 0.
3 2

This tells us that f ( x ) = 0 cannot have any negative roots. It is also clear that 0 is not a
root of f ( x ) = 0 , so each of a, b and c is positive.
If we calculate a few values of f ( x ) = x 3 − 6x 2 + 5x − 1, we obtain f (0) = −1 , f (1) = −1 ,
f (2) = −7 , f ( 3) = −13 , f ( 4) = −13 , f (5) = −1 , and f (6) = 29 .
Therefore, one of the roots is between 5 and 6.
However, we know from part (a) that a + b + c = 6 , so since all three roots are positive, then
we must have 5 < c < 6 and 0 < a,b < 1 . (Since one root is bigger than 5, each root is
positive, and the sum of the three roots is 6, then neither a nor b is bigger than 1.)
Since 5 < c < 6 and a + b + c = 6 , then 0 < a + b < 1.
We also know from part (a) that abc = 1. Since 5 < c < 6 , then 16 < ab < 15 and since each
1
of a and b is less than 1, then each of a and b must be bigger than 6.
Since a and b are each bigger than 1
6 and 0 < a + b < 1, then 1
6 < a,b < 56 .
(We could have proceeded less formally by doing some quick calculations to see that
f (0.1) = −0.559 , f (0.2) = −0.232 , f (0.3) = −0.013 , f (0.4 ) = 0.104 , f (0.5) = 0.125 ,
2004 Canadian Open Mathematics Challenge Solutions 18

f (0.6) = 0.056 , f (0.7) = −0.097 , and so a must be between 0.3 and 0.4, and b must be
between 0.6 and 0.7.)

n n n
Step 2: Show that a + b + c is an integer for every positive integer n
n n n
In part (a), we saw that a + b + c is definitely an integer for n equal to 1, 2 and 3.
If we return to the set of equations (*) in Solution 1 of part (a) and multiply the three
n −3 n− 3 n− 3
equations by a , b and c , respectively, we obtain
n −3
a = 6a
n n− 1
− 5a n− 2
+a
bn = 6bn −1 − 5b n− 2 + b n− 3 (***)
n n −1
c = 6c − 5c n− 2 + c n− 3
and adding, we get
n n n
a +b +c =6 a (
n− 1
+b
n− 1
+c
n− 1
−5 a ) (
n− 2
+b
n− 2
+c
n− 2
) (
+ a
n −3
+b
n −3
+c
n −3
) (****)
for every n greater than or equal to 4.
k k k
If we set n equal to 4, then since a + b + c is an integer for k equal to 1, 2 and 3, then by
4 4 4
(****), a + b + c is also an integer.
k k k
If we set n equal to 5, then since a + b + c is an integer for k equal to 2, 3 and 4, then by
5 5 5
(****), a + b + c is also an integer.
k k k
It is now clear than we can continue this process inductively, since if a + b + c is an
n n n
integer for k equal to n − 3, n − 2 , and n − 1, then a + b + c will also be an integer, by
(****).
2003
In particular, we can conclude that a + b2003 + c 2003 and a2004 + b 2004 + c 2004 are both
2003
integers, say a + b2003 + c 2003 = M and a2004 + b 2004 + c 2004 = N .

Step 3: Final conclusion


2003
Since each of a and b is between 0 and 1, then a > a2004 and b 2003 > b2004 , so
a2003 + b2003 > a2004 + b 2004 .
Since a is less than 56 , then a is less than 0.9, so a < 0.81, so a4 < (0.81) < 0.7, so
2 2

a8 < (0.7) < 0.5 , so a16 < (0.5) < 0.25 .


2 2

16
Similarly, since b is less than 56 , then b < 0.25 .
16 16
Therefore, a + b < 0.5 , and since each of a and b is less than 1, then
a2004 + b 2004 < a2003 + b2003 < a16 + b16 < 0.5 .
Therefore, since c
2003
( )
= M − a2003 + b2003 and a2003 + b2003 < 0.5 , then the closest
2003 2003 2003
integer to c is M and the distance between them is a +b . Similarly, the closest
2004 2004 2004
integer to c is N and the distance between them is a +b .
2004 2004 2003 2003 2004 2003
But a +b <a +b , so c is closer to N than c is to M, as required.
The Canadian Mathematical Society
in collaboration with

The CENTRE for EDUCATION


in MATHEMATICS and COMPUTING
presents the

Canadian Open
Mathematics Challenge
Wednesday, November 24, 2004

Solutions

2004
c Canadian Mathematical Society
2004 COMC Solutions Page 2 of 19

Part A

1. If x + 2y = 84 = 2x + y, what is the value of x + y?

Solution 1
Since x + 2y = 84 and 2x + y = 84, then adding these two equations together, we obtain
3x + 3y = 168 or x + y = 56.

Solution 2
Since x + 2y = 84, then x = 84 − 2y.
Substituting into the second equation, we get

2(84 − 2y) + y = 84
168 − 3y = 84
84 = 3y
y = 28

Therefore, x = 84 − 2(28) = 28 and so x + y = 56.

Solution 3
Since 2x + y = 84, then y = 84 − 2x.
Substituting into the first equation, we get

x + 2(84 − 2x) = 84
168 − 3x = 84
84 = 3x
x = 28

Therefore, y = 84 − 2(28) = 28 and so x + y = 56.

Solution 4
SInce these two expressions are identical when x is replaced by y and y is replaced by x, then
x = y.
Therefore, 3x = 84 or x = 28 and so y = 28.
Thus, x + y = 56.

Answer: 56
2004 COMC Solutions Page 3 of 19

2. Let S be the set of all three-digit positive integers whose digits are 3, 5 and 7, with no digit
repeated in the same integer. Calculate the remainder when the sum of all of the integers in S
is divided by 9.

Solution 1
We can write down the elements of S: 357, 375, 537, 573, 735, 753.
The sum of these elements is 357 + 375 + 537 + 573 + 735 + 753 = 3330.
Since 3330 is divisible by 9 (because the sum of its digits is divisible by 9), the remainder when
we divide by 9 is 0.

Solution 2
There are six numbers formed with the three given numbers.
Two of these numbers have a 3 in the 100s position, two have a 5 in the 100s position, and two
have a 7 in the 100s position.
The same can be said about the distribution of numbers in the 10s and units positions.
Therefore, the sum of the six numbers is

2(3 + 5 + 7)(100) + 2(3 + 5 + 7)(10) + 2(3 + 5 + 7)(1) = 3330

The remainder is 0 when 3330 is divided by 9.

Answer: 0

3. In the diagram, point E has coordinates (0, 2), and B lies on y



the positive x-axis so that BE = 7. Also, point C lies on the
positive x-axis so that BC = OB. If point D lies in the first E

quadrant such that ∠CBD = 30◦ and ∠BCD = 90◦ , what is the
length of ED? D

x
O B C
Solution
In order to find the length of ED, we will try to find the coordinates of D. Let the coordinates
of B be (b, 0).

Since BE = 7 and the coordinates of E are (0, 2), then
p √
(b − 0)2 + (0 − 2)2 = 7
b2 + 4 = 7
b2 = 3
√ √
Since the point B lies on the positive x-axis, then b = 3 (not − 3), so B has coordinates

( 3, 0).
2004 COMC Solutions Page 4 of 19

(Note that it would have also been possible to find the coordinates of B by using Pythagoras
√ √
– OE 2 + OB 2 = EB 2 so OB 2 = 7 − 4 = 3 so OB = 3 and so B has coordinates ( 3, 0).)

Since BC = OB, then C has coordinates (2 3, 0).

Since ∠BCD = 90◦ and D lies in the first quadrant, then D has coordinates (2 3, d), with
d > 0.
Since 4DBC has ∠BCD = 90◦ and ∠CBD = 30◦ , then it is a 30◦ − 60◦ − 90◦ triangle. Since

CB = 3 (and CB is opposite the 60◦ angle), then DC (which is opposite the 30◦ angle) has
length 1.

Therefore, D has coordinates (2 3, 1).

E (0,2 )

(
D 2 3, 1 )
x
O
(
C 2 3, 0 )
B ( 3, 0 )
q √ √ √
Thus, ED = (2 3 − 0)2 + (1 − 2)2 = 12 + 1 = 13.

Answer: 13

4. A function f (x) has the following properties:

i) f (1) = 1
ii) f (2x) = 4f (x) + 6
iii) f (x + 2) = f (x) + 12x + 12

Calculate f (6).

Solution 1
Using property ii) with x = 1,

f (2) = 4f (1) + 6 = 4(1) + 6 = 10

since f (1) = 1 by property i).


Using property ii) with x = 2,

f (4) = 4f (2) + 6 = 4(10) + 6 = 46


2004 COMC Solutions Page 5 of 19

Using property iii) with x = 4,

f (6) = f (4) + 12(4) + 12 = 46 + 48 + 12 = 106

Therefore, the value of f (6) is 106.

Solution 2
Using property iii) with x = 1,

f (3) = f (1) + 12(1) + 12 = 1 + 12 + 12 = 25

since f (1) = 1 by property i).


Using property ii) with x = 3,

f (6) = 4f (3) + 6 = 4(25) + 6 = 106

Therefore, the value of f (6) is 106.

Solution 3
Working backwards,

f (6) = 4f (3) + 6 (by property ii) with x = 3)


= 4(f (1) + 12(1) + 12) + 6 (by property iii) with x = 1)
= 4f (1) + 4(24) + 6
= 4(1) + 102 (by property i))

Therefore, the value of f (6) is 106.

Answer: f (6) = 106

5. The Rice Tent Company sells tents in two different sizes, large and small. Last year, the Com-
pany sold 200 tents, of which one quarter were large. The sale of the large tents produced one
third of the company’s income. What was the ratio of the price of a large tent to the price of
a small tent?

Solution
Since the Rice Tent Company sold 200 tents, of which one quarter were large, then they sold
50 large tents and 150 small tents last year.
Let L be the price of a large tent and S the price of a small tent.
Then their income from large tents was 50L and from small tents was 150S.
Their total income last year was 50L + 150S.
2004 COMC Solutions Page 6 of 19

From the given information,


1
50L = (50L + 150S)
3
150L = 50L + 150S
100L = 150S
L 150 3
= =
S 100 2
Therefore, the ratio of the price of a large tent to the price of a small tent was 3 : 2.

Answer: 3 : 2

6. In the diagram, a square of side length 2 has semicircles drawn on


each side. An “elastic band” is stretched tightly around the figure.
What is the length of the elastic band in this position?

Solution
Label the four vertices of the square as W , X, Y , Z, in clockwise order.
Label the four midpoints of the sides of the square (that is, the centres of the four semicircles)
as M , N , O, P , in clockwise order, starting with M being the midpoint of W X.
In each semicircle, join the centre to the two points on that semicircle where the band just
starts (or stops) to contact the circle. Label these eight points as A, B, C, D, E, F , G, and H
in clockwise order, starting with A and B on the semicircle with centre M .

A B

H W M X C

P N

G Z O Y D

F E

By symmetry, the four straight parts of the band will be equal in length (that is, BC =
DE = F G = HA) and the four arc segments of the band will be equal in length (that is,
AB = CD = EF = GH).
Therefore, the total length of the band is 4(Length of arc AB) + 4(Length of BC).

Now, BC will actually be tangent to the two semicircles (with centres M and N ) where it
initially just touches them.
Thus, M B and N C are both perpendicular to BC.
2004 COMC Solutions Page 7 of 19

Since M B = N C = 1 (because they are radii of the semicircles and each semicircle has diame-
ter 2), then M BCN must actually be rectangle, so BC is equal and parallel to M N .
Since M and N are the midpoints of the sides of the square of side length 2, then M Y = Y N =
√ √
1, so M N = 2, so BC = 2.

Next, we determine the length of AB. Previously, we saw that M BCN is a rectangle, so
BC was parallel to M N . Similarly, HA is parallel to P M .
But P M is perpendicular to M N , so HA is perpendicular to BC.
Therefore, ∠AM B = 90◦ , ie. AB is one-quarter of the circumference of a circle with radius 1
or 41 (2π(1)) = 12 π.

√ √
Therefore, the total length of the band is 4( 21 π) + 4( 2) = 2π + 4 2.

Answer: 2π + 4 2

7. Let a and b be real numbers, with a > 1 and b > 0.


a
If ab = ab and = a3b , determine the value of a.
b

Solution 1
a
Since ab = ab and = a3b , then multiplying these two equations together, we get
b
a2 = ab · a3b = a4b .

Dividing both sides by a2 (since a 6= 0), we get a4b−2 = 1.


Since a > 1, then 4b − 2 = 0 or b = 12 .
√ √
Substituting back into the first equation, we get 21 a = a1/2 = a or a = 2 a.
Squaring both sides gives a2 = 4a or a2 − 4a = 0 or a(a − 4) = 0.
Since a > 1, then a = 4.

Solution 2
Since ab = ab , then, dividing both sides by a which is not equal to 0, we get b = ab−1 .
a
Since = a3b , then a = ba3b = ab−1 a3b = a4b−1 .
b
Comparing exponents, we get 1 = 4b − 1 or b = 21 .
Substituting b = 12 into ab = ab , we have 21 a = a1/2 or a1/2 = 2 or a = 4. So a = 4.

Solution 3
Since a > 1 and b > 0, we can take logarithms of both sides of both equations.

In the first equation, using log rules on log(ab) = log ab gives log(a) + log(b) = b log(a).
In the first equation, using log rules on log ab = log a3b gives log(a) − log(b) = 3b log(a).
 

Adding these two new equations gives 2 log(a) = 4b log(a) or (4b − 2) log(a) = 0.
Since a > 1, then log(a) > 0, so we must have 4b − 2 = 0 or b = 12 .
2004 COMC Solutions Page 8 of 19

Substituting this back into the first log equation gives log(a) + log 12 = 21 log(a) or

1
log(a) = − log 12 = log(2) or log(a) = 2 log(2) = log(4), so a = 4.

2

Answer: 4

8. A rectangular sheet of paper, ABCD, has AD = 1 and AB = r, where 1 < r < 2. The paper is
folded along a line through A so that the edge AD falls onto the edge AB. Without unfolding,
the paper is folded again along a line through B so that the edge CB also lies on AB. The
result is a triangular piece of paper. A region of this triangle is four sheets thick. In terms of
r, what is the area of this region?

Solution
Start with the rectangular sheet of paper, ABCD, with A in the top left and B in the bottom
left.
Fold AD across so that AD lies along AB. Let D0 be the point were D touches AB and let E
be the point on DC where the fold hits DC.
Since AD0 is the old AD, then AD0 = 1.
Since D0 E is perpendicular to D0 A (since DC was perpendicular to AD) then D0 E is parallel
to BC, so D0 E = 1 as well.

A D

1
D′ E
r −1
B C

We can also conclude that D0 B = EC = r − 1, since AB has length r.

Next, we fold the paper so that BC lies along AB. Unfold this paper and lay it flat so
that we can see the crease.
Since BC is folded onto AB, then the crease bisects ∠ABC, that is the crease makes an angle
of 45◦ with both AB and BC.
Suppose that the crease crosses D0 E at X and AE at Y .
2004 COMC Solutions Page 9 of 19

D′ E
X

B C

Now when the paper had been folded the second time (before we unfolded it!), the only way to
obtain a region four sheets thick was to fold a region two sheets thick on top of a region which
is also two sheets thick.
Since 4XY E is the only part of the paper “below” the second crease which is two sheets thick,
and, when the second fold is made, it lies entirely over another region which is two sheets thick,
then the desired area is the area of 4XY E.

Since ∠ABX = 45◦ , then 4BD0 X is isosceles and right-angled, so D0 X = D0 B = r − 1.


Thus, EX = 1 − D0 X = 1 − (r − 1) = 2 − r.
Since ∠D0 XB = 45◦ , then ∠Y XE = 45◦ . Also, since 4AD0 E is right isosceles, then ∠Y EX =
45◦ , so 4XY E is isosceles and right-angled.

s s

X 2−r E


Suppose XY = s. Then 2s = XE = 2 − r or 2s2 = (2 − r)2 .
The area of 4XY E is 21 s2 or 14 (2 − r)2 .
Therefore, the area of the desired region is 41 (2 − r)2 .

Answer: 14 (2 − r)2
2004 COMC Solutions Page 10 of 19

Part B

1. The points A(−8, 6) and B(−6, −8) lie on the circle x2 + y 2 = 100.

(a) Determine the equation of the line through A and B.

Solution
6 − (−8)
First, we determine the slope of the line segment AB. The slope is = −7.
−8 − (−6)
We could now proceed to find the equation of the line in several different ways.
Using the point-slope form, we obtain y − 6 = −7(x − (−8)) or y = −7x − 50.

(b) Determine the equation of the perpendicular bisector of AB.

Solution
Since the slope of AB is −7, then the slope of the perpendicular bisector of AB is 71 .
Also, the perpendicular bisector passes through the midpoint of AB, which is
 
1 1
((−8) + (−6)), (6 + (−8)) = (−7, −1).
2 2

Therefore, the equation of the perpendicular bisector is y − (−1) = 17 (x − (−7)) or y = 17 x.

(c) The perpendicular bisector of AB cuts the circle at two points, P in the first quadrant
and Q in the third quadrant. Determine the coordinates of P and Q.

Solution 1
y

A
P
x
Q

We want to find the points of intersection of the circle x2 + y 2 = 100 and the line y = 17 x.
From the equation of the line, x = 7y. Substituting this into the equation of the circle we
2004 COMC Solutions Page 11 of 19

obtain

(7y)2 + y 2 = 100
49y 2 + y 2 = 100
50y 2 = 100
y2 = 2

y = ± 2
√ √ √ √
Since x = 7y, then if y = 2, then x = 7 2, and if y = − 2, then x = −7 2.
√ √
Since P is in the first quadrant, then P has coordinates (7 2, 2).
√ √
Since Q is in the third quadrant, then Q has coordinates (−7 2, − 2).

Solution 2
We want to find the points of intersection of the circle x2 + y 2 = 100 and the line y = 17 x.
Substituting y = 71 x into the equation of the circle we obtain
 2
2 1
x + x = 100
7
1
x2 + x2 = 100
49
50 2
x = 100
49
x2 = 98
√ √
x = ± 98 = ±7 2
√ √ √ √
Since y = 17 x, then if x = 7 2, then y = 2, and if x = −7 2, then y = − 2.
√ √
Since P is in the first quadrant, then P has coordinates (7 2, 2).
√ √
Since Q is in the third quadrant, then Q has coordinates (−7 2, − 2).

(d) What is the length of P Q? Justify your answer.

Solution 1
The points P and Q are joined by the line y = 71 x, which passes through the origin.
Since the origin is the centre of the circle, then P Q must be a diameter of the circle.
Since the circle has equation x2 + y 2 = 100 = 102 , then its radius is 10, so its diameter is
20.
Therefore, P Q = 20.

Solution 2
√ √ √ √
Since we know that P (7 2, 2) and Q(−7 2, − 2), then we can determine the distance
2004 COMC Solutions Page 12 of 19

P Q by direct calculation:
r
√  √ 2 √  √ 2
PQ = 7 2 − −7 2 + 2− − 2
r
√ 2  √ 2
= 14 2 + 2 2
r
√ 2
= 2 [142 + 22 ]
p
= 2[196 + 4]

= 400
= 20

Therefore, the length of P Q is 20.

2. (a) Determine the two values of x such that x2 − 4x − 12 = 0.

Solution
Factoring the given equation x2 − 4x − 12 = 0, we obtain (x − 6)(x + 2) = 0.
Therefore, the two solutions are x = 6 and x = −2.


(b) Determine the one value of x such that x − 4x + 12 = 0. Justify your answer.

Solution
We first eliminate the square root by isolating it on one side and squaring:

x− 4x + 12 = 0

x = 4x + 12
x2 = 4x + 12
x2 − 4x − 12 = 0
(x − 6)(x + 2) = 0

Therefore, the two possible solutions are x = 6 and x = −2. (Since we have squared both
sides, it is possible that we have introduced an extraneous root, so we should verify both
of these.)
p √
If x = 6, then 6 − 4(6) + 12 = 6 − 36 = 0.
p √
If x = −2, then (−2) − 4(−2) + 12 = −2 − 4 = −4 6= 0.
Therefore, the one value of x that solves the equation is x = 6.
2004 COMC Solutions Page 13 of 19

(c) Determine all real values of c such that



x2 − 4x − c − 8x2 − 32x − 8c = 0

has precisely two distinct real solutions for x.

Solution
We start by attempting to solve this equation and then seeing what conditions on c arise.
Since 8x2 − 32x − 8c = 8(x2 − 4x − c), we let T = x2 − 4x − c.
Then the equation is

T − 8T = 0 (∗)

T = 8T
T 2 = 8T
T (T − 8) = 0

Therefore, T = 0 or T = 8. We can check that neither root is extraneous, so x2 − x − c = 0


or x2 − 4x − c = 8.
Let’s look at these last two equations.
First, we look at x2 − 4x − c = 0. The discriminant of this quadratic equation is (−4)2 −
4(−c) = 16 + 4c. Therefore, this equation has
• zero solutions if 16 + 4c < 0, so c < −4,
• exactly one solution if 16 + 4c = 0, so c = −4, and
• two distinct solutions if 16 + 4c > 0, so c > −4.
Next, we look at x2 − 4x − c = 8 or x2 − 4x − (c + 8) = 0. The discriminant of this
quadratic equation is (−4)2 − 4(−(c + 8)) = 48 + 4c. Therefore, this equation has
• zero solutions if 48 + 4c < 0, so c < −12,
• exactly one solution if 48 + 4c = 0, so c = −12, and
• two distinct solutions if 48 + 4c > 0, so c > −12.
We see also that any value of x that satisfies one of these two equations cannot satisfy the
other, since we cannot have both x2 − 4x − c = 0 and x2 − 4x − c = 8. (In other words,
no roots overlap between these two equations.)
We make a table to combine our observations:
c < −12 c = −12 −12 < c < −4 c = −4 c > −4
2
x − 4x − c = 0 0 solutions 0 solutions 0 solutions 1 solution 2 solutions
x2 − 4x − c = 8 0 solutions 1 solution 2 solutions 2 solutions 2 solutions
Total solutions 0 solutions 1 solution 2 solutions 3 solutions 4 solutions
Therefore, for there to be exactly two distinct solutions, we must have −12 < c < −4.
2004 COMC Solutions Page 14 of 19

3. A map shows all Beryl’s Llamaburgers restaurant locations in North America. On this map,
a line segment is drawn from each restaurant to the restaurant that is closest to it. Every
restaurant has a unique closest neighbour. (Note that if A and B are two of the restaurants,
then A may be the closest to B without B being closest to A.)

(a) Prove that no three line segments on the map can form a triangle.

Solution 1
We start by assuming that three line segments on the map do form a triangle, and show
that this is in fact impossible.
Notice that if restaurants X and Y are joined by a line segment, then either X is the
closest restaurant to Y or Y is the closest restaurant to X (or both).

Assume that A, B and C are the three points on the map connect by segments.
B

A C

To begin, we focus on the segment joining A to B. Let’s assume that A is the closest
restaurant to B. (It doesn’t matter which direction we assume here.) This means that C
is not the closest restaurant to B, so BA < BC.
But B and C are connected and C is not the closest restaurant to B. Therefore, B is the
closest restaurant to C, which means CB < CA.
But C and A are also connected and A is not the closest restaurant to C. Therefore, C is
the closest restaurant to A, which means AC < AB.

But this means that BA < BC, BC < AC and AC < BA. This cannot be the case.
Therefore, it is impossible for three line segments to form a triangle.

Solution 2
We prove this by showing that constructing a triangle is impossible.
We start by considering two locations A and B and the line segment AB.
Since A and B are connected, we can assume without loss of generality that A is closest
to B. (The case B closest to A involves interchanging A and B, and the case of A and B
closest to each other is included in the case of A closest to B.)

If A is closest to B and we add a new location C which is connected to B, then B


must be closest to C (since C can’t also be closest to B along with A).
2004 COMC Solutions Page 15 of 19

If we join C to A, then either C is closest to A or A is closest to C.


But A can’t be closest to C since B is closest to C.
Therefore, we must have C closest to A.
But then AC is shorter than AB, along with AB being shorter than BC (since A is closest
to B), which means that AC is shorter than BC or A is closer to C than B is, which isn’t
true. This is a contradiction.

Therefore, we can’t construct a triangle.

(b) Prove that no restaurant can be connected to more than five other restaurants.

Solution
We start by assuming that one restaurant can be connected to six others and show that
this is impossible. From this we can conclude that no restaurant can be connected to
more than five other restaurants (for if it could be joined to 8 others, say, then we could
consider six of them only and reach a contradiction).

Assume that restaurant A can be connected to restaurants B, C, D, E, F , and H, where


these restaurants are listed in clockwise order of their line segments joining to A.

H B

A C
F

D
E

Consider restaurants A, B and C.


We know that B and C are both connected to A and both cannot be the closest neighbour
to A. Thus, A must be the closest neighbour to one of these, say B. (It doesn’t matter
which we choose).
Since A is the closest restaurant to B, then BA < BC.
Now consider the line joining C to A.
If C is the closest neighbour to A, then AC < AB, so AC < AB < BC.
If A is the closest neighbour to C, then CA < CB so CA < CB and BA < CB.
In either case, BC is the (strictly) longest side in 4ABC, and so must be opposite the
(strictly) largest angle.
Since the angles in a triangle add to 180◦ , then if there is a largest angle, then this angle
must be larger than 60◦ . Therefore, from the above reasoning, ∠BAC > 60◦ .
2004 COMC Solutions Page 16 of 19

But we can reapply this reasoning to conclude that ∠CAD, ∠DAE, ∠EAF , ∠F AH,
and ∠HAB are each greater than 60◦ . But the sum of these six angles is 360◦ , since they
will form a full circle around A, and six angles, each greater than 60◦ , cannot add to 360◦ .
So we have a contradiction.

Therefore, it is impossible for a restaurant to be connected to more than five other restau-
rants.

4. In a sumac sequence, t1 , t2 , t3 , . . ., tm , each term is an integer greater than or equal to 0.


Also, each term, starting with the third, is the difference of the preceding two terms (that is,
tn+2 = tn − tn+1 for n ≥ 1). The sequence terminates at tm if tm−1 − tm < 0. For example, 120,
71, 49, 22, 27 is a sumac sequence of length 5.

(a) Find the positive integer B so that the sumac sequence 150, B, . . . has the maximum
possible number of terms.

Solution
Suppose that we have a sumac sequence with t1 = 150 and t2 = B. Let’s write out the
next several terms (assuming that they exist) in terms of B:

t3 = 150 − B t4 = 2B − 150 t5 = 300 − 3B t6 = 5B − 450


t7 = 750 − 8B t8 = 13B − 1200 t9 = 1950 − 21B t10 = 34B − 3150
In order to maximize the length of this sumac sequence, we would like to choose B so that
as many terms as possible starting from t1 are non-negative. (When we reach the first
negative “term”, we know that the sequence terminated at the previous term.)
For t2 ≥ 0, B ≥ 0.
For t3 ≥ 0, 150 − B ≥ 0 or B ≤ 150.
For t4 ≥ 0, 2B − 150 ≥ 0 or B ≥ 75.
For t5 ≥ 0, 300 − 3B ≥ 0 or B ≤ 100.
For t6 ≥ 0, 5B − 450 ≥ 0 or B ≥ 90.
For t7 ≥ 0, 750 − 8B ≥ 0 or B ≤ 750
8
= 93 68 .
For t8 ≥ 0, 13B − 1200 ≥ 0 or B ≥ 1200
13
= 92 134
.
1950 18
For t9 ≥ 0, 1950 − 21B ≥ 0 or B ≤ 21 = 92 21 .
Therefore, since B must be a positive integer, if we choose B = 93, then we can ensure
that each of t1 through t8 are non-negative. This will maximize the number of terms
4
starting from the beginning, since B must satisfy 92 13 ≤ B ≤ 93 86 in order for at least the
first eight terms to be non-negative. (Note that t9 will in fact be negative when B = 93.)

When we set B = 93, we obtain the sumac sequence 150, 93, 57, 36, 21, 15, 6, 9.
2004 COMC Solutions Page 17 of 19

(b) Let m be a positive integer with m ≥ 5. Determine the number of sumac sequences of
length m with tm ≤ 2000 and with no term divisible by 5.

Solution
We begin our solution by making some observations about sumac sequences.
• A sumac sequence is completely determined by its first two terms. This is true since
the first two terms give us the third, the second and third give us the fourth, and so
on. The sequence will terminate when the “next term” would be negative.
• In a sumac sequence, since for every (valid) n we have tn+2 = tn − tn+1 , then tn =
tn+1 + tn+2 . This means that we can “reverse engineer” a sumac sequence – if we know
terms (n + 1) and (n + 2), then we can determine term n. Thus, if we know the final
two terms in a sumac sequence, then we can determine all of the previous terms.
• From the first observation, the first two terms of a sumac sequence completely de-
termines the sequence. Is the same true of the last two terms? No. When we start
looking at a sumac sequence from the back, every new term as we proceed towards
the front will always be non-negative (since we are adding non-negative terms). Thus,
there is no “stopping condition” as there is when we work forwards. (For example, 3,
1, 2 is a sumac sequence ending with 1, 2, as is 4, 3, 1, 2.)
• However, if we know the final two terms and the length of the sequence, this completely
determines the sumac sequence (and we will always be able to find such a sequence).
Now we proceed. Let m be a fixed positive integer with m ≥ 5.
Suppose that t1 , t2 , . . . , tm−1 , tm is a sumac sequence of length m.
Because we are given a condition on the final term of the sequence, we will examine the
sequence from the back.
Let x = tm and y = tm−1 . Note that x, y and m determine the sequence.
Since x and y are the last two terms in the sumac sequence, then tm−1 − tm = y − x < 0
or x > y.
Since we have m fixed, we would like to determine how many sumac sequences we can
form with tm = x ≤ 2000, tm−1 = y < x and no term divisible by 5.

Let’s write out the last five terms of the sequence (in reverse order): x, y, x + y, x + 2y,
2x + 3y. (Since m ≥ 5, we know that there are at least five terms in the sequence.)
Since we want no term divisible by 5, let us consider x and y modulo 5 to see what hap-
pens. (There are 25 possible pairs for (x, y) modulo 5.)
Since no term is divisible by 5, then we don’t want x ≡ 0 (mod 5) or y ≡ 0 (mod 5). This
cuts us down to 16 possibilities for (x, y).
We make a table of these possibilities to determine which pairs can be eliminated simply
by looking at the last five terms. (All entries in the table are modulo 5. In any given row,
we stop if we reach a 0, since this possibility can then be discarded.)
2004 COMC Solutions Page 18 of 19

x y x+y x + 2y 2x + 3y
1 1 2 3 0
1 2 3 0
1 3 4 2 1
1 4 0
2 1 3 4 2
2 2 4 1 0
2 3 0
2 4 1 0
3 1 4 0
3 2 0
3 3 1 4 0
3 4 2 1 3
4 1 0
4 2 1 3 4
4 3 2 0
4 4 3 2 0
So the only possible pairs for (x, y) modulo 5 are (1, 3), (2, 1), (3, 4) and (4, 2).
If we start with (x, y) = (1, 3) modulo 5, then the terms in the sequence modulo 5 are 1,
3, 4, 2, 1, 3, 4, 2, 1, . . ., ie. the terms cycle modulo 5 with no terms divisible by 5.
This similar cycling will happen with each of the other 3 pairs, so each of these 4 pairs
give no terms divisible by 5.

So for each of these pairs, we need to determine the number of pairs of non-negative
integers (x, y) with x ≤ 2000, y < x and congruent to the appropriate things modulo
5. Each such pair will give a sumac sequence of length m ≥ 5 with no term divisible by
5. (Since the divisibility of the terms is independent of length, this also means that the
number of such sequences will be independent of m!)

Case 1: (x, y) congruent to (1, 3) modulo 5


Here x can take the values 1996, 1991, . . ., 6, 1.
If x = 1996, y can be 1993, 1988, . . ., 8, 3. (399 possibilities)
If x = 1991, y can be 1988, 1983, . . ., 8, 3. (398 possibilities)
This pattern continues, with one fewer possibility each time x decreases by 5, until we
reach x = 6, where y = 3 is the only possibility.
Thus, there are 399 + 398 + · · · + 2 + 1 possibilities for this case.
2004 COMC Solutions Page 19 of 19

Case 2: (x, y) congruent to (2, 1) modulo 5


Here x can take the values 1997, 1992, . . ., 7, 2.
If x = 1997, y can be 1996, 1991, . . ., 6, 1. (400 possibilities)
If x = 1992, y can be 1991, 1986, . . ., 6, 1. (399 possibilities)
This pattern continues, with one fewer possibility each time x decreases by 5, until we
reach x = 2, where y = 1 is the only possibility.
Thus, there are 400 + 399 + · · · + 2 + 1 possibilities for this case.

Case 3: (x, y) congruent to (3, 4) modulo 5


Here x can take the values 1998, 1993, . . ., 8, 3.
If x = 1998, y can be 1994, 1989, . . ., 9, 4. (399 possibilities)
If x = 1993, y can be 1989, 1984, . . ., 9, 4. (398 possibilities)
This pattern continues, with one fewer possibility each time x decreases by 5, until we
reach x = 8, where y = 4 is the only possibility.
Thus, there are 399 + 398 + · · · + 2 + 1 possibilities for this case.

Case 4: (x, y) congruent to (4, 2) modulo 5


Here x can take the values 1999, 1994, . . ., 9, 4.
If x = 1999, y can be 1997, 1992, . . ., 7, 2. (400 possibilities)
If x = 1994, y can be 1992, 1987, . . ., 7, 2. (399 possibilities)
This pattern continues, with one fewer possibility each time x decreases by 5, until we
reach x = 4, where y = 2 is the only possibility.
Thus, there are 400 + 399 + · · · + 2 + 1 possibilities for this case.

Therefore, overall there are

2(399+398+· · ·+2+1)+2(400+399+· · ·+2+1) = 399(400)+400(401) = 400(800) = 320 000

possibilities. Therefore, there are exactly 320 000 sumac sequences of length m with no
term divisible by 5 and with tm ≤ 2000.
The Canadian Mathematical Society
in collaboration with

The CENTRE for EDUCATION


in MATHEMATICS and COMPUTING

presents the

Canadian Open
Mathematics Challenge
Wednesday, November 23, 2005

Solutions

2005
c Canadian Mathematical Society
2005 COMC Solutions Page 2 of 26

Part A

1. Determine the value of 102 − 92 + 82 − 72 + 62 − 52 + 42 − 32 + 22 − 12 .

Solution 1
Using differences of squares,

102 − 92 + 82 − 72 + 62 − 52 + 42 − 32 + 22 − 12
= (10 − 9)(10 + 9) + (8 − 7)(8 + 7) + (6 − 5)(6 + 5) + (4 − 3)(4 + 3) + (2 − 1)(2 + 1)
= 1(10 + 9) + 1(8 + 7) + 1(6 + 5) + 1(4 + 3) + 1(2 + 1)
= 10 + 9 + 8 + 7 + 6 + 5 + 4 + 3 + 2 + 1
= 55

(We can get the answer 55 either by computing the sum directly, or by using the fact that
1 + 2 + 3 + 4 + 5 + 6 + 7 + 8 + 9 + 10 = 21 (10)(11) = 55.)

Solution 2
Computing directly,

102 − 92 + 82 − 72 + 62 − 52 + 42 − 32 + 22 − 12
= 100 − 81 + 64 − 49 + 36 − 25 + 16 − 9 + 4 − 1
= 19 + 15 + 11 + 7 + 3 (computing difference of each pair)
= 55

Answer: 55

2. A bug in the xy-plane starts at the point (1, 9). It moves first to the point (2, 10) and then to
the point (3, 11), and so on. It continues to move in this way until it reaches a point whose
y-coordinate is twice its x-coordinate. What are the coordinates of this point?

Solution 1
The bug starts at (1, 9) and each time moves 1 unit to the right and 1 unit up.
Thus, after k moves, the bug will be at the point (1 + k, 9 + k).
When its y-coordinate is twice its x-coordinate, we have 9 + k = 2(1 + k) or 9 + k = 2 + 2k
or k = 7.
When k = 7, the bug is at point (1 + 7, 9 + 7) = (8, 16), and the bug stops here.

Solution 2
The bug starts at (1, 9) and each time moves 1 unit to the right and 1 unit up.
Thus, at any point to which the bug moves, the y-coordinate will be 8 more than
2005 COMC Solutions Page 3 of 26

the x-coordinate, so every such point is of the form (n, n + 8).


For the y-coordinate to be twice the x-coordinate, n + 8 = 2n or n = 8.
When n = 8, the bug is at the point (8, 16), and the bug stops here.

Solution 3
We write out the sequence of points to which the bug moves and stop when we get to a point
where the y-coordinate is twice the x-coordinate:

(1, 9), (2, 10), (3, 11), (4, 12), (5, 13), (6, 14), (7, 15), (8, 16)

Thus, the bug stops at (8, 16).

Answer: (8, 16)

3. If ax3 + bx2 + cx + d = (x2 + x − 2)(x − 4) − (x + 2)(x2 − 5x + 4) for all values of x, what is


the value of a + b + c + d?

Solution 1
We use the fact that a + b + c + d = a(13 ) + b(12 ) + c(1) + d, so a + b + c + d must be equal to
the right side of the given equation with x set equal to 1.
Thus,

a + b + c + d = (12 + 1 − 2)(1 − 4) − (1 + 2)(12 − 5 + 4) = 0(−3) − 3(0) = 0

Solution 2
We simplify the right side of the given equation by factoring the two quadratic polynomials:

(x2 + x − 2)(x − 4) − (x + 2)(x2 − 5x + 4) = (x − 1)(x + 2)(x − 4) − (x + 2)(x − 1)(x − 4)


= 0

Therefore, ax3 + bx2 + cx + d = 0 for all values of x. (In other words, ax3 + bx2 + cx + d = 0 is
the zero polynomial, so all of its coefficients are equal to 0.)
Therefore, a = b = c = d = 0, so a + b + c + d = 0.

Solution 3
We expand and simplify the right side:

ax3 + bx2 + cx + d = (x2 + x − 2)(x − 4) − (x + 2)(x2 − 5x + 4)


= x3 + x2 − 2x − 4x2 − 4x + 8 − (x3 + 2x2 − 5x2 − 10x + 4x + 8)
= x3 − 3x2 − 6x + 8 − (x3 − 3x2 − 6x + 8)
= 0

Therefore, each of the coefficients of ax3 + bx2 + cx + d are 0, so a = b = c = d = 0 and


2005 COMC Solutions Page 4 of 26

thus a + b + c + d = 0.

Answer: 0

p
4. A fractionis in lowest terms if p and q have no common factor larger than 1.
q
1 2 70 71
How many of the 71 fractions , ,..., , are in lowest terms?
72 72 72 72

Solution 1
First, we note that 72 = 23 × 32 .
a
For one of the fractions to be in lowest terms, then a and 72 have no common factors.
72
In other words, a cannot be divisible by 2 or 3 (since 2 and 3 are the only prime numbers which
are divisors of 72).
How many of the positive integers from 1 to 71 are not divisible by 2 or 3?
Of these integers, 35 of the them are divisible by 2 (namely, 2, 4, 6, . . ., 70).
Also, 23 of them (namely, 3, 6, . . ., 69) are divisible by 3.
But some numbers are counted twice in these lists: all of the multiples of both 2 and 3 (ie. the
multiples of 6). These are 6, 12, . . ., 66 – that is, 11 numbers in total.
So the number of positive integers from 1 to 71 which are divisible by 2 or 3 is 35 + 23 − 11 = 47
(11 is subtracted to remove the double-counted numbers).
So the number of positive integers from 1 to 71 which are not divisible by 2 or 3 is 71 − 47 = 24.
1 2 70 71
Therefore, 24 of the 71 fractions , ,··· , , are irreducible.
72 72 72 72

Solution 2
First, we note that 72 = 23 × 32 .
a
For one of the fractions to be in lowest terms, then a and 72 have no common factors.
72
Since the only primes which are divisors of 72 are 2 and 3, then a and 72 have no common
factors when a is not divisible by 2 or 3.
Look at the first few fractions in the list:
1 2 3 4 5 6 7 8 9 10 11 12
, , , , , , , , , , ,
72 72 72 72 72 72 72 72 72 72 72 72
From this list, the ones which are in lowest terms are
1 5 7 11
, , ,
72 72 72 72
So the 1st and 5th of each of the two sets of 6 fractions above are in lowest terms.
72
This pattern will continue, so if we include the fraction (which we know is not in lowest
72
terms) at the end of the list, we obtain 12 sets of 6 fractions, and 2 fractions out of each set
will be in lowest terms, giving 12 × 2 = 24 fractions in lowest terms.
2005 COMC Solutions Page 5 of 26

(Why does this pattern continue? Each of the fractions in the list can be written in one
of the following forms:
6k + 1 6k + 2 6k + 3 6k + 4 6k + 5 6k + 6
, , , , ,
72 72 72 72 72 72
Since the numerators 6k + 2, 6k + 4 and 6k + 6 are divisible by 2 and the numerator 6k + 3 is
divisible by 3, then none of the fractions with these as numerators is in lowest terms.
Also, 6k + 1 and 6k + 5 are never divisible by 2 or 3, so these corresponding fractions are always
in lowest terms.
Thus, 2 out of each set of 6 fractions is in lowest terms.)

Answer: 24

5. An office building has 50 storeys, 25 of which are painted black and the other 25 of which are
painted gold. If the number of gold storeys in the top half of the building is added to the
number of black storeys in the bottom half of the building, the sum is 28. How many gold
storeys are there in the top half of the building?

Solution 1
Let G be the number of gold storeys in the top half of the building.
Then there are 25 − G black storeys in the top half of the building.
Since there are 25 black storeys in total, then the number of black storeys in the bottom half
of the building is 25 − (25 − G) = G.
Since the sum of the number of gold storeys in the top half of the building and the number of
black storeys in the bottom half of the building is 28, then G + G = 28, or G = 14.
Thus, there are 14 gold storeys in the top half of the building.

Solution 2
Let G and g be the number of gold storeys in the top and bottom halfs of the building, and
B and b the number of black storeys in the top and bottom halfs of the building.
Then G + B = 25 and g + b = 25, looking at the top and bottom halfs of the building.
Also, G + g = 25 and B + b = 25, since 25 of the storeys are painted in each colour.
Also, G + b = 28 from the given information, or b = 28 − G.
Since B + b = 25, then B + 28 − G = 25, so B = G − 3.
Since G + B = 25, then G + G − 3 = 25 or 2G = 28 or G = 14.
Thus, there are 14 gold storeys in the top half of the building.

Answer: 14
2005 COMC Solutions Page 6 of 26

6. In the grid shown, each row has a value as-


signed to it and each column has a value as- 3 0 5 6 −2
signed to it. The number in each cell is the −2 −5 0 1 y
sum of its row and column values. For exam- 5 2 x 8 0
ple, the “8” is the sum of the value assigned to 0 −3 2 3 −5
the 3rd row and the value assigned to the 4th −4 −7 −2 −1 −9
column. Determine the values of x and y.
Solution 1
First, we label the values assigned to the five columns A, B, C, D, E and the values assigned to
the five rows a, b, c, d, e.
0 1
Look at the sub-grid .
x 8
Since the 0 is in row 2 and column 3, then 0 = b + C.
Similarly, 1 = b + D, 8 = c + D and x = c + C.
But then 0 + 8 = (b + C) + (c + D) = (c + C) + (b + D) = x + 1 or x = 7.
1 y
In a similar way, we can show by looking at the sub-grid that we must have
8 0
1 + 0 = y + 8 or y = −7.
Thus, x = 7 and y = −7.
p q
(In fact, in any sub-grid of the form , we must have p + s = q + r.)
r s

Solution 2
First, we label the values assigned to the five columns A, B, C, D, E and the values assigned to
the five rows a, b, c, d, e.
Suppose that we try A = 0.
Looking at the “3” in the first row and first column, A + a = 3, so a = 3.
Since a = 3 and the entry in the first row and second column is 0, then a + B = 0, or B = −3.
Similarly, C = 2, D = 3 and E = −5.
Since A = 0 and the entry in the second row and first column is −2, then b + A = 0, then
b = −2.
Since y = b + E, then y = −2 + (−5) = −7.
Since A = 0 and the entry in the third row and first column is 5, then c + A = 5, so c = 5.
Since x = c + C, then x = 5 + 2 = 7.
Thus, x = 7 and y = −7.

Solution 3
First, we label the values assigned to the five columns A, B, C, D, E and the values assigned to
the five rows a, b, c, d, e.
If we choose five entries from the table which include one from each row and one from each
2005 COMC Solutions Page 7 of 26

column, then the sum of these entries is constant no matter how we choose the entries, as it is
always equal to A + B + C + D + E + a + b + c + d + e.
Here are three ways in which this can be done (looking at the bolded numbers):

3 0 5 6 −2 3 0 5 6 −2 3 0 5 6 −2
−2 −5 0 1 y −2 −5 0 1 y −2 −5 0 1 y
5 2 x 8 0 5 2 x 8 0 5 2 x 8 0
0 −3 2 3 −5 0 −3 2 3 −5 0 −3 2 3 −5
−4 −7 −2 −1 −9 −4 −7 −2 −1 −9 −4 −7 −2 −1 −9

Therefore, 3 + (−5) + 2 + 8 + (−9) = (−4) + (−3) + x + 1 + (−2) = 3 + y + 2 + (−2) + 3 or


−1 = x − 8 = y + 6.
Thus, x = 7 and y = −7.

Solution 4
First, we label the values assigned to the five columns A, B, C, D, E and the values assigned to
the five rows a, b, c, d, e.
Consider the first two entries in row 1.
We have 3 = A + a and 0 = B + a.
Subtracting these, we obtain 3 = 3 − 0 = (A + a) − (B + a) = A − B.
Notice that whenever we take entries in columns 1 and 2 from the same row, their difference
will always equal A − B, which is equal to 3.
Similarly, since the difference between the 0 and the 5 in the first row is 5, then every entry in
column 3 will be 5 greater than the entry in column 2 from the same row.
Thus, x = 2 + 5 = 7.
Also, since the difference between the 6 and the −2 in the first row is 8, then every entry in
column 5 will be 8 less than the entry in column 4 from the same row.
Thus, y = 1 − 8 = −7.
Therefore, x = 7 and y = −7.

Answer: x = 7 and y = −7
2005 COMC Solutions Page 8 of 26

7. In the diagram, the semi-circle has centre O and diameter AB. A ray of light leaves point P
in a direction perpendicular to AB. It bounces off the semi-circle at point D in such a way
that ∠P DO = ∠EDO. (In other words, the angle of incidence equals the angle of reflection
at D.) The ray DE then bounces off the circle in a similar way at E before finally hitting the
semicircle again at B. Determine ∠DOP .

A B
P O

Solution 1
Join D and E to O, and let ∠DOP = x.
Since DP ⊥ AB, then ∠P DO = 90◦ − x.
Since the angle of incidence equals the angle of reflection at D, then ∠EDO = ∠P DO = 90◦ −x.

E
90 x
90 x

D 90 x

90 x

x 90 x
A B
P O

Since DO and EO are both radii, then DO = EO, so 4EDO is isosceles, and so
∠DEO = ∠EDO = 90◦ − x.
Since the angle of incidence equals the angle of reflection at E, then ∠DEO = ∠BEO = 90◦ −x.
Since EO and BO are both radii, then EO = BO, so 4BEO is isosceles, and so
∠EBO = ∠BEO = 90◦ − x.
Consider quadrilateral P DEB.
We have ∠DP B = 90◦ , ∠P DE = (90◦ − x) + (90◦ − x) = 180◦ − 2x,
∠DEB = (90◦ − x) + (90◦ − x) = 180◦ − 2x, and ∠EBP = 90◦ − x.
Since the sum of the angles in the quadrilateral is 360◦ , then
90◦ + 180◦ − 2x + 180◦ − 2x + 90◦ − x = 360◦ or 540◦ − 5x = 360◦ or 5x = 180◦ or x = 36◦ .
Therefore, ∠DOP = x = 36◦ .

Solution 2
Join D and E to O, and let ∠DOP = x.
Since DP ⊥ AB, then ∠P DO = 90◦ − x.
Since the angle of incidence equals the angle of reflection at D, then ∠EDO = ∠P DO = 90◦ −x.
2005 COMC Solutions Page 9 of 26

E
90 x
90 x

D 90 x

90 x
2x
x 2x 90 x
A B
P O

Since DO and EO are both radii, then DO = EO, so 4EDO is isosceles, and so
∠DEO = ∠EDO = 90◦ − x. Also, ∠DOE = 180◦ − 2(90◦ − x) = 2x.
Since the angle of incidence equals the angle of reflection at E, then ∠DEO = ∠BEO = 90◦ −x.
Since EO and BO are both radii, then EO = BO, so 4BEO is isosceles, and so
∠EBO = ∠BEO = 90◦ − x. Also, ∠EOB = 180◦ − 2(90◦ − x) = 2x.
Since P OB is a straight line, then ∠P OD + ∠DOE + ∠EOB = 180◦ or x + 2x + 2x = 180◦ or
5x = 180◦ or x = 36◦ .
Therefore, ∠DOP = x = 36◦ .

Solution 3
Reflect the diagram across AB to complete the circle and form the pentagon DEBE 0 D0 . (Note
that DP D0 is a straight line since ∠DP O = ∠D0 P O = 90◦ .)

A O
B

Since DO, EO, BO, E 0 O and D0 O are all radii, then DO = EO = BO = E 0 O = D0 O.


Let ∠DOP = x. Since DP ⊥ AB, then ∠P DO = 90◦ − x.
By reflection, ∠D0 OP = ∠DOP = x, so ∠DOD0 = 2x.
Since the angle of incidence equals the angle of reflection at D, then ∠EDO = ∠P DO = 90◦ −x.
Since DO and EO are both radii, then DO = EO, so 4EDO is isosceles, and so
∠DEO = ∠EDO = 90◦ − x. Also, ∠DOE = 180◦ − 2(90◦ − x) = 2x.
Since the angle of incidence equals the angle of reflection at E, then ∠DEO = ∠BEO = 90◦ −x.
Since EO and BO are both radii, then EO = BO, so 4BEO is isosceles, and so
∠EBO = ∠BEO = 90◦ − x. Also, ∠EOB = 180◦ − 2(90◦ − x) = 2x.
2005 COMC Solutions Page 10 of 26

Therefore, the triangles DOE, EOB, BOE 0 , E 0 OD0 and D0 OD are all congruent by side-angle-
side. Therefore, pentagon DEBE 0 D0 is a regular pentagon.
Thus, ∠DOD0 = 51 (360◦ ) = 72◦ since the central angles of each of the five sides of the pentagon
are equal.
Since 4DOD0 is isosceles and OP is perpendicular to DD0 , then ∠P OD = 12 ∠DOD0 = 36◦ .
Thus, ∠P OD = 36◦ .

Answer: ∠DOP = 36◦

8. The number 18 is not the sum of any 2 consecutive positive integers, but is the sum of consec-
utive positive integers in at least 2 different ways, since 5 + 6 + 7 = 18 and 3 + 4 + 5 + 6 = 18.
Determine a positive integer less than 400 that is not the sum of any 11 consecutive positive
integers, but is the sum of consecutive positive integers in at least 11 different ways.

Solution
Suppose that the positive integer N is the sum of an odd number of consecutive integers, say
2k + 1 consecutive integers. Then for some integer a,

N = (a − k) + (a − (k − 1)) + · · · + (a − 1) + a + (a + 1) + · · · + (a + k) = (2k + 1)a

Thus, 2k + 1 is a divisor of N (ie. the number of integers in the representation is a divisor of N ).


Next, suppose that N is the sum of an even number of consecutive integers, say 2k consecutive
integers. Then for some integer b,
1
N = (b−k)+(b−(k−1))+· · ·+(b−1)+b+(b+1)+· · · (b+(k−1)) = 2kb−k = k(2b−1) = (2k)(2b−1)
2
Thus, k is a divisor of N and 2k is not a divisor of N (since 2b−1 is odd and so has no factor of 2).

We would like to find a positive integer N which is not the sum of 11 consecutive positive
integers (and so is not a multiple of 11) but is the sum of consecutive positive integers in 11
different ways.

Let’s consider the number of integers in each of the ways in which we write N as the sum
of consecutive integers. Note that if N is the sum of m consecutive positive integers, then N
is at least 1 + 2 + · · · + m. We make a table of what properties N must have for N to be the
sum of m consecutive integers for m = 2 to m = 10:
2005 COMC Solutions Page 11 of 26

m N at least Property of N
2 3 Divisible by 1, not by 2
3 6 Divisible by 3
4 10 Divisible by 2, not by 4
5 15 Divisible by 5
6 21 Divisible by 3, not by 6
(ie. divisible by 3, not by 2)
7 28 Divisible by 7
8 36 Divisible by 4, not by 8
9 45 Divisible by 9
10 55 Divisible by 5, not by 10
(ie. divisible by 5, not by 2)

How can we combine as many of these as possible? If we make N at least 55 and divisible
by 5, 7 and 9 and not divisible by 2, then N will be the sum of 2, 3, 5, 6, 7, 9 and 10 con-
secutive positive integers (7 representations in total). In this case, N must be divisible by
5 × 7 × 9 = 315. So following this line of thought, if N is less than 400, then we must have
N = 315. Now, 315 is also

• the sum of 15 consecutive positive integers since 315 is divisible by 15 and is at least 120
(ie. 1 + 2 + 3 + · · · + 15),
• the sum of 14 consecutive positive integers since 315 is divisible by 7, not by 14, and is at
least 105 (ie. 1 + 2 + 3 + · · · + 14)
• the sum of 18 consecutive positive integers since 315 is divisible by 9, not by 18, and is at
least 171 (ie. 1 + 2 + 3 + · · · + 18)
• the sum of 21 consecutive positive integers since 315 is divisible by 21 and is at least 231
(ie. 1 + 2 + 3 + · · · + 21)

So 315 is the sum of consecutive positive integers in at least 11 ways, and is not the sum
of 11 consecutive positive integers. (In fact, 315 is the unique answer, but we are not asked to
justify this.)

(Note: A good way to write a solution to this problem would be to first figure out in rough
that 315 was the answer, and then begin the solution by claiming that 315 is the answer. We
could then demonstrate that 315 works by showing that it can be represented in the correct
number of ways. While this approach is perfectly correct, it would not give much of a clue as
to how the answer was obtained.)

Answer: 315
2005 COMC Solutions Page 12 of 26

Part B

1. A line with slope −3 intersects the positive x-axis at A and the positive y-axis at B. A second
line intersects the x-axis at C(7, 0) and the y-axis at D. The lines intersect at E(3, 4).

D
E (3, 4)

x
O A C (7, 0)

(a) Find the slope of the line through C and E.

Solution
Since C has coordinates (7, 0) and E has coordinates (3, 4), then the slope of the line
through C and E is
0−4 −4
= = −1
7−3 4

(b) Find the equation of the line through C and E, and the coordinates of the point D.

Solution 1
Since the line through C and E has slope −1 and passes through the point (7, 0), then
the line has equation y − 0 = (−1)(x − 7) or y = −x + 7.
From the equation of the line, y = 7 is the y-intercept of the line.
Since D is the point where this line crosses the y-axis, then D has coordinates (0, 7).

Solution 2
Since the line through C and E has slope −1 and passes through the point (3, 4), then
the line has equation y − 4 = (−1)(x − 3) or y = −x + 7.
From the equation of the line, y = 7 is the y-intercept of the line.
Since D is the point where this line crosses the y-axis, then D has coordinates (0, 7).

(c) Find the equation of the line through A and B, and the coordinates of the point B.

Solution
Since the line through A and B has slope −3 and passes through the point E(3, 4), then
2005 COMC Solutions Page 13 of 26

the line has equation y − 4 = (−3)(x − 3) or y = −3x + 13.


From the equation of the line, y = 13 is the y-intercept of the line.
Since B is the point where this line crosses the y-axis, then B has coordinates (0, 13).

(d) Determine the area of the shaded region.

Solution 1
The area of the shaded region is the sum of the areas of 4DOC and 4BDE.
4DOC is right-angled at O, so the area of 4DOC is 21 (DO)(OC) = 12 (7)(7) = 49
2
.
We can consider 4BDE as having base BD of length 13 − 7 = 6 and height equal to the
distance of E from the y-axis (a distance of 3).
Therefore, the area of 4BDE is 21 (6)(3) = 9.
Thus, the area of the shaded region is 49
2
+ 9 = 67
2
.

Solution 2
The area of the shaded region is the sum of the areas of 4BOA and 4AEC.
4BOA is right-angled at O, so the area of 4BOA is 21 (BO)(OA).
Point A is the point where the line y = −3x + 13 crosses the x-axis, so it has x-coordinate
which satisfies −3x + 13 = 0, ie. x = 13
3
.
Therefore, the area of 4BOA is 12 (13) 13
 169
3
= 6 .
We can consider 4AEC as having base AC of length 7 − 13 3
= 83 and height equal to the
distance of E from the x-axis (a distance of 4).
Therefore, the area of 4AEC is 21 (4) 83 = 16

3
.
Thus, the area of the shaded region is 6 + 3 = 201
169 16
6
= 67
2
.

Solution 3
Drop perpendiculars from E to point X on the x-axis and to point Y on the y-axis.
y

D
E (3, 4)
Y

x
O X A C (7, 0)

Then Y has coordinates (0, 4), X has coordinates (3, 0), and OXEY is a rectangle.
The area of the shaded region is thus the sum of the areas of 4BY E, rectangle OXEY
and 4EXC.
2005 COMC Solutions Page 14 of 26

Since 4BY E is right-angled at Y , its area is 12 (BY )(Y E) = 21 (13 − 4)(3 − 0) = 27


2
.
The area of rectangle OY EX is 3 × 4 = 12.
Since 4EXC is right-angled at X, its area is 21 (EX)(XC) = 12 (4 − 0)(7 − 3) = 8.
Therefore, the area of the shaded region is 27
2
+ 12 + 8 = 672
.

2. (a) Determine all possible ordered pairs (a, b) such that

a−b = 1
2a2 + ab − 3b2 = 22

Solution 1
Factoring the left side of the second equation, we get 2a2 + ab − 3b2 = (a − b)(2a + 3b).
Since a − b = 1, we get (1)(2a + 3b) = 22 or 2a + 3b = 22.
So we now have a − b = 1 and 2a + 3b = 22.
Adding 3 times the first equation to the second equation, we get 5a = 25 or a = 5.
Substituting back into the first equation, we get b = 4.
Thus, the only solution is (a, b) = (5, 4).

Solution 2
From the first equation, a = b + 1.
Substituting into the second equation, we obtain

2(b + 1)2 + (b + 1)(b) − 3b2 = 22


(2b2 + 4b + 2) + (b2 + b) − 3b2 = 22
5b = 20
b = 4

Substituting back into the first equation, we get a = 5, so the only solution is (a, b) = (5, 4).

Solution 3
From the first equation, b = a − 1.
Substituting into the second equation, we obtain

2a2 + a(a − 1) − 3(a − 1)2 = 22


2a2 + (a2 − a) − (3a2 − 6a + 3) = 22
5a = 25
a = 5

Substituting back into the first equation, we get b = 4, so the only solution is (a, b) = (5, 4).
2005 COMC Solutions Page 15 of 26

(b) Determine all possible ordered triples (x, y, z) such that

x2 − yz + xy + zx = 82
y 2 − zx + xy + yz = −18
z 2 − xy + zx + yz = 18

Solution 1
If we add the second equation to the third equation, we obtain

y 2 − zx + xy + yz + z 2 − xy + zx + yz = −18 + 18
y 2 + 2yz + z 2 = 0
(y + z)2 = 0
y+z = 0
z = −y

Substituting back into the three equations, we obtain

x2 + y 2 = 82
2xy = −18
−2xy = 18

Thus, x2 + y 2 = 82 and xy = −9.


Therefore, (x + y)2 = x2 + 2xy + y 2 = 82 + (−18) = 64, so x + y = ±8.

If x + y = 8, then y = 8 − x and so since xy = −9, then x(8 − x) = −9 or x2 − 8x − 9 = 0


or (x − 9)(x + 1) = 0 so x = 9 or x = −1.
Since x + y = 8, then if x = 9, we have y = −1 and z = −y = 1.
Since x + y = 8, then if x = −1, we have y = 9 and z = −y = −9.

If x+y = −8, then y = −8−x and so since xy = −9, then x(−8−x) = −9 or x2 +8x−9 = 0
or (x + 9)(x − 1) = 0 so x = −9 or x = 1.
Since x + y = −8, then if x = −9, we have y = 1 and z = −y = −1.
Since x + y = −8, then if x = 1, we have y = −9 and z = −y = 9.

Therefore, the four solutions are (x, y, z) = (9, −1, 1), (−1, 9, −9), (−9, 1, −1), (1, −9, 9).
2005 COMC Solutions Page 16 of 26

Solution 2
If we add the first equation to the second equation, we obtain

x2 − yz + xy + zx + y 2 − zx + xy + yz = 82 − 18
x2 + 2xy + y 2 = 64
(x + y)2 = 64
x + y = ±8

Similarly, adding the first equation to the third equation, we obtain x2 + 2xz + z 2 = 100
or x + z = ±10.
Also, adding the second equation to the third equation, we obtain y 2 + 2yz + z 2 = 0
or y + z = 0, and so z = −y.
Using x + z = ±10 and z = −y, we obtain x − y = ±10.
Thus, we have x + y = ±8 and x − y = ±10.

If x + y = 8 and x − y = 10, then adding these equations, we get 2x = 18 or x = 9


and so y = −1 and z = −y = 1.
If x + y = 8 and x − y = −10, then adding these equations, we get 2x = −2 or x = −1
and so y = 9 and z = −y = −9.
If x + y = −8 and x − y = 10, then adding these equations, we get 2x = 2 or x = 1 and
so y = −9 and z = −y = 9.
If x + y = −8 and x − y = −10, then adding these equations, we get 2x = −18 or x = −9
and so y = 1 and z = −y = −1.

Therefore, the four solutions are (x, y, z) = (9, −1, 1), (−1, 9, −9), (−9, 1, −1), (1, −9, 9).

3. Four tiles identical to the one shown, with a > b > 0, b


are arranged without overlap to form a square with
b
a square hole in the middle.
a
a

(a) If the outer square has area (a + b)2 , show that the area of the inner square
is (a − b)2 .

Solution 1
Each tile can be split into two right-angled triangles along a diagonal, each with legs of
lengths a and b.
The area of each of these triangles is 21 ab, so the area of each tile is ab.
2005 COMC Solutions Page 17 of 26

If the outer square has area (a + b)2 and this is partially covered with four tiles each of
area ab, then the area of the leftover portion (ie. the square hole) is

(a + b)2 − 4ab = a2 + 2ab + b2 − 4ab = a2 − 2ab + b2 = (a − b)2

Solution 2
If the outer square has area (a + b)2 , then the side length of the outer square is a + b.
In order to get a side length of a + b, we need to line up the “a” side of a tile with the “b”
side of a second tile, as shown.
b a

b b
a
a b

(Note that the tiles do fit together in this way, since each is a quadrilateral with two right
angles, so the remaining two angles add to 180◦ , that is, a straight line.)
We can complete the square as follows:
b a

b b
a

b
a
b
b
a b

Now the inner hole is clearly a rectangle (as it has four right angles) and is in fact a square
as its four sides are all of length a − b (as each of its sides are the remaining portion of a
line segment of length a when a segment of length b is cut off from one end).
Since the inner square has side length a − b, then its area is (a − b)2 .

(b) Determine the smallest integer value of N for which there are prime numbers a and b such
that the ratio of the area of the inner square to the area of the outer square is 1 : N .

Solution
(a − b)2
From (a), the ratio of the area of the inner square to the area of the outer square is .
(a + b)2
We would like to find integers N for which there are prime numbers a and b such
(a − b)2 1
that 2
= (and in fact find the minimum such N ).
(a + b) N
a−b 1
Taking the positive square root of both sides, we obtain =√ .
a+b N
2005 COMC Solutions Page 18 of 26


Since the left side is a rational number (since a and b are integers), then N must be
rational, so N must be a perfect square.
Suppose N = k 2 , for some positive integer k.
a−b 1
Thus, we have = or a + b = k(a − b) or (k − 1)a = (k + 1)b.
a+b k

Since we would like to find the smallest value of N which works, then we try to find
the smallest value of k which works.
Does k = 1 work? Are there prime numbers a and b so that 0 = 2b? No, since this means
b = 0.
Does k = 2 work? Are there prime numbers a and b so that a = 3b? No, since here a is a
multiple of 3, so the only possible prime value of a is 3, which would make b = 1, which
is not a prime.
Does k = 3 work? Are there prime numbers a and b so that 2a = 4b (ie. a = 2b)? No,
since here a is a multiple of 2, so the only possible prime value of a is 2, which would make
b = 1, which is not a prime.
Does k = 4 work? Are there prime numbers a and b so that 3a = 5b?
Yes: a = 5 and b = 3.
Therefore, the smallest value of k which works is k = 4, so the smallest value of N which
works is N = 16.

(c) Determine, with justification, all positive integers N for which there are odd integers
a > b > 0 such that the ratio of the area of the inner square to the area of the outer
square is 1 : N .

Solution
Suppose that N is a positive integer for which there are odd integers a > b > 0 such that
(a − b)2 1
2
= .
(a + b) N
Then, as in (b), N must be a perfect square, say N = k 2 , for some positive integer k.
Since a and b are odd, then set a = 2A + 1 and b = 2B + 1, for some integers A and B.
(2A − 2B)2 1 A−B 1
Thus we have 2
= 2 or = or k(A − B) = A + B + 1.
(2A + 2B + 2) k A+B+1 k
If A and B have the same parity (ie. both even or both odd), then A − B is even so the
left side is even and A + B + 1 is odd, so the right side is odd. Since we cannot have an
odd number equal to an even number, then this cannot happen.
Thus, A and B must have opposite parity (ie. one even and the other odd). In this case,
A − B is odd and A + B + 1 is even. Since k(A − B) = A + B + 1, then k is even.
Therefore, N must be an even perfect square.

We must now check if every even perfect square is a possible value for N .
2005 COMC Solutions Page 19 of 26

Suppose N = (2m)2 .
Using our substitutions from above, can we find integers A and B so that
2m(A − B) = A + B + 1?
If A = m and B = m − 1, then A − B = 1 and A + B + 1 = 2m, so 2m(A − B) = A + B + 1.
(a − b)2 1 1
So if a = 2A + 1 = 2m + 1 and b = 2B + 1 = 2m − 1, then 2
= 2
= .
(a + b) (2m) N
Therefore, the positive integers N which have the required property are all even perfect
squares.

4. Triangle ABC has its base on line segment P N and vertex A on line P M . Circles with centres
O and Q, having radii r1 and r2 , respectively, are tangent to the triangle ABC externally and
to each of P M and P N .
M

E
A
D Q
O L
K
P F B C G N

(a) Prove that the line through K and L bisects the perimeter of triangle ABC.

Solution
We must show that KB + BC + CL = KA + AL.
Since BK and BF are tangents to the left circle from the same point B, then BK = BF .
Since CL and CG are tangents to the right circle from the same point C, then CL = CG.
Since AK and AD are tangents to the left circle from the same point A, then AK = AD.
Since AL and AE are tangents to the right circle from the same point A, then AL = AE.
Therefore, KB + BC + CL = F B + BC + CG = F G and KA + AL = DA + AE = DE.
Now F G = P G − P F and DE = P E − P D.
Since P E and P G are tangents to the right circle from the same point P , then P E = P G.
Since P D and P F are tangents to the left circle from the same point P , then P D = P F .
Therefore, F G = P G − P F = P E − P D = DE, so KB + BC + CL = KA + AL, ie. the
line through K and L bisects the perimeter of triangle ABC.

(b) Let T be the point of contact of BC with the circle inscribed in triangle ABC.
Prove that (T C)(r1 ) + (T B)(r2 ) is equal to the area of triangle ABC.

Solution 1
2005 COMC Solutions Page 20 of 26

Let I be the centre of the circle inscribed in 4ABC, T be the point of contact of this
circle with BC, and r the radius of this circle.
Join O to K and B, and I to B and T .
M

D Q

O L
K I
P N
F B T C G

(Note that the circle with centre I is not necessarily tangent to AB at K or AC at L.)
Note that OK is perpendicular to KB and IT is perpendicular to BC.
Now OB bisects ∠F BK and IB bisects ∠KBC, since the circles with centres O and I
are tangent to F B and BK, and BA and BC, respectively.
Now ∠KOB = 90◦ − ∠KBO = 90◦ − 21 ∠F BK = 12 (180◦ − ∠F BK) = 21 ∠KBC = ∠IBT ,
so 4OKB is similar to 4BT I.
BK IT BK r (T B)(BK)
Therefore, = or = or r1 = .
KO TB r1 TB r
(T C)(LC)
Similarly, r2 = .
r
Therefore,

(T C)(T B)(BK) (T B)(T C)(LC) (T B)(T C)


(T C)(r1 ) + (T B)(r2 ) = + = (BK + LC)
r r r
Let BC = a, AB = c, AC = b, and let s be the semi-perimeter of 4ABC (that is, s is
half of the perimeter of 4ABC).
Now, from (a), since KB + BC + LC = s, then BK + LC = s − BC = s − a.
(T B)(T C)
Therefore, (T C)(r1 ) + (T B)(r2 ) = (s − a).
r

We can now focus entirely on 4ABC.


Let X and Y be the points where the circle with centre I is tangent to sides AB and AC,
respectively.
2005 COMC Solutions Page 21 of 26

Y
X I

B C
T

Using tangent arguments as in (a), we see that AX = AY , BX = BT and CY = CT .


Since AX + AY + BX + BT + CY + CT = 2s, then BT + AY + Y C = s,
so T B = s − (AY + Y C) = s − AC = s − b.
Similarly, T C = s − c.
(s − b)(s − c)(s − a) s(s − a)(s − b)(s − c)
Therefore, (T C)(r1 ) + (T B)(r2 ) = = .
r sr
Let |4ABC| denote the area of 4ABC.
Then s(s − a)(s − b)(s − c) = |4ABC|2 by Heron’s formula.
Also,
1 1 1 1
sr = r(AB + BC + AC) = (IX)(AB) + (IT )(BC) + (IY )(AC) (∗)
2 2 2 2
since IX, IT and IY are all radii of the circle with centre I.
Since IX, IT and IY are perpendicular to AB, BC and AC, respectively, then the three
terms on the right side of (∗) are the areas of 4IAB, 4IBC and 4ICA, respectively,
and so their sum is |4ABC|, ie. sr = |4ABC|.
|4ABC|2
Thus, (T C)(r1 ) + (T B)(r2 ) = = |4ABC|, as required.
|4ABC|

Solution 2
Join O to F and B and Q to C and G.
Since the circle with centre O is tangent to P B and AB at F and K, then OF is perpen-
dicular to P B and OB bisects ∠F BK.
Similarly, QG is perpendicular to CN and QC bisects ∠GCL.
Extend AB and AC through B and C, respectively, and construct the circle which is
tangent to AB extended, BC, and AC extended, and lies outside 4ABC. This circle is
called an excircle of 4ABC.
The centre of this excircle, which we label U , is on the angle bisector of the angle formed
by AB extended and BC, as the circle is tangent to these two lines, so U lies on OB
extended. Similarly, U lies on QC extended.
2005 COMC Solutions Page 22 of 26

D Q

O L
K
P V N
F B C G
Y
X
U

Let AB = c, AC = b, BC = a, let s be the semi-perimeter (that is, half of the perimeter)


of 4ABC, and let |4ABC| denote the area of 4ABC.
|4ABC|
Then the radius of the excircle, which we will denote rA is equal to . (See the
s−a
end of this solution for a proof of this fact.)
Let V be the point where the excircle is tangent to BC.
Then U V is perpendicular to BC.
Thus, 4OF B is similar to 4U V B and 4QGC is similar to 4U V C (since they have
opposite angles which are equal and right angles).
OF UV r1 rA QG UV r2 rA
This tells us that = or = and = or = .
FB VB FB VB GC VC GC VC
Therefore, since F B = BK and CG = CL and KB + BC + CL = s by (a), then

(V B)(r1 )+(V C)(r2 ) = rA (F B+CG) = rA (KB+LC) = rA (s−BC) = rA (s−a) = |4ABC|

Now suppose that the excircle is tangent to AB extended and AC extended at X and Y ,
respectively.
Then AX = AY , and AX = AB + BX = AB + BV and AY = AC + CY = AC + CV (by
equal tangents from B and C), so AX +AY = AB+AC+BV +V C = AB+AC+BC = 2s,
ie. AX = AY = s.
Thus, V B = BX = AX − AB = s − c and similarly V C = s − b.
But T B = s − b = V C and T C = s − c = V B (see Solution 1), so

|4ABC| = (T C)(r1 ) + (T B)(r2 )

as required.

|4ABC|
(Why is rA = ?
s−a
2005 COMC Solutions Page 23 of 26

Join X, Y and V to U . Note that ∠AXU = ∠AY U = ∠BV U = ∠CV U = 90◦ .


Then AXU Y is a shape of the same type as in Problem 3, so its area is equal to
AX · U X = srA .
Similarly, the areas of BV U X and CV U Y are rA (s − c) and rA (s − b).
Thus,

|4ABC| = Area of AXU Y − Area of BV U X − Area of CV U Y = rA (s − (s − c) − (s − b))

But, s − (s − c) − (s − b) = b + c − s = a + b + c − a − s = 2s − a − s = s − a, so
|4ABC| = rA (s − a), which is what we wanted to show.)

Solution 3
Let AB = c, AC = b, BC = a, and let s denote the semi-perimeter of 4ABC (that is,
half of its perimeter).
Then by (a), AK + AL = KB + BC + LC = s.
Since P M and P N are tangent to both circles, then the line through O and Q passes
through P .
Join O to D, F and K, and Q to L, E and G.
M

D Q

O L
K
P N
F B C G

In each case, the centre of a circle is being joined to a point where the circle is tangent to
a line, so creates a right angle.
PF PG
Therefore, 4P OF is similar to 4P QG, so = or r1 (P G) = r2 (P F ).
OF QG
Each of the shapes P DOF , ADOK, BF OK, AEQL, CGQL and P EQG has two right
angles and two pairs of equal sides (ie. each is a shape as in Problem 3).
The area of each of these shapes is the product of the lengths of two of the sides which
meet at a right angle.
We use |P EQG| to denote the area of the shape P EQG, and so on.
2005 COMC Solutions Page 24 of 26

Therefore,

|P EQG| = |4ABC| + |P DOF | + |ADOK| + |BF OK| + |AEQL| + |CGQL|


(P G)(QG) = |4ABC| + (P F )(OF ) + (AK)(OK) + (KB)(OK) + (AL)(QL) + (CL)(LQ)
|4ABC| = r2 (P G) − r2 (AL) − r2 (CL) − r1 (P F ) − r1 (AK) − r1 (KB)
= r2 (P G − AL − CL) + r1 (−P F − AK − KB)
= r2 (P G − CG − AL) + r1 (−P F − AB) (equal tangents)
= r2 (P C − AL) + r1 (−P F − AB)
= r2 (P F + F B + BC − AL) + r1 (−P F − AB)
= r2 (P F + BK + a − (s − AK)) + r1 (−P F − AB)
(since AK + AL = s, and BK = F B by equal tangents)
= r2 (P F + AK + BK + a − s) + r1 (−P F − AB)
= r2 (P F + AB + a − s) + r1 (−P F − AB)
= r2 (P F ) + r2 (c + a − s) + r1 (−P F − AB)
= r1 (P G) + r2 (a + b + c − b − s) + r1 (−P F − AB) (since r1 (P G) = r2 (P F ))
= r2 (2s − b − s) + r1 (P G − P F − AB)
= r2 (s − b) + r1 (GF − AB)
= r2 (s − b) + r1 (F B + BC + CG − c)
= r2 (s − b) + r1 (KB + BC + CL − c) (equal tangents)
= r2 (s − b) + r1 (s − c)

As in Solution 1, T B = s − c and T C = s − b.
Therefore, |4ABC| = (T C)(r1 ) + (T B)(r2 ), as required.

Solution 4
Let ∠ABC = 2β, ∠ACB = 2γ and ∠M P N = 2θ.
Then ∠P AB = 2β − 2θ and ∠M AC = 2γ + 2θ, using external angles in 4P AB and
4P AC. Also, ∠ABP = 180◦ − 2β.
Since the circle with centre O is tangent to AP and AK, then O lies on the bisector of
∠P AK, so ∠KAO = β − θ. Similarly, ∠LAQ = γ + θ and ∠KBO = 90◦ − β.
Since 4OKB is right-angled at K (since AB is tangent to the circle with centre O at K),
then ∠KOB = β.
KO KB
Thus, tan(∠KAO) = tan(β − θ) = and tan(∠KOB) = tan(β) = .
AK KO
2005 COMC Solutions Page 25 of 26

Therefore,

AB = AK + KB
KO
AB = + KO tan(β)
tan(β − θ)
 
1 + tan(β) tan(θ)
AB = r1 + tan(β) (since KO = r1 )
tan(β) − tan(θ)
1 + tan(β) tan(θ) tan2 (β) − tan(β) tan(θ)
 
AB = r1 +
tan(β) − tan(θ) tan(β) − tan(θ)
2
 
1 + tan (β)
AB = r1
tan(β) − tan(θ)
AB(tan(β) − tan(θ))
r1 =
1 + tan2 (β)
AB(tan(β) − tan(θ))
r1 =
sec2 (β)
r1 = AB sin(β) cos(β) − AB cos2 (β) tan(θ)
1
r1 = 2
AB sin(2β) − AB cos2 (β) tan(θ)

But AB sin(2β) is the length of the height, h, of 4ABC from A to BC.


Thus r1 = 21 h − AB cos2 (β) tan(θ).
Similarly, r2 = 21 h + AC cos2 (γ) tan(θ).

Since the circle with centre I is tangent to AB and BC, then I lies on the angle bi-
IT
sector of ∠ABC, so ∠IBT = β, so tan(β) = .
TB
IT r
Thus, T B = = .
tan(β) tan(β)
r
Similarly, T C = .
tan(γ)
Therefore,

r1 (T C) + r2 (T B) = T C 21 h − AB cos2 (β) tan(θ) + T B 21 h + AC cos2 (γ) tan(θ)


   

1
= 2
h(T C + T B) + tan(θ) [−T C · AB cos2 (β) + T B · AC cos2 (γ)] (∗)

The first term on the right side of (∗) equals 12 h(BC) which equals the area of 4ABC.
Considering the second factor of the second term, we obtain

T B · AC cos2 (γ) − T C · AB cos2 (β)


r r
= AC cos2 (γ) − AB cos2 (β)
tan(β) tan(γ)
r
2AC tan(γ) cos2 (γ) − 2AB tan(β) cos2 (β)

=
2 tan(β) tan(γ)
r
= (2AC sin(γ) cos(γ) − 2AB sin(β) cos(β))
2 tan(β) tan(γ)
r
= (AC sin(2γ) − AB sin(2β))
2 tan(β) tan(γ)
2005 COMC Solutions Page 26 of 26

But AC sin(2γ) = AB sin(2β) = h, so this second factor equals 0, so the second term of
the right side of (∗) equals 0.
Therefore, r1 (T C) + r2 (T B) equals the area of 4ABC, as required.
The Canadian Mathematical Society
in collaboration with

The CENTRE for EDUCATION


in MATHEMATICS and COMPUTING

presents the

Canadian Open
Mathematics Challenge
Wednesday, November 22, 2006

Supported by:

Solutions

2006
c Canadian Mathematical Society
2006 COMC Solutions Page 2

Part A
1 1 1 1
   
1. What is the value of 1 + 2
1+ 3
1+ 4
1+ 5
?

Solution 1

1 1 1 1 3 4 5 6
       
1+ 2
1+ 3
1+ 4
1+ 5
= 2 3 4 5
    
= 3 4 5 6
2 3 4 5
(simplifying numerators and denominators)
6
= 2

= 3

Solution 2

1 1 1 1 3 4 5 6
       
1+ 2
1+ 3
1+ 4
1+ 5
= 2 3 4 5
360
= 120

= 3

2. If f (2x + 1) = (x − 12)(x + 13), what is the value of f (31)?

Solution 1
Since f (2x + 1) = (x − 12)(x + 13), then

f (31) = f (2(15) + 1) = (15 − 12)(15 + 13) = 3(28) = 84

Solution 2
w−1
If w = 2x + 1, then x = .
2
Since f (2x + 1) = (x − 12)(x + 13), then
     
w−1 w−1 w − 25 w + 25
f (w) = − 12 + 13 =
2 2 2 2

Therefore,   
31 − 25 31 + 25
f (31) = = 3(28) = 84
2 2
2006 COMC Solutions Page 3

3. In 4ABC, M is the midpoint of BC, as shown. If


A
∠ABM = 15◦ and ∠AM C = 30◦ , what is the size of
∠BCA?
B M C

Solution
Since ∠AM C = 30◦ , then ∠AM B = 180◦ − ∠AM C = 150◦ .
Since ∠ABM = 15◦ and ∠AM B = 150◦ , then ∠BAM = 180◦ − ∠ABM − ∠AM B = 15◦ .
Since ∠ABM = ∠BAM , then BM = M A.
Since BM = M A and BM = M C, then M A = M C, so ∠M AC = ∠M CA.
Thus, ∠M CA = 21 (180◦ − ∠AM C) = 75◦ .
Therefore, ∠BCA = ∠M CA = 75◦ .

4. Determine all solutions (x, y) to the system of equations


4 5
+ 2 = 12
x y
3 7
+ 2 = 22
x y
Solution 1
Subtracting 5 times the second equation from 7 times the first equation, we obtain
   
4 5 3 7
7 + −5 + = 7(12) − 5(22)
x y2 x y2
13
= −26
x
x = − 21

4 5 5
Substituting x = − 12 into the first equation, we obtain 1 + 2 = 12 or −8 + 2 = 12 or
−2 y y
5
= 20 or y 2 = 14 .
y2
Therefore, y = ± 12 .
Thus, the solutions are − 12 , 12 and − 12 , − 12 .
 

(We can check by substitution that both of these solutions work.)


2006 COMC Solutions Page 4

Solution 2
Subtracting 4 times the second equation from 3 times the first equation, we obtain
   
4 5 3 7
3 + −4 + = 3(12) − 4(22)
x y2 x y2
13
− 2 = −52
y
y 2 = 14
y = ± 12

4 5 4
Substituting y = ± 12 into the first equation, we obtain + 2 = 12 or + 20 = 12 or
x ± 12 x

4
= −8 or x = − 12 .
x
Thus, the solutions are − 21 , 12 and − 12 , − 12 .
 

(We can check by substitution that both of these solutions work.)

x+8
5. In 4ABC, BC = 4, AB = x, AC = x + 2, and cos(∠BAC) = .
2x + 4
Determine all possible values of x.

Solution
Using the cosine law in 4ABC,

BC 2 = AB 2 + AC 2 − 2(AB)(AC) cos(∠BAC)
x+8
42 = x2 + (x + 2)2 − 2x(x + 2)
2x + 4
2 2
16 = x + x + 4x + 4 − x(x + 8)
0 = x2 − 4x − 12
0 = (x − 6)(x + 2)

Therefore, x = 6 or x = −2.
Since AB = x, then x must be positive, so x = 6.

6. Determine the number of integers n that satisfy all three of the conditions below:

• each digit of n is either 1 or 0,


• n is divisible by 6, and
• 0 < n < 107 .

Solution 1
Since 0 < n < 107 , then n is a positive integer with fewer than 8 digits.
2006 COMC Solutions Page 5

Since n is divisible by 6, then n is even. Since each digit of n is either 1 or 0, then n must end
with a 0.
Since n is divisible by 6, then n is divisible by 3, so n has the sum of its digits divisible by 3.
Since each digit of n is 0 or 1 and n has at most 6 non-zero digits, then the sum of the digits
of n must be 3 or 6 (that is, n contains either 3 or 6 digits equal to 1).

Since n has at most 7 digits, we can write n in terms of its digits as abcdef 0, where each
of a, b, c, d, e, f can be 0 or 1. (We allow n to begin with a 0 in this representation.)
If n contains 6 digits equal to 1, then there is no choice in where the 1’s are placed so
n = 1111110.
If n contains 3 digits equal to 1, then 3 of!the 6 digits a through f are 1 (and the other 3 are
6
0). The number of such possibilities is = 20.
3
Therefore, there are 20 + 1 = 21 such integers n.

Solution 2
Since 0 < n < 107 , then n is a positive integer with fewer than 8 digits.
Since n is divisible by 6, then n is even. Since each digit of n is either 1 or 0, then n ends with
a 0.
Since n is divisible by 6, then n is divisible by 3, so has the sum of its digits divisible by 3.
Since each digit of n is 0 or 1 and n has at most 6 non-zero digits, then the sum of the digits
of n must be 3 or 6 (that is, n contains either 3 or 6 digits equal to 1).

If n contains 6 digits equal to 1, then n = 1111110, since n has at most 7 digits.


If n contains 3 digits equal to 1, then n has between 4 and 7 digits, and must begin with 1.
If n has 4 digits, then n must be 1110. !
3
If n has 5 digits, then n has the form 1abc0 with 2 of a, b, c equal to 1. There are =3
2
such possibilities. !
4
If n has 6 digits, then n has the form 1abcd0 with 2 of a, b, c, d equal to 1. There are =6
2
such possibilities.
If n!has 7 digits, then n has the form 1abcde0 with 2 of a, b, c, d, e equal to 1. There are
5
= 10 such possibilities.
2
Therefore, there are 1 + 1 + 3 + 6 + 10 = 21 possibilities for n.
2006 COMC Solutions Page 6

7. Suppose n and D are integers with n positive and 0 ≤ D ≤ 9.


n
Determine n if = 0.9D5 = 0.9D59D59D5 . . . .
810

Solution
9D5
First, we note that 0.9D5 = since
999
1000(0.9D5) = 9D5.9D5
1000(0.9D5) − 0.9D5 = 9D5.9D5 − 0.9D5
999(0.9D5) = 9D5
9D5
0.9D5 =
999
Alternatively, we could derive this result by noticing that

0.9D5 = 0.9D59D59D5 . . .
9D5 9D5 9D5
= + + + ···
103 106 109
9D5
= 103 (summing the infinite geometric series)
1
1− 3
10
9D5
=
1000 − 1
9D5
=
999
Therefore,
n 9D5
=
810 999
999n = 810(9D5)
111n = 90(9D5)
37n = 30(9D5)

Thus, 30(9D5) is divisible by 37. Since 30 is not divisible by 37 and 37 is prime, then 9D5
must be divisible by 37.
The multiples of 37 between 900 and 1000 are 925, 962 and 999.
Thus, 9D5 must be 925, so D = 2.
So 37n = 30(925) or n = 30(25) = 750.
2006 COMC Solutions Page 7

8. What is the probability that 2 or more successive heads will occur at least once in 10 tosses of
a fair coin?

Solution 1
For a given toss, we use T to represent a result of tails and H for heads.
There are 210 = 1024 possible sequences of outcomes when a fair coin is tossed 10 times.
Let tn be the number of sequences of n tosses of a fair coin which do not contain 2 or more
successive heads.
(So the number of sequences of length 10 that contain 2 or more successive heads is 1024 − t10
1024 − t10
which means that the desired probability is .)
1024

Note that t1 = 2, as both T and H do not contain 2 successive H’s.


Also, t2 = 3. (These are the sequences T T , T H, HT .)
Consider a sequence of n tosses of a fair coin which do not contain 2 or more successive heads,
where n ≥ 3.
Such a sequence must begin with H or T .
If the sequence begins with H, the second toss must be T , and the last n − 2 can be any
sequence of n − 2 tosses that does not contain 2 or more successive heads. There are tn−2 such
sequences of n − 2 tosses, so there are tn−2 sequences of n tosses beginning with H and not
containing 2 or more successive heads.
If the sequence begins with T , the last n − 1 tosses can be any sequence of n − 1 tosses that
does not contain 2 or more successive heads. There are tn−1 such sequences of n − 1 tosses, so
there are tn−1 sequences of n tosses beginning with T and not containing 2 or more successive
heads.
Therefore, tn = tn−1 + tn−2 , since each sequence begins with H or T .

Starting with t1 = 2 and t2 = 3, we can then generate the sequence tn for n = 1 to n = 10 by


adding the two previous terms to obtain the next term:

2, 3, 5, 8, 13, 21, 34, 55, 89, 144


1024 − 144 880 55
So t10 = 144, so the desired probability is = = .
1024 1024 64

Solution 2
For a given toss, we use T to represent a result of tails and H for heads.
There are 210 = 1024 possible sequences of outcomes when a fair coin is tossed 10 times.
Let us count the number of such sequences which do not contain 2 or more successive H’s, by
grouping them by the number of H’s that they contain. (Note that not containing 2 or more
successive H’s is equivalent to not containing the pair HH.)
2006 COMC Solutions Page 8

If a sequence of length 10 consists of 0 H and 10 T ’s, there is only !


1 possibility.
10
If a sequence of length 10 consists of 1 H and 9 T ’s, there are = 10 possibilities.
1

If a sequence of length 10 consists of 2 H’s and 8 T ’s, then we start with

T T T T T T T T

Each of the two H’s must be placed in separate spaces. We can then eliminate any unused
spaces to obtain a sequence of 8 T ’s and!2 H’s containing no consecutive H’s (and we get all
9
such sequences this way). There are = 36 ways of positioning the H’s, and so 36 such
2
sequences.
!
8
In a similar way, with 3 H’s and 7 T ’s, there are = 56 such sequences.
3
!
7
With 4 H’s and 6 T ’s, there are = 35 such sequences.
4
!
6
With 5 H’s and 5 T ’s, there are = 6 such sequences.
5

Therefore, there is a total of 1 + 10 + 36 + 56 + 35 + 6 = 144 sequences of 10 tosses which do


not contain 2 or more successive H’s.
Thus, there are 1024 − 144 = 880 sequences of 10 tosses which do contain 2 or more successive
H’s. Therefore, the probability that a given sequence contains 2 or more successive H’s is
880
1024
= 55
64
.
2006 COMC Solutions Page 9

Part B

1. Piotr places numbers on a 3 by 3 grid using the following rule, called “Piotr’s Principle”:

For any three adjacent numbers in a horizontal, vertical or diagonal line, the
middle number is always the average (mean) of its two neighbours.

(a) Using Piotr’s principle, determine the missing numbers


3 19
in the grid to the right. (You should fill in the missing
8
numbers in the grid in your answer booklet.)

Solution
Since the average of 3 and 19 is 12 (3 + 19) = 11, then 11 goes between the 3 and 19.
The number which goes below 8 is the number whose average with 3 is 8, so 13 goes
below 8.
The average of 13 and 19, or 16, goes in the middle square.
The number which goes to the right of the 16 is the number whose average with 8 is 16,
or 24.
The number which goes below 24 is the number whose average with 19 is 24, or 29.
The number which goes between 13 and 29 is their average, which is 21.
3 11 19
Therefore, the completed grid is 8 16 24 .
13 21 29
(We can check that each line obeys Piotr’s Principle.)

Note
There are other orders in which the squares can be filled.

(b) Determine, with justification, the total of the nine num-


x
bers when the grid to the right is completed using Piotr’s
5 23
Principle.

Note
When we have the three numbers a, X, b on a line, then X is the average of a and b, so
X = 12 (a + b).
When we have the three numbers a, b, X on a line, then b is the average of a and X, so
b = 12 (a + X) or 2b = a + X or X = 2b − a.
These facts will be useful as we solve (b) and (c).
2006 COMC Solutions Page 10

Solution 1
The average of 5 and 23 is 12 (5 + 23) = 14, which goes in the square between the 5 and 23.
Since the average of the numbers above and below the 5 equals 5, then their sum is
2(5) = 10. (Note that we do not need to know the actual numbers, only their sum.)
Similarly, the sum of the numbers above and below the 14 is 2(14) = 28 and the sum of
the numbers above and below the 23 is 2(23) = 46.
Therefore, the sum of the numbers in the whole grid is 5 + 10 + 14 + 28 + 23 + 46 = 126.

Solution 2
The average of 5 and 23 is 12 (5 + 23) = 14, which goes in the square between the 5 and 23.
Since the average of the x and the number below the 5 is 5, then the number below the 5
is 10 − x.
Since the average of the x and the bottom right number is 14, then the bottom right
number is 28 − x.
The average of 10 − x and 28 − x is 21 (10 − x + 28 − x) = 19 − x, which goes in the middle
square on the bottom row.
Since the average of 19 − x and the number above the 14 is 14, then the number above
the 14 is 2(14) − (19 − x) = 9 + x.
Since the average of 28 − x and the number above the 23 is 23, then the number above
the 23 is 2(23) − (28 − x) = 18 + x.
x 9 + x 18 + x
Thus, the completed grid is 5 14 23 and so the sum of the entries is
10 − x 19 − x 28 − x
x + 9 + x + 18 + x + 5 + 14 + 23 + 10 − x + 19 − x + 28 − x = 126.

(c) Determine, with justification, the values of x and y when


x 7
the grid to the right is completed using Piotr’s Principle.
9 y
20

Solution
The centre square is the average of 9 and y and is also the average of x and 20.
Comparing these facts, 21 (9 + y) = 21 (x + 20) or 9 + y = x + 20 or x − y = −11.
The number in the top right corner gives an average of 7 when combined with x (so equals
2(7) − x = 14 − x) and gives an average of y when combined with 20 (so equals 2y − 20).
Therefore, 14 − x = 2y − 20 or x + 2y = 34.
Subtracting the first equation from the second, we obtain 3y = 45 or y = 15.
Substituting back into the first equation, we obtain x = 4.
2006 COMC Solutions Page 11

4 7
We check by completing the grid. Starting with 9 15 gives, after some work,
20
4 7 10
9 12 15 , which does obey Piotr’s Principle.
14 17 20
Therefore, x = 4 and y = 15.

2. In the diagram, the circle x2 +y 2 = 25 intersects


y x = 11
the x-axis at points A and B. The line x = 11
intersects the x-axis at point C. Point P moves P
along the line x = 11 above the x-axis and AP Q
intersects the circle at Q.
x
A B C

(a) Determine the coordinates of P when 4AQB has maximum area. Justify your answer.

Solution
Since the circle has equation x2 + y 2 = 25, then to find the coordinates of A and B, the
x-intercepts of the circle, we set y = 0 to obtain x2 = 25 or x = ±5. Therefore, A and B
have coordinates (−5, 0) and (5, 0), respectively.
Since 4AQB has a base AB of constant length and a variable height, then the area of
4AQB is maximized when the height of 4AQB is maximized (that is, when Q is furthest
from AB).
To maximize the height of 4AQB, we would like Q to have as large a y-coordinate as
possible. Thus, we would like Q to be at the “top” of the circle – that is, at the place
where the circle intersects the y-axis.
Since the circle has equation x2 + y 2 = 25, then setting x = 0, we obtain y 2 = 25 or
y = ±5, so Q has coordinates (0, 5) as Q lies above the x-axis.
Therefore, P lies on the line through A(−5, 0) and Q(0, 5). This line has slope 1 and
y-intercept 5, so has equation y = x + 5.
Since P has x-coordinate 11 and lies on the line with equation y = x + 5, then P has
coordinates (11, 16).
2006 COMC Solutions Page 12

(b) Determine the coordinates of P when Q is the midpoint of AP . Justify your answer.

Solution
Suppose the coordinates of P are (11, p).
We will determine p so that the midpoint of P A lies on the circle. (This is equivalent to
finding P so that the point on the circle is the midpoint of P and A.)
Since A has coordinates (−5, 0), then for Q to be the midpoint of AP , Q must have coor-
dinates 21 (−5 + 11), 12 (0 + p) = (3, 21 p).


For (3, 12 p) to lie on the circle,


2
32 + 1
2
p = 25
1 2
4
p = 16
2
p = 64
p = ±8

Since P must lie above the x-axis, then p = 8.


Therefore, P has coordinates (11, 8).

1
(c) Determine the coordinates of P when the area of 4AQB is 4
of the area of 4AP C. Jus-
tify your answer.

Solution 1
Join Q to B.
y x = 11

P
Q

x
A B C

Since AB is a diameter of the circle, then ∠AQB = 90◦ .


Thus, since 4AQB and 4ACP are both right-angled and share a common angle at A,
then 4AQB and 4ACP are similar.
Since the area of 4ACP is 4 times the area of 4AQB, then the sides of 4ACP are

4 = 2 times as long as the corresponding sides of 4AQB.
Since AB = 10, then AP = 2AB = 20.
2006 COMC Solutions Page 13

We also know that AC = 16 (since C has coordinates (11, 0) and A has coordinates
(−5, 0)).
√ √
Therefore, by the Pythagorean Theorem, P C = AP 2 − AC 2 = 202 − 162 = 12.
Thus, P has coordinates (11, 12).

Solution 2
Let the coordinates of P be (11, p), and the coordinates of Q be (a, b). Thus, the height
of 4AQB is b.
The area of 4AQB is 21 (AB)(b) = 5b since AB = 10.
The area of 4AP C is 21 (AC)(p) = 8p since AC = 16.
Since the area of 4AQB is 14 that of 4AP C, then 5b = 2p or b = 25 p.
This tells us that Q must be 52 of the way along from A to P .
Since A has x-coordinate −5 and P has x-coordinate 11, then Q has x-coordinate
−5 + 25 (11 − (−5)) = 75 . Therefore, Q has coordinates 57 , 25 p .


Since the circle has equation x2 + y 2 = 25, then

7 2
 2
2
5
+ 5
p = 25
49 4 2 625
25
+ 25
p = 25
2
4p = 576
p2 = 144

and so p = 12 since p > 0. Therefore, P has coordinates (11, 12).

3. (a) In the diagram, trapezoid ABCD has parallel


A B
sides AB and DC of lengths 10 and 20 and sides
AD and BC of lengths 6 and 8. Determine the
area of trapezoid ABCD. D C
Solution 1
Extend DA and CB to meet at P .
P

A B

D C

Since AB is parallel to DC, then ∠P AB = ∠P DC and ∠P BA = ∠P CD.


Therefore, 4P AB is similar to 4P DC.
2006 COMC Solutions Page 14

Since AB = 12 DC, then the sides of 4P AB are 21 the length of the corresponding sides of
4P DC.
Therefore, P A = AD = 6 and P B = BC = 8.
Thus, the sides of 4P DC have lengths 12, 16 and 20. Since 122 + 162 = 202 , then 4P DC
is right-angled at P by the Pythagorean Theorem.
Thus, the area of 4P DC is 21 (12)(16) = 96.
Since 4P AB is right-angled at P , its area is 12 (6)(8) = 24.
Therefore, the area of trapezoid ABCD is 96 − 24 = 72.

Solution 2
Drop perpendiculars from A and B to P and Q on DC.
A B

D P Q C

Let AP = BQ = h and let DP = x.


Since AB = 10 and ABQP is a rectangle, then P Q = 10.
Since DC has length 20, then QC = 20 − x − 10 = 10 − x.
Using the Pythagorean Theorem in 4DP A, we obtain x2 + h2 = 62 .
Using the Pythagorean Theorem in 4CQB, we obtain (10 − x)2 + h2 = 82 .
Subtracting the first of these equations from the second, we obtain 100 − 20x = 28 or
20x = 72 or x = 185
.
2 576
Substituting back into the first equation, h2 = 36 − 18
5
= 25 so h = 24
5
.
1 24

Therefore, the area of trapezoid ABCD is 2 5 (10 + 20) = 72.

Solution 3
Drop perpendiculars from A and B to P and Q on DC.
A B

D P Q C

Cut out rectangle ABQP and join the two remaining pieces along the cut line.
The remaining shape is a triangle DCX with side lengths DX = 6, XC = 8 and DC =
20 − 10 = 10. Since 62 + 82 = 102 , then 4DCX is right-angled by the Pythagorean
Theorem.
XC 8 4
Since sin(∠XDC) = = = , then the length of the altitude from X to DC is
DC 10 5
XD sin(∠XDC) = 6 45 = 24

5
2006 COMC Solutions Page 15

which is also the height of the original trapezoid.


(We could also determine the length of this altitude by calculating the area of 4XDC in
two ways: once as 21 (DX)(XC) = 24 and once as 21 h(DC) = 5h.)
Therefore, the area of the original trapezoid is 21 24

5
(10 + 20) = 72.
(Alternatively, the area of the original trapezoid is the sum of the areas of rectangle ABQP
( 24
5
× 10 = 48) and 4XDC (24), for a total of 72.)

Solution 4
Draw BX from B to X on DC so that BX is parallel to AD.
A B

D X C

Then ABXD is a parallelogram so BX = AD = 6 and DX = AB = 10.


Therefore, XC = DC − DX = 10.
Thus, 4BXC has sides of length 6, 8 and 10. Since 62 + 82 = 102 , then 4BXC is right-
angled at B by the Pythagorean Theorem. The area of 4BXC is thus 12 (6)(8) = 24.
Join B to D. BD divides the area of ABXD in half.
Also, BX divides the area of 4BDC in half, since it is a median.
Therefore, the areas of 4ABD, 4BDX and 4XBC are all equal.
So the area of trapezoid ABCD is 3(24) = 72.

Solution 5
Let X be the midpoint of DC. Join X to A and B.
Then AB = DX = XC = 10.
Since AB = DX and AB is parallel to DX, then AD and BX are parallel and equal, so
BX = 6.
Since AB = XC and AB is parallel to XC, then AX and BC are parallel and equal, so
AX = 6.
Therefore, the trapezoid is divided into three triangles, each of which has side lengths 6,
8 and 10.
A triangle with side lengths 6, 8 and 10 is right-angled (since 62 + 82 = 102 ), so has area
1
2
(6)(8) = 24.
Therefore, the area of the trapezoid is 3 × 24 = 72.
2006 COMC Solutions Page 16

(b) In the diagram, P QRS is a rectangle and


R T S
T is the midpoint of RS. The inscribed
circles of 4P T S and 4RT Q each have
radius 3. The inscribed circle of 4QP T
has radius 4. Determine the dimensions of
rectangle P QRS. Q P
Solution 1
Let RT = a (so RS = 2a) and RQ = b.
Drop a perpendicular from T to Z on QP . By symmetry, Z is also the point of tangency
of the middle circle to QP .
Let O be the centre of the circle inscribed in 4QT P and C be the centre of the circle
inscribed in 4RT Q.
Let A, B and D be the points of tangency of the circle with centre C to QR, RT and QT ,
respectively.
Let Y be the point of tangency of the circle with centre O to QT .
Join C to A, B and D, and O to Y . These radii are perpendicular to QR, RT , QT , and
QT , respectively.

R B T S
A C
Y
D
O

Q Z P

We know that OY = OZ = 4.
Since T Z = b, then T O = b − 4.
Since QZ = a, then QY = a (equal tangents).
Since CA = CB = 3 and RACB is a rectangle (as it has three right angles), then RACB
is a square and RA = RB = 3.
Therefore, AQ = b − 3 and BT = a − 3.
By equal tangents, T D = BT = a − 3 and QD = QA = b − 3.
Now, T Y = QT − QY = QD + T D − QY = (b − 3) + (a − 3) − a = b − 6.
Therefore, 4T OY is right-angled at Y with sides of length T O = b − 4, T Y = b − 6 and
OY = 4.
By the Pythagorean Theorem, 42 + (b − 6)2 = (b − 4)2 or 4b = 36 or b = 9.
OY 4
Therefore, T Y = 9 − 6 = 3 and tan(∠OT Y ) = = .
TY 3
4 a
Also, 3 = tan(∠QT Z) = .
b
Since b = 9, a = 12.
2006 COMC Solutions Page 17

Therefore, the rectangle is 24 by 9.

Solution 2
Let RT = a (so RS = 2a) and RQ = b.
Drop a perpendicular from T to Z on QP . By symmetry, Z is also the point of tangency
of the middle circle to QP , and QZ = a.
Since the incircle of 4QRT has radius 3, then so does the incircle of 4QZT .
Let O be the centre of the circle inscribed in 4QT P and C 0 the centre of the circle in-
scribed in 4QZT .
Since C 0 and O both lie on the angle bisector of ∠T QP , then tan(∠C 0 QP ) = tan(∠OQP ).
3 4
Since C 0 is 3 units from the line T Z, then tan(∠C 0 QP ) = and tan(∠OQP ) = so
a−3 a
3 4
= or 3a = 4a − 12 or a = 12.
a−3 a
We can calculate b = 9 as in Solution 1, to obtain that the rectangle is 24 by 9.

Solution 3
Let RT = a (so RS = 2a) and RQ = b.
We calculate the areas of each of 4QRT and 4QT P in two ways: once using the stan-
dard 12 bh formula and once using the less well-known Area = sr formula, where s is the
semi-perimeter of the triangle (that is, half of the perimeter) and r is the inradius (that
is, the radius of the inscribed circle).

In 4QRT , RT = a, RQ = b, QT = a2 + b2 and the inradius is 3, so

1

2
ab = 12 (a + b + a2 + b2 )(3)

In 4QT P , QT = T P = a2 + b2 , QP = 2a, the height is b, and the inradius is 4, so

1

2
(2a)b = 12 (2 a2 + b2 + 2a)(4)

Simplifying these equations, we get the system of equations



ab = 3(a + b) + 3 a2 + b2

ab = 4a + 4 a2 + b2

Eliminating the ab terms, we obtain 3b − a = a2 + b2 .
Squaring both sides, we obtain 9b2 − 6ab + a2 = a2 + b2 or 8b2 − 6ab = 0.
a 4
Since b 6= 0, = .
b 3
2006 COMC Solutions Page 18

Substituting a = 43 b into the first equation yields


q
4 2
3
b = 4b + 3b + 3 16
9
b2 + b2

4 2
3
b = 7b + 25b2
4 2
3
b = 7b + 5b (since b > 0)
2
4b = 36b
b = 9

since b 6= 0. Therefore, a = 12 and the rectangle is 24 by 9.

Solution 3
We use the notation and diagram from Solution 1.
Since RS and QP are parallel, then ∠BT D = ∠Y QZ.
Since C and O are the centres of inscribed circles, then C lies on the angle bisector of
∠BT D and O lies on the angle bisector of ∠Y QZ.
Therefore, ∠BT C = 12 ∠BT D = 12 ∠Y QZ = ∠OQZ.
Therefore, 4BT C and 4ZQO are similar, as each is right-angled.
Thus, BT : QZ = BC : OZ = 3 : 4.
Suppose that BT = 3x and QZ = 4x.
Then RT = RB + BT = AC + 3x = 3 + 3x, since RBCA is a square.
But RT = QZ so 4x = 3 + 3x or x = 3.
Let QA = y.
Then QT = QD + DT = QA + BT by equal tangents, so QT = y + 3x = y + 9.
Since 4QRT is right-angled, then

QR2 + RT 2 = QT 2
(y + 3)2 + 122 = (y + 9)2
y 2 + 6y + 9 + 144 = y 2 + 18y + 81
12y = 72
y = 6

Therefore, since RT = 3 + 3x, then RS = 24 and RQ = 3 + y = 9, so the rectangle is 24


by 9.
2006 COMC Solutions Page 19

p
4. (a) Determine, with justification, the fraction , where p and q are positive integers and
q
q < 100, that is closest to, but not equal to, 37 .

Solution
We would like to find positive integers p and q with q < 100 which minimizes

p 3 7p − 3q |7p − 3q|
− =
q 7 7q =

7q

To minimize such a fraction, we would like to make the numerator small while making the
denominator large.
Since the two fractions pq and 73 are not equal, the numerator of their difference cannot be
0. Since the numerator is a positive integer, its minimum possible value is 1.
We consider the largest possible values of q (starting with 99) and determine if 7p − 3q
can possibly be equal to 1 or −1.

If q = 99, 7p − 3q = 7p − 297, which cannot equal ±1 since the nearest multiple of 7


to 297 is 294.
If q = 98, 7p − 3q = 7p − 294, which cannot equal ±1 since 7p − 294 is always divisible
by 7.
If q = 97, 7p − 3q = 7p − 291, which cannot equal ±1 since the nearest multiple of 7 to
291 is 294.
If q = 96, 7p − 3q = 7p − 288, which equals −1 if p = 41.

3 p 1
If p = 41 and q = 96, the difference between the fractions and is .
7 q 7(96)
|7p − 3q|
If q > 96, the numerator of is always at least 2, so the difference is at least
7q
2 1
> .
7(99) 7(96)
p 3 1 1
If q < 96, the difference between and is at least > .
q 7 7(95) 7(96)
41
So 96
minimizes the difference, so it is the closest fraction to 37 under the given conditions.
2006 COMC Solutions Page 20

a c a+c
(b) The baseball sum of two rational numbers and is defined to be . (A rational
b d b+d
number is a fraction whose numerator and denominator are both integers and whose de-
nominator is not equal to 0.) Starting with the rational numbers 01 and 11 as Stage 0, the
baseball sum of each consecutive pair of rational numbers in a stage is inserted between
the pair to arrive at the next stage. The first few stages of this process are shown below:
0 1
STAGE 0: 1 1

0 1 1
STAGE 1: 1 2 1

0 1 1 2 1
STAGE 2: 1 3 2 3 1

0 1 1 2 1 3 2 3 1
STAGE 3: 1 4 3 5 2 5 3 4 1

Prove that
(i) no rational number will be inserted more than once,

Solution
a c
Consider two rational numbers and which occur next to each other at a given
b d
a c
stage with < .
b d
Note that this means that ad < bc or bc − ad > 0.
a+c
The rational number that will be inserted between them at the next stage is .
b+d
Now
a a+c
< ⇔ a(b + d) < b(a + c) ⇔ 0 < bc − ad
b b+d
which we know to be true, and
a+c c
< ⇔ d(a + c) < c(b + d) ⇔ 0 < bc − ad
b+d d
which is again true.
a a+c c
Therefore, < < .
b b+d d
This tells us that every rational number which is inserted at any given stage is strictly
between the two rational numbers on either side.
Therefore, once a given rational number is inserted, every other rational number which
is inserted must be either strictly larger or strictly smaller, as the list at each stage
must be strictly increasing.
Therefore, no rational number will be inserted more than once.
2006 COMC Solutions Page 21

(ii) no inserted fraction is reducible,

Solution
First, we prove a lemma.

Lemma
a c
If < are consecutive rational numbers in a given stage, then bc − ad = 1.
b d

Proof
At Stage 0, the two fractions obey this property.
Assume that the property holds for all fractions in Stage k.
a c
Consider two consecutive fractions < at Stage k.
b d
a c a+c
The fraction that will be inserted between and at Stage k + 1 is ,
b d b+d
a a+c c
giving < < .
b b+d d
Note that b(a + c) − a(b + d) = bc − ad = 1 and c(b + d) − d(a + c) = bc − ad = 1.
This tells us that each pair of consecutive fraction at Stage k + 1 obeys this property.
Therefore, by induction, the required property holds.

kp a c
Suppose then that a fraction (k, p, q ∈ Z+ ) is inserted between and .
kq b d
By the Lemma, we must have b(kp) − a(kq) = 1 or k(bp − aq) = 1.
Therefore, k divides 1, so k = 1.
Thus, any inserted fraction can only have a common factor of 1 between its numerator
and denominator, so is irreducible. Thus, no inserted fraction is reducible.

(iii) every rational number between 0 and 1 will be inserted in the pattern at some stage.

Solution 1
1 n−1
We note first that every rational number of the forms and for n ≥ 2 do enter
n n
the pattern as the first and last new entry in Stage n − 1. (These rational numbers
0 1 n−2 1
enter between and , and and , respectively.)
1 n−1 n−1 1

Assume that there are rational numbers between 0 and 1 which are not inserted
in the pattern at the some stage.

p
Suppose that with p, q ∈ Z+ and gcd(p, q) = 1 is such a rational number with
q
minimal denominator. (Note that all irreducible fractions with denominators 1, 2, 3
are inserted already.)
2006 COMC Solutions Page 22

Since gcd(p, q) = 1, the Diophantine equation py − qx = 1 has solutions.


In fact, this Diophantine equation has a unique solution with 0 ≤ y < q. We can say
further that 0 < y < q since q > 1. (If y = 0, we have −qx = 1 so q would have to
be 1.)
Since 0 < y < q and qx = py − 1, then 0 < qx < pq so 0 < x < p as well.

Suppose that (x, y) = (a, b) is this unique solution.


a p
Note that pb − qa = 1 so pb > qa so < .
b q
p−a
Consider the fraction .
q−b
Its numerator and denominator are each a positive integer since 0 < a < p and
0 < b < q. Also, note that b(p − a) − a(q − b) = bp − aq = 1.
a p−a
Among other things, this tells us that < .
b q−b
a p−a p
If we can prove that and are consecutive at some stage, then will be
b q−b q
inserted between them at the next stage.

a p−a
Since each of and has a denominator less than q, it appears in the pat-
b q−b
p
tern as has the smallest denominator among those fractions which do not appear.
q
a 0
Note next that cannot be .
b 1
(If it was, a = 0 so bp − aq = 1 gives bp = 1 so p = 1.
We know that every fraction with p = 1 enters the pattern, so p 6= 1.)
p−a 1
Also, cannot be .
q−b 1
(If it was, then p − a = q − b so b(p − a) − a(q − b) = 1 gives (b − a)(q − b) = 1 so
b − a = 1.
Since p − a = q − b then q − p = b − a = 1.
n−1
But every fraction of the form enters the pattern, so q − p 6= 1.)
n
a p−a
This tells us that each of and actually entered the pattern at some stage.
b q−b

There are now three cases: b < q − b, b > q − b and b = q − b.


• Assume that b < q − b.
p−a
Consider the point when the fraction was inserted into the pattern.
q−b
p−a m M
Suppose that was inserted immediately between < .
q−b n N
2006 COMC Solutions Page 23

(That is, m + M = p − a and n + N = q − b).


Note that 0 < n < q − b. (n and N cannot be 0 since denominators cannot be 0.)

m p−a
Since < are consecutive fractions at this stage, we must also have
n q−b
(p − a)n − (q − b)m = 1.

But since gcd(p − a, q − b) = 1, the Diophantine equation (p − a)Y − (q − b)X = 1


must have a unique solution with 0 < Y < q − b.
But (X, Y ) = (a, b) is such a solution since 0 < b < q −b and b(p−a)−a(q −b) = 1
(from above).
a p−a
Therefore, must be the fraction immediately to the left of at the stage
b q−b
p−a p
where enters, which means that will be inserted into the pattern, contra-
q−b q
dicting our assumption.

• Assume that q − b < b.


a
Consider the point when the fraction was inserted into the pattern.
b
a m M
Suppose that was inserted immediately between < (that is, m + M = a
b n N
and n + N = b).
Note that 0 < n < b. (n and N cannot be 0 since denominators cannot be 0.)

m a
Since < are consecutive fractions at this stage, we must also have
n b
an − bm = 1.

But since gcd(a, b) = 1, the Diophantine equation aY − bX = 1 must have a


unique solution with 0 < Y < b.
But (X, Y ) = (p − a, q − b) is such a solution since 0 < q − b < b and
b(p − a) − a(q − b) = 1 (from above).
p−a a
Therefore, must be the fraction immediately to the left of at the stage
q−b b
a p
where enters, which means that will be inserted into the pattern, contradict-
b q
ing our assumption.

• Assume that q − b = b.
In this case, q = 2b.
But bp − aq = 1 so b(p − 2a) = 1, and so b = 1 giving q = 2.
But we know that every irreducible fraction with denominator 2 does enter.
(Namely, the fraction 12 .)
So this case cannot occur.
2006 COMC Solutions Page 24

Thus, every rational number between 0 and 1 will be inserted in the pattern at some
stage.

Solution 2
a c
Suppose and are consecutive (irreducible) fractions at some stage.
b a c d
Define S , = a + b + c + d, the sum of the numerators and denominators of the
b d
consecutive fractions.
We consider the minimum value of S at a given stage.
a+c a a+c c
When is inserted with < < , the two new sums are
b+d b b+d d
 
a a+c
S , = a + b + a + c + b + d = 2a + 2b + c + d
b b+d

and  
a+c c
S , = a + c + b + d + c + d = a + b + 2c + 2d
b+d d
a c 
each of which is larger than S , .
b d
So the minimum value of these sums must increase from one stage to the next.

a
Suppose that the fraction between 0 and 1 with a, b ∈ Z+ and gcd(a, b) = 1 is
b
never inserted into the pattern.
a
At any given Stage, since the fraction does not occur, it must be strictly between
b
m1 a m2 a
two consecutive fractions, say < < . (Since never occurs, we must be able
n1 b n2 b
to find such a pair of fractions at every Stage.)
(We know that m2 n1 − n2 m1 = 1 from (b).)
Thus, m1 b < n1 a and n2 a < m2 b. Since each of these quantities is a positive integer,
n1 a − m1 b ≥ 1 and m2 b − n2 a ≥ 1.
Now

m2 + n2 + m1 + n1
= (m2 + n2 )(1) + (m1 + n1 )(1)
≤ (m2 + n2 )(n1 a − m1 b) + (m1 + n1 )(m2 b − n2 a)
= m2 n1 a + n1 n2 a − m1 m2 b − m1 n2 b + m1 m2 b + m2 n1 b − m1 n2 a − n1 n2 a
= a(m2 n1 − m1 n2 ) + b(m2 n1 − m1 n2 )
= a+b
a
But for a fixed fraction , a + b is fixed and the minimum possible value of
b
m2 + n2 + m1 + n1
2006 COMC Solutions Page 25

increases from one stage to the next, so eventually a + b < m2 + n2 + m1 + n1 , a


contradiction, since once this happens we cannot find two fractions in that Stage
a
between which to put .
b
That is, there will be a stage beyond which we cannot find two consecutive fractions
a a
with between them. This means that must actually occur in the pattern.
b b
Therefore, every rational number between 0 and 1 will be inserted in the pattern at
some stage.
The Canadian Mathematical Society
in collaboration with

The CENTRE for EDUCATION


in MATHEMATICS and COMPUTING

presents the

Sun Life Financial


Canadian Open Mathematics Challenge
Wednesday, November 21, 2007

Solutions

2007
c Canadian Mathematical Society
2007 COMC Solutions Page 2

Part A

1. Solution 1
If a = 15 and b = −9, then

a2 + 2ab + b2 = (a + b)2 = (15 + (−9))2 = 62 = 36

Solution 2
If a = 15 and b = −9, then

a2 + 2ab + b2 = 152 + 2(15)(−9) + (−9)2 = 225 − 270 + 81 = 36

Answer: 36

30 1
2. Since there are 60 seconds in a minute, the wind power generator turns = of a revolution
60 2
each second.
Since a full revolution is 360◦ , then the generator turns 21 (360◦ ) = 180◦ each second.
(Alternatively, the generator turns through 30×360◦ in one minute, so through 30×360◦ ÷60 =
180◦ in one second.)

Answer: 180

3. Since AD = 4 and AD is perpendicular to the x-axis, then A has y-coordinate 4.


Suppose that the coordinates of A are (a, 4). (This tells us also that D has coordinates (a, 0).)
Since A lies on the line y = x + 10, then 4 = a + 10, or a = −6.
Therefore, A has coordinates (−6, 4) and D has coordinates (−6, 0).
Since ABCD is a rectangle, then AB is parallel to the x-axis, so B has y-coordinate 4.
Suppose that the coordinates of B are (b, 4). (This tells us also that C has coordinates (b, 0)
since BC is perpendicular to the x-axis.)
Since B lies on the line y = −2x + 10, then 4 = −2b + 10 so 2b = 6 or b = 3.
Therefore, B has coordinates (3, 4) and C has coordinates (3, 0).
Now the height of rectangle ABCD equals the length of AD, so is 4.
The width of rectangle ABCD equals the length of CD, which is 3 − (−6) = 9.
Therefore, the area of rectangle is 9 × 4 = 36.

Answer: 36

4. Solution 1
Suppose that there were 3k boys and 2k girls in the school in June, for some positive integer k.
2007 COMC Solutions Page 3

In September, there were thus 3k − 80 boys and 2k − 20 girls in the school. Since the new ratio
is 7 : 5, then
3k − 80 7
=
2k − 20 5
5(3k − 80) = 7(2k − 20)
15k − 400 = 14k − 140
k = 260

Therefore, the total number of the students in the school in June was 3k + 2k = 5k = 5(260),
or 1300 students.

Solution 2
Suppose that there were b boys and g girls in the school in June.
In September, there were thus b − 80 boys and g − 20 girls in the school.
b 3 b − 80 7
From the given information, we know that = and = .
g 2 g − 20 5
Eliminating fractions gives the equations 2b = 3g and 5(b−80) = 7(g−20) or 5b−400 = 7g−140
or 5b − 7g = 260.
Multiplying the second equation by 2 gives 10b − 14g = 520, and substituting 10b = 15g gives
g = 520.
Therefore, b = 32 (520) = 780, so there were b + g = 780 + 520 = 1300 students in the school in
June.

Answer: 1300

5. Solution 1
When the nine numbers are placed in the array in any arrangement, the sum of the row sums
is always 1 + 2 + 3 + 4 + 5 + 6 + 7 + 8 + 9 = 45, because each of the nine numbers appears in
exactly one row.
Similarly, the sum of the column sums is also always 45, as again each of the nine numbers
appears in exactly one column.
Therefore, the grand sum S equals 90 plus the sum of the diagonal sums, and so depends only
on the diagonal entries, labelled in the array below:

a c
e
g k

So S = 90 + (a + e + k) + (c + e + g) = 90 + 2e + a + c + g + k. To make S as large as possible,


we must make 2e + a + c + g + k as large as possible.
2007 COMC Solutions Page 4

Since a, c, e, g, k can be any of the numbers from 1 to 9, then S is largest when e = 9 and
a, c, g, k are 5, 6, 7, 8 in some order, for example in the configuration below:

5 1 6
2 9 3
7 4 8

Therefore, the maximum possible value of S is 90 + 2(9) + 8 + 7 + 6 + 5 = 90 + 44 = 134.

Solution 2
Suppose that a, b, c, d, e, f, g, h, k represent the numbers 1 to 9 in some order, and are entered
in the array as shown:

a b c
d e f
g h k

The grand sum is thus

S = (a + b + c) + (d + e + f ) + (g + h + k) + (a + d + g) + (b + e + h) + (c + f + k) +
(a + e + k) + (c + e + g)
= 4e + 3a + 3c + 3g + 3k + 2b + 2d + 2f + 2h

To make S as large as possible, we should assign the values of a, b, c, d, e, f, g, h, k so that the


largest values go to the variables with the largest coefficients in the expression for S.
In other words, S will be maximized when e = 9, a, c, g, k equal 8, 7, 6, 5 in some order, and
b, d, f, h equal 4, 3, 2, 1 in some order.
Therefore, the maximum possible value of S is

S = 4(9) + 3(8 + 7 + 6 + 5) + 2(4 + 3 + 2 + 1) = 36 + 3(36) + 2(10) = 134

Answer: 134

6. Suppose that r is the radius of the circle.


Join O to P and drop a perpendicular from P to Q on OA.

O
Q P

A N
2007 COMC Solutions Page 5

Since OA and P N are perpendicular to AN and P Q is perpendicular to OA, then QP N A is


a rectangle. Therefore, QP = AN = 15 and QA = P N = 9.
Since O is the centre of the circle and A and P are on the circumference, then OA = OP = r.
Since OA = r and QA = 9, then OQ = r − 9.
Since 4OQP is right-angled at Q, then, by the Pythagorean Theorem,

OP 2 = OQ2 + QP 2
r2 = (r − 9)2 + 152
r2 = r2 − 18r + 81 + 225
18r = 306
r = 17

Therefore, the radius is 17.

Answer: 17

7. From the second equation, x + y = 7 − z, so after squaring both sides, we obtain

x2 + 2xy + y 2 = 49 − 14z + z 2 (∗)

From the third equation, x2 + y 2 = 133 − z 2 , so using this and the first equation to substitute
into (∗), we get

(133 − z 2 ) + 2(z 2 ) = 49 − 14z + z 2


14z = −84
z = −6

Substituting this value for z back into the first two equations, we get xy = (−6)2 = 36 and
x + y = 7 − (−6) = 13.
Therefore, y = 13 − x and so x(13 − x) = 36 or 0 = x2 − 13x + 36.
This tells us that 0 = (x − 4)(x − 9) so x = 4 or x = 9.
If x = 4, then y = 13 − x tells us that y = 9.
If x = 9, then y = 13 − x tells us that y = 4.
Therefore, the solutions are (x, y, z) = (4, 9, −6), (9, 4, −6).

Answer: (4, 9, −6), (9, 4, −6)

8. In order to travel from A to B along the segments without travelling along any segment more
than once, we must always move up, down or to the right. (In other words, we can never
travel to the left without retracing our steps.) To see this, we note that if we do travel along
2007 COMC Solutions Page 6

a segment to the left, then we must have travelled along the other horizontal segment in this
square to the right at an earlier stage. We would then need to travel back along one of these
segments to get to B, thus retracing our steps.
Any route from A to B involves exactly 9 moves to the right and some number of moves up
and down.
Any route from A to B involves exactly one more move down than moves up, as we start at the
top of the grid and end up at the bottom. Therefore, the total number of up and down moves
must be odd, as it equals (x + 1) + x = 2x + 1, where x is the total number of up moves.
There are 10 vertical segments. Any choice of an odd number of these vertical segments uniquely
determines a route from A to B, as we must start at A, travel to the top of the leftmost of
these segments, travel down the segment, travel to the right to the bottom of the next segment,
travel up it, and so on.
Therefore, the routes from A to B are in exact correspondence with choices of an odd number
of the 10 vertical segments.
We compute the number of routes using n of these segments, for n = 1, 3, 5, 7, 9. In each case,
the length of the route will be 9 + n. ! !
10 10
For n = 1 and n = 9, the number of routes is = = 10.
1 9
! !
10 10 10(9)(8)
For n = 3 and n = 7, the number of routes is = = = 120.
3 7 3(2)(1)
!
10 10(9)(8)(7)(6) 10(9)(8)(7)
For n = 5, the number of routes is = = = 2(9)(2)(7) = 252.
5 5(4)(3)(2)(1) 5(4)
Therefore, the route length with the maximum number of routes is when n = 5. In this case,
the route length is 14 and the number of routes is 252.
(Instead of going !through all of the above calculations, we could have remarked that among
10
the numbers , the largest occurs when n is exactly half of 10.)
n

Answer: Length= 14, Number of Routes= 252


2007 COMC Solutions Page 7

Part B

1. (a) Since x − 1, 2x + 2, and 7x + 1 form an arithmetic sequence, then

(2x + 2) − (x − 1) = (7x + 1) − (2x + 2)


x + 3 = 5x − 1
4 = 4x
x = 1

so x = 1.
(b) Solution 1
Since x = 1, the first term of the sequence is 0.
Since the last term is 72, the sequence is arithmetic, and we are told that there is a middle
0 + 72
term, then this middle term is equal to = 36.
2
(Note that if there was an even number of terms, there would not necessarily be a middle
term. Since we are asked to find the middle term, we can safely assume that there is one!)

Solution 2
Since x = 1, the first three terms of the sequence are 0, 4, 8.
Since the common difference is 4 and the first term is 0, the number of times that the
72 − 0
difference needs to be added to get to the final term of 72 is = 18.
4
Therefore, 72 is the 19th term.
The middle term is thus the 10th term, or 0 + 4(10 − 1) = 36.
(c) Since y − 1, 2y + 2, and 7y + 1 form a geometric sequence, then
2y + 2 7y + 1
=
y−1 2y + 2
2
(2y + 2) = (y − 1)(7y + 1)
4y 2 + 8y + 4 = 7y 2 − 6y − 1
0 = 3y 2 − 14y − 5
0 = (3y + 1)(y − 5)

Therefore, y = − 13 or y = 5.
(d) If y = − 13 , the first three terms of the sequence are − 34 , 43 , − 34 .
4
3
In this case, the common ratio between successive terms is = −1.
− 34
Therefore, the 6th term in this sequence is − 34 (−1)5 = 43 .

If y = 5, the first three terms of the sequence are 4, 12, 36.


2007 COMC Solutions Page 8

12
In this case, the common ratio between successive terms is = 3.
4
Therefore, the 6th term in this sequence is 4(35 ) = 4(243) = 972.

2. (a) Solution 1
Since ∠ABC = ∠BCD = 90◦ , then BA and CD are parallel, so ABCD is a trapezoid.
Thus, the area of ABCD is 12 (24)(9 + 18) = 12(27) = 324.

Solution 2
Since ∠ABC = 90◦ , then the area of 4ABC is 12 (9)(24) = 9(12) = 108.
Also, since ∠BCD = 90◦ , then 4ACD has height 24.
Therefore, the area of 4ACD is 12 (18)(24) = 9(24) = 216.
Thus, the area of quadrilateral ABCD is 108 + 216 = 324.
(b) Solution 1
Since BA is parallel to CD, then ∠ABD = ∠BDC.
Since ∠BEA = ∠DEC as well, then 4ABE is similar to 4CDE.
DE CD 18
Therefore, = = = 2, so DE : EB = 2 : 1, as required.
BE AB 9

Solution 2
As suggested by the diagram, we coordinatize the diagram.
Put C at the origin, D on the positive x-axis (with coordinates (18, 0)) and B on the
positive y-axis (with coordinates (0, 24)).
Since ∠ABC = 90◦ , then A has coordinates (9, 24).
Therefore, the line through C and A has slope 24 9
= 83 so has equation y = 83 x.
Also, the line through B and D has slope −24 18
= − 43 , so has equation y = − 34 x + 24.
Point E lies at the point of intersection of these lines, so we combine the equations to find
the coordinates of E, getting 38 x = − 43 x + 24 or 4x = 24 or x = 6.
Therefore, E has y-coordinate 38 (6) = 16, so E has coordinates (6, 16).
To show that DE : EB = 2 : 1, we can note that E lies one-third of the way along from B
to D since the x-coordinate of E is one-third that of D (and the x-coordinate of B is 0),
or since the y-coordinate of E is two-thirds that of B (and the y-coordinate of D is 0).
Alternatively, we could calculate the length BE (which is 10) and the length of ED (which
is 20).
Using any of these methods, DE : EB = 2 : 1.
(c) Solution 1
From (b), 4ABE is similar to 4CDE and their sides are in the ratio 1 : 2.
This also tells us that the height of 4CDE is twice that of 4ABE.
Since the sum of the heights of the two triangles is 24, then the height of 4CDE is
2
3
(24) = 16.
2007 COMC Solutions Page 9

Therefore, the area of 4DEC is 12 (18)(16) = 144.

Solution 2
From (b), the coordinates of E are (6, 16).
Therefore, the height of 4DEC is 16.
Therefore, the area of 4DEC is 12 (18)(16) = 144.
(d) Solution 1
From (c), the area of 4DEC is 144.
From Solution 2 of (a), the area of 4ACD is 216.
The area of 4DAE is the difference in these areas, or 216 − 144 = 72.

Solution 2
Using the coordinatization from (b), the coordinates of A are (9, 24), the coordinates of
E are (6, 16), and the coordinates of D are (18, 0).
Using the “up products and down products” method, the area of the triangle is

9 24


1 6 16 1
= |9(16) + 6(0) + 18(24) − 24(6) − 16(18) − 0(9)|
2 18 0
2
1

9 24
= |144 + 0 + 432 − 144 − 288 − 0|
2
= 72

3. (a) Alphonse should win.


If Alphonse starts by taking 1 stone, then by Rule #3, Beryl must remove at least 1 stone
and at most 2(1) − 1 = 1 stone. In other words, Beryl must remove 1 stone.
This in turn forces Alphonse to remove 1 stone, and so on.
Continuing in this way, Alphonse removes 1 stone from an odd-sized pile at each turn and
Beryl removes 1 stone from an even-sized pile at each turn. Thus, Alphonse removes the
last stone.
Therefore, Alphonse wins by removing 1 stone initially since 7 is odd.
(In fact, this argument shows that Alphonse should win whenever N is odd.)
(b) Beryl should win.
We make a chart in which the rows enumerate a possible combination of moves. Each
move indicates the number of stones removed.
2007 COMC Solutions Page 10

A1 B1 A2 B2 A3 B3 A4 B4 Winner
1 1 1 1 1 1 1 1 B
2 2 1 1 1 1 B
2 2 2 2 B
2 2 3 1 B
3 5 B
4 4 B
5 3 B
6 2 B
7 1 B
Thus, no matter what number of stones Alphonse removes initially, there is a move that
Beryl can make which allows her to win. (There are possible combinations of moves where
Alphonse wins that are not listed in this chart.) Therefore, Beryl should win when N = 8.
Her strategy is:
• If Alphonse removes 3 or more stones, then he can remove the remaining stones in
the pile and win.
• If Alphonse removes 1 or 2 stones, then he can win by using the table above, choosing
a row for which she wins. In effect, Beryl repeats Alphonse’s move on her first turn.
This ensures that Alphonse receives a pile with an even number of stones and that he
can remove no more than 3 stones on his next turn. Thus, she can win, as the table
shows.
(c) Solution 1
We show that Beryl has a winning strategy if and only if N = 2m , with m a positive
integer.

First, if N is odd, we know that Alphonse has a winning strategy as in (a) (Alphonse
removes 1 stone, forcing Beryl to remove 1 stone, and so on).
Second, if N = 2, then Beryl wins as Alphonse must remove 1 stone to begin, so Beryl
removes the remaining stone.

Next, we show that if N = 2k, then the player who has the winning strategy for N = k
also has a winning strategy for N = 2k. This will tell us that Beryl has a winning strategy
for N = 2, 4, 8, 16, . . . (in general, for N = 2m ) and that Alphonse has a winning strategy
if N = 2m q where q is an odd integer (since Alphonse wins for N = q, 2q, 4q, . . .). Since
every even integer can be written in one of these two forms, this will complete our proof.

So consider N = 2k.
• If either player removes an odd number of stones from an even-sized pile (leaving an
2007 COMC Solutions Page 11

odd-sized pile), then they can be forced to lose, as the other player can then remove
1 stone from an odd-sized pile and force a win as in (a). So if Alphonse removes
an even number of stones to start, then Beryl should next remove an even number
of stones (so that Alphonse can’t immediately force her to lose), so the pile size will
always remain even and each player’s move will always be to remove an even number
of stones.
• Suppose that Alphonse has a winning strategy for N = k of the form a1 , b1 , a2 , b2 , . . . , aj .
Here, we mean that Alphonse removes a1 stones on his first turn and responds to
Beryl’s first move b1 by removing a2 and so on. (Of course, a2 will depend on b1
which could take a number of values, and so on.) Since these are valid moves, then
1 ≤ a1 < k, and b1 < 2a1 , and a2 < 2b1 , and so on.
Then 2a1 , 2b1 , 2a2 , 2b2 , . . . , 2aj will be a winning strategy for Alphonse for N = 2k
since 1 < 2a1 < 2k, and 2b1 < 2(2a1 ), and 2a2 < 2(2b1 ), and so on, so this is a valid
sequence of moves and they exhaust the pile with Alphonse taking the last stone.
In other words, to win when N = 2k, Alphonse consults his winning strategy for
N = k. He removes twice his initial winning move for N = k. If Beryl removes 2b
stones next, Alphonse then removes 2a stones, where a is his winning response to
Beryl removing b stones in the N = k game. This guarantees that he will win.
• Suppose that Beryl has a winning strategy for N = k.
By an analogous argument, Beryl has a winning strategy for N = 2k, for if Alphonse
removes 2a stones, then she removes 2b stones, where b is her winning responding
move to Alphonse removing a stones in the N = k game.
Therefore, Beryl wins if and only if N = 2m , with m a positive integer.

Solution 2
We show that Beryl has a winning strategy if and only if N = 2m , with m a positive
integer.

Suppose first that N is not a power of 2.


We can write N as a sum of distinct powers of 2, in the form N = 2k1 + 2k2 + · · · + 2kj ,
where k1 > k2 > · · · > kj ≥ 0. (In essence, we are writing N in binary.) Since N is not
itself a power of 2, then this representation uses more than one power of 2 (that is, j ≥ 2).
We will show that Alphonse has a strategy where he can always reduce the number of
powers of 2 being used, while Beryl can never reduce the number of powers of 2 being
used. This will show that Alphonse can always remove the final stone, as only he can
reduce the number of powers of 2 being used to 0.
Alphonse’s strategy is to initially remove the smallest power of 2 from the representation
of N (that is, he removes 2kj stones).
2007 COMC Solutions Page 12

On her first turn, Beryl thus receives a pile with 2k1 + 2k2 + · · · + 2kj−1 stones. By rule #3,
she must remove fewer than 2(2kj ) = 2kj +1 stones. Since kj−1 > kj , then kj−1 ≥ kj + 1, so
Beryl must remove fewer than 2kj−1 stones.
When she removes these stones, the 2kj−1 will be removed from the representation of the
number of remaining stones, but will be replaced by at least one (if not more) smaller
powers of 2.
Thus, Beryl cannot reduce the number of powers of 2 in the representation.
Suppose that Alphonse thus receives a pile with 2k1 + 2k2 + · · · + 2kj−2 + 2d1 + 2d2 + · · · + 2dh
stones, with k1 > k2 > · · · > kj−2 > d1 > d2 > · · · > dh and h ≥ 1.
This means that Beryl removed B = 2kj−1 − (2d1 + 2d2 + · · · + 2dh ) stones.
But B > 0 and B is divisible by 2dh (since B = 2dh 2kj−1 −dh − (2d1 −dh + 2d2 −dh + · · · + 20


and each of the exponents initially were larger than dh ), so B ≥ 2dh .


Therefore, Alphonse can remove 2dh stones on his turn (that is, the smallest power of 2 in
the representation of the number of remaining stones) since 2dh stones satisfies Rule #3,
and so his strategy can continue.
Therefore, Alphonse has a winning strategy if N is not a power of 2.

If N is a power of 2, then Alphonse on his first turn cannot decrease the number of
powers of 2 in the representation of N . (This is a similar argument to the one above for
Beryl’s first turn.) On Beryl’s first turn, though, she can reduce the number of powers of
2 (as in Alphonse’s second turn above).
Therefore, the roles are reversed, and Beryl can always reduce the number of powers of 2,
while Alphonse cannot. Therefore, Beryl has a winning strategy when N is a power of 2.

Therefore, Beryl has a winning strategy if and only if N is a power of 2.

4. (a) Solution 1
In t seconds, the mouse runs 7t metres and the cat runs 13t metres.
Using this, we get a triangle with the cat and mouse meeting at point P .
P

13t
7t
120
C 60 M
2007 COMC Solutions Page 13

By the cosine law,

CP 2 = CM 2 + M P 2 − 2(CM )(M P ) cos(∠CM P )


(13t)2 = 602 + (7t)2 − 2(60)(7t) cos(120◦ )
169t2 = 3600 + 49t2 − 120(7t)(− 21 )
169t2 = 3600 + 49t2 + 60(7t)
120t2 − 420t − 3600 = 0
2t2 − 7t − 60 = 0
(2t − 15)(t + 4) = 0

Therefore, t = 15
2
or t = −4.
15
Since t represents a time, then t > 0, so t = 2
.

Solution 2
In t seconds, the mouse runs 7t metres and the cat runs 13t metres.
Using this, we get a triangle with the cat and mouse meeting at point P . Drop a perpen-
dicular from P to N on CM extended.
P

13t
7t

60
C 60 M N

Since ∠P M N = 60◦ , then 4P M N is a 30◦ -60◦ -90◦ triangle.


√ √
Therefore, M N = 21 P M = 72 t and P N = 3M N = 7 2 3 t.

This gives us right-angled 4CP N with CP = 13t, P N = 7 2 3 t, and CN = 60 + 72 t.
By the Pythagorean Theorem,

CP 2 = CN 2 + N P 2
 √ 2
2 7 2
(13t) = 60 + 2 t + 7 2 3 t


169t2 = 3600 + 420t + 49 2


4
t + 147 2
4
t
169t2 = 3600 + 420 + 49t 2

120t2 − 420t − 3600 = 0


2t2 − 7t − 60 = 0
(2t − 15)(t + 4) = 0

Therefore, t = 15
2
or t = −4.
15
Since t represents a time, then t > 0, so t = 2
.
2007 COMC Solutions Page 14

(b) Solution 1
We coordinatize the situation, as suggested in the diagram with C having coordinates
(−60, 0) and M having coordinates (0, 0).
Suppose that the cat intercepts the mouse at point P (x, y).
CP 13
Since the cat runs at 13 m/s and the mouse at 7 m/s, then = . Thus,
MP 7
p
(x + 60)2 + y 2 13
p =
x2 + y 2 7
(x + 60)2 + y 2 169
2 2
=
x +y 49
49((x + 60) + y ) = 169(x2 + y 2 )
2 2

0 = 120x2 − 2(49)(60)x − 49(602 ) + 120y 2


0 = x2 − 49x − 49(30) + y 2

Since this equation is of the form 0 = x2 + ax + y 2 + by + c and there is at least one


point whose coordinates satisfy the equation (for example, setting y = 0 gives a quadratic
equation with positive discriminant), then it is the equation of a circle, so all points of
interception lie on a circle. (We could also complete the square to obtain the equation
(x − 49
2
)2 + y 2 = ( 91
2
)2 , which is the equation of the circle with centre ( 49
2
, 0) and radius 91
2
.)

Solution 2
We coordinatize the situation, as suggested in the diagram, with C having coordinates
(−60, 0) and M having coordinates (0, 0).
Suppose that the cat intercepts the mouse at point P (x, y).
Suppose that the cat intercepts the mouse after t seconds and that the mouse runs in the
direction θ East of North. (θ here could be negative. We can assume that −90◦ ≤ θ ≤ 90◦
to keep the situation in the upper half of the plane. If θ did not lie in this range, then
P would be in the lower half plane and we could reflect it in the x-axis and use this
argument.)
P(x, y)

13t
7t

C 60 m M
2007 COMC Solutions Page 15

As in (a),

CP 2 = CM 2 + M P 2 − 2(CM )(M P ) cos(∠CM P )


(13t)2 = 602 + (7t)2 − 2(60)(7t) cos(90◦ + θ)
120t2 = 3600 + 120(7t) sin θ
t2 − 7t sin θ = 30

But M P 2 = 49t2 = x2 + y 2 and x = 7t cos(90◦ − θ) = 7t sin θ, so

x2 + y 2
− x = 30
49
x2 − 49x − 49(30) + y 2 = 0

so all possible points P lie on a circle, as in Solution 1.

Solution 3
We coordinatize the situation, as suggested in the diagram with C having coordinates
(−60, 0) and M having coordinates (0, 0).
Suppose that the cat intercepts the mouse at point P (x, y).
Suppose that the cat intercepts the mouse after t seconds and that the mouse runs in the
direction θ East of North. (θ here could be negative. We can assume that −90◦ ≤ θ ≤ 90◦
to keep the situation in the upper half of the plane. If θ did not lie in this range, then
P would be in the lower half plane and we could reflect it in the x-axis and use this
argument.)
P(x, y)

13t
7t

C 60 m M

If the mouse decides to run due East, then it will be caught when −60+13t = 7t or t = 10,
so will be caught at B(70, 0).
If the mouse decides to run due West, then it will be caught when −60 + 13t = −7t or
t = 3, so will be caught at A(−21, 0).
The positions above the x-axis where the mouse will be caught should be exactly symmetric
with the positions below the x-axis where the mouse will be caught. Therefore, if these
positions lie on a circle, then a diameter of this circle should lie on the x-axis.
Since the only positions on the x-axis where the mouse will be caught are A(−21, 0) and
2007 COMC Solutions Page 16

B(70, 0), then these must be endpoints of the diameter.


Therefore, the circle will have centre E with coordinates 21 (−21 + 70), 0 = 49
 
2
,0 and
radius 12 (70 − (−21)) = 91
2
.
From the diagram above, the coordinates of the point P of intersection will be

(7t cos(90◦ − θ), 7t sin(90◦ − θ)) = (7t sin θ, 7t cos θ)

If we can show that P E = 91 2


for every value of θ, then we will have shown that every
point of intersection lies on the circle with centre E and radius 91
2
.
Now
2
P E 2 = 7t sin θ − 49 2
+ (7t cos θ)2
492
= 49t2 sin2 θ − 7(49)t sin θ + 4
+ 49t2 cos2 θ
492
= 49t2 (sin2 θ + cos2 θ) − 7(49)t sin θ + 4
492
= 49t2 − 7(49)t sin θ + 4

From Solution 2, t2 − 7t sin θ = 30, so


492 72 132 91 2
P E 2 = 49(30) + 49 169
  
4
= 49 30 + 4
= 49 4
= 4
= 2

so P E = 91
2
, as required.
Therefore, all points of intersection lie on a circle.
(c) From (b), we know that the points of intersection lie on the circle with diameter AB,
where A has coordinates (−21, 0) and B has coordinates (70, 0).
Suppose that the mouse is intercepted at point P1 after running d1 metres and at point
P2 after running d2 metres.
P1

d1
21 70
A B
M
d2

P2

By the Intersecting Chords Theorem, d1 d2 = 21(70).


By the Arithmetic Mean-Geometric Mean Inequality,
d1 + d2 p p √
≥ d1 d2 = 21(70) = 7 30
2
√ √
Therefore, d1 + d2 ≥ 2(7 30) = 14 30.
2007 COMC Solutions Page 17

(The Arithmetic Mean-Geometric Mean Inequality (known as the AM-GM Inequality)


comes from the fact that if d1 and d2 are non-negative, then (d1 − d2 )2 ≥ 0.
 2
2 2 d1 + d2 d1 + d2 p
Thus, d1 + 2d1 d2 + d ≥ 4d1 d2 , so ≥ d1 d2 , so ≥ d1 d2 .)
2 2

You might also like